Geomania.Org Forumları

Fantezi Cebir => Sayılar Teorisi => Konuyu başlatan: AtakanCİCEK - Temmuz 02, 2019, 03:15:29 ös

Başlık: Diyafont Denklemler Çalışma Soruları ($138$ Tane)
Gönderen: AtakanCİCEK - Temmuz 02, 2019, 03:15:29 ös
$1.$ aşama $2.$ aşama ve uluslararası matematik olimpiyatları düzeyinde Diyafont Denklem soruları seçtim. Çözümleri de isimlerinizle beraber paylaşırsak sevinirim. Pdf haline getiriyorum.


NOT:Siteye PDF eklemeyi bilmediğim için bu şekilde gönderiyorum.


NOT 2: $78.$ soru ve $99$'den sonraki sorular daha çözülmemiştir ($99$ dahil, $102$, $104$, $105$ $109$, $110$, $113$, $118$, $134$, $136$ hariç).


$1)$ $\frac{x^3+y^3+z^3}{3}=xyz+27$ denklemini $x<y<z$ için pozitif tam sayılarda çözünüz.

$2)$ $n^4+3n^2+1$ sayısının hiçbir $n$ pozitif tam sayısı için tamkare olmadığını gösteriniz.(UMO $2.$ Aşama $1992$)

$3)$ $y^2=x^3+7$ denklemini tam sayılarda çözünüz.

$4)$ $k!+48=48.(k+1)^m$ olacak şekilde $k$ ve $m$ negatif olmayan tam sayılarının bulunmadığını gösteriniz.(Kanada Matematik Olimpiyatı)

$5)$ $2^m-3^n=7$ olacak şekilde tüm $(m,n)$ ikililerini bulunuz.(Avusturya-Polonya Matematik Olimpiyatı $1993$)

$6)$ $a^2+b=b^{1999}$ denklemini tam sayılarda çözünüz.(Estonya Matematik Olimpiyatı $1999$)

$7)$ $a^3+b^3+c^3=2001$ denklemini sağlayan kaç tane $(a,b,c)$ pozitif tam sayı üçlüsü vardır?($Balkan Junior 2001$)

$8)$ $5.(xy+xz+yz)=3xyz$ denkleminin pozitif tam sayılarda çözümlerini bulunuz.

$9)$ $x^3-y^3=xy+61$ denklemini pozitif tam sayılarda çözünüz.(Sovyet Matematik Olimpiyatı $1981$)

$10)$ $x^3+9xy+127=y^3$ denklemini sağlayan kaç tane $(x,y)$ tam sayı çifti vardır?

$11)$ $n^3+7n-133$ ifadesi pozitif bir tam sayının kübü ise $n$ sayısına ''iyi sayı'' diyelim. Tüm iyi sayıları bulunuz.(USC Math Contest)

$12)$ $n^2-19n+99$ sayısı tamkare olacak şekildeki tüm $n$ tam sayılarının toplamı kaçtır?(AIME $1999$)

$13)$ $n^4+n^3+1$ tamkare olacak şekilde tüm $n$ tam sayılarını bulunuz.

$14)$ $5n^2=36a^2+18b^2+6c^2$ denklemini tam sayılarda çözünüz.(Asya-Pasifik Matematik Olimpiyatı$1989$)

$15)$ $10x^3+20y^3+8xyz=1999z^3$ denkleminin tam sayılarda kaç çözümü vardır?(Municipial $1999$)

$16)$ $(m-n)^2=\frac{4mn}{m+n-1}$ denklemini sağlayan $0<m+n<100$ olacak şekilde kaç $(m,n)$ tam sayı çifti vardır?(Estonya Matematik Olimpiyatı $1999$)

17)Aşağıdaki denklem sistemini pozitif tam sayılar kümesinde çözünüz.(İsveç Matematik Olimpiyatı $1984$)\begin{equation*}
\begin{cases}
a^3-b^3-c^3=3abc,
\\
a^2=2.(a+b+c)
\end{cases}
\end{equation*}
 
$18)$ $2n^3-m^3=mn^2+11$ denklemini tam sayılarda çözünüz.(İsveç Matematik Olimpiyatı $1994$)

$19)$ $x,y,\frac{x^2+y^2+6}{xy}$ sayıları tam sayı ise $\frac{x^2+y^2+6}{xy}$ sayısının tamküp olduğunu gösteriniz.(Estonya Matematik Olimpiyatı $1995$)

$20)$ $m,n$ tam sayılar, $p$ bir asal sayı olmak üzere $\frac{13^m+p.2^n}{13^m-p.2^n}$ bir pozitif tam sayı olacak şekilde kaç $(m,n,p)$ üçlüsü vardır?(UMO $1.$ Aşama $2018$)

$21)$ $x$ bir asal sayı olmak üzere $x-y^4=4$ eşitliğini tam sayılarda çözünüz.

$22)$ $x^6+3x^3+1=y^4$ olacak şekilde $(x,y)$ ikililerini bulunuz.(Romanya Matematik Olimpiyatı)

$23)$ $a,b,c$ tam sayı ve $1<a<b<c$ olmak üzere, (a-1).(b-1).(c-1) in abc-1 in bölenlerinden biri olmasını sağlayan $(a,b,c)$ üçlülerini bulunuz.(IMO $1992$)

$24)$ $(x+1)^4-(x-1)^4=y^3$ eşitliğini sağlayan tüm $(x,y)$ tam sayı ikililerini bulunuz.(Avusturya Matematik Olimpiyatı)

$25)$ $p^3-q^5=(p+q)^2$ eşitliğini asal sayılar için çözünüz.(Rusya Matematik Olimpiyatı)

$26)$ $x^2-y!=2001$ denklemini pozitif tam sayılarda çözünüz.

$27)$ $5^x.7^y+4=3^z$ denklemini negatif olmayan tam sayılarda çözünüz.(Bulgaristan Matematik Olimpiyatı)

$28)$ $x^3-4xy+y^3=-1$ eşitliğini sağlayan tüm tam sayı değerlerini bulunuz.

$29)$ $x^2-y^2=2xyz$ denklemini pozitif tam sayılarda çözünüz.

$30)$ $x^2+axy+y^2=z^2$ denkleminin $(x,y,z)$ için genel çözümlerini bulunuz .

$31)$ $xy.(x^2+y^2)=2z^2$ denklemini tam sayılarda çözünüz.

$32)$ $ p^3+p^2+11p+2=q$ denklemini asal sayılarda çözünüz.

$33)$ $x^3-y^3=2y^2+1$ denklemini tam sayılarda çözünüz.

$34)$ $x^4+4^x=p$ denklemini $p$ asal sayısı ve $x$ tam sayısı için çözünüz.

$35)$ $c^2+1=(a^2-1).(b^2-1)$ denkleminin bütün çözümlerini bulunuz.

$36)$ $9^x-3^x=y^4+2y^3+y^2+2y$ denkleminin tam sayı çözümlerini bulunuz.

$37)$ $x$ ve $y$ $5$' ten büyük asal çarpanı olmayan sayılar olmayan pozitif sayılar olmak üzere $k\ge0$ olan bir $k$ tam sayısı için $x^2-y^2=2^k$ denklemini çözünüz.

$38)$ $n^2+3^n$ sayısını tamkare yapan bütün pozitif tam sayıları bulunuz.

$39)$ $a,b,m,n$ pozitif tam sayılardır. $n>1$ için  $a^n+b^n=2^m$ ise $a=b$ olduğunu kanıtlayınız.

$40)$ $2^m+3^n=k^2$ denklemini sağlayan pozitif tam sayıları bulunuz.

$41)$ $p^m.q^n=(p+q)^2+1$ olacak şekildeki tüm $(m,n,p,q)$ pozitif tam sayılarını bulunuz.

$42)$ $ab\neq1$ $a,b\ge0$ olmak üzere  $k=\dfrac{a^2+ab+b^2}{ab-1}$ ifadesi ile elde edilebilecek tüm $k\ge0$ değerlerini bulunuz.

$43)$ $x^{2006}-4y^{2006}-2006=4y^{2007}+2007y$ denkleminin pozitif tam sayılarda kaç çözümü vardır?

$44)$ $n\ge3$ koşulunu sağlayan her doğal sayı için $7x_n^2+y_n^2=2^n$ denklemini sağlayacak $x_n,y_n$  tek  bir doğal sayı ikilisinin bulunduğunu gösteriniz.

$45)$ $\varphi(n)=\varphi(2n)$ denklemini pozitif tam sayılarda çözünüz.($\varphi(n)$, $1$ ile $n$ arasında ($n$ dahil), $n$'den küçük $n$ ile aralarında asal olan pozitif tam sayıların sayısını belirtmektedir.)

$46)$ $x^4+y^4=z^2$ denkleminin pozitif tam sayılar kümesinde  çözümünün olmadığını gösteriniz.

$47)$ $a!+b^3=18+c^3$ denklemini pozitif tam sayılarda çözünüz.(UMO $1.$ Aşama $2013$)

$48)$ $m^3-n^3=9^k+123$ eşitliğini sağlayan kaç $(m,n,k)$ negatif olmayan tam sayı üçlüsü vardır?(UMO $1.$ Aşama $2014$)

$49)$ $p$ , $4p^2+1$ $6p^2+1$  birer asal sayı olacak şekildeki $p$ sayıları için $2x^3-y^3=p$ denklemini pozitif tam sayılarda çözünüz.

$50)$ $t^2+1=s.(s+1)$ denklemini tam sayılarda çözünüz.(UMO $2.$ Aşama $1994$)

$51)$ $x^3+3367=2^n$ denklemini pozitif tam sayılarda çözünüz.(UMO $2.$ Aşama $1998$)

$52)$ $3^x+11^y=z^2$ denklemini pozitif tam sayılarda çözünüz.(UMO $2.$ Aşama $2001$)

$53)$ $5^m+7^n=k^3$ eşitliğini sağlayan tüm $(m,n,k)$ negatif olmayan tam sayı üçlülerini bulunuz.(UMO $2.$ Aşama $2005$)

$54)$ $k>1$ bir sayı ve $p=6k+1$ bir sayı ve $m=2^p-1$ ise $\frac{2^{m-1}-1}{127m}$ sayısının bir tam sayı olduğunu gösteriniz.(UMO $2.$ Aşama $2007$)

$55)$ $2^n+n=m!$ denklemini $(m,n)$ pozitif tam sayıları için çözünüz.(UMO $2.$ Aşama $2013$)

$56)$ $x^3=3^y.7^z+8$ eşitliğini sağlayan $(x,y,z)$ pozitif tam sayı üçlülerini bulunuz.(UMO $2.$ Aşama $2014$)

$57)$ $m,n$ pozitif tam sayılar olmak üzere, $k=\dfrac{(m+n)^2}{4m.(m-n)^2+4}$ ifadesi tam sayı ise $k$ nın tamkare olduğunu gösteriniz.(UMO $2.$ Aşama $2015$)

$58)$ $a^2+b^2+c^2+d^2=a^2b^2c^2d^2$ denkleminin pozitif tam sayılarda kaç çözümü vardır?(UMO TSÇ $1991$)

$59)$ $a^2+b^2+3=abc$ olacak şekilde $(a,b,c)$ çözümlerini bulunuz.(UMO TSÇ $1994$)

$60)$ $n$ pozitif bir tam sayı olduğuna göre $x^2-xy+y^2=n$ denklemini sağlayan $(x,y)$ pozitif tam sayı sıralı ikililerinin sayısının $3$ ile bölündüğünü ispatlayınız.(UMO TSÇ $2000$)

$61)$ $5^x=1+4y+y^4$ eşitliğini sağlayan $(x,y)$ tam sayı ikililerini bulunuz.(UMO TSÇ $2001$)

$62)$ $m^6=n^{n+1}+n-1$ eşitliğini sağlayan $(m,n)$ pozitif tam sayı ikililerini bulunuz.(UMO TSÇ $2013$)

$63)$ $l,m,n$ pozitif tam sayılar ve $p$ bir asal sayı olmak üzere, $p^{2l-1}.m.(mn+1)^2+m^2$ bir tamkare ise $m$ nin de bir tamkare olduğunu ispatlayınız.(UMO TSÇ $2015$)

$64)$ Ondalık yazılımındaki rakamların çarpımı $x^2-10x-22$ ye eşit olan tüm $x$ doğal sayılarını bulunuz.(IMO $1968$)

$65)$ $2^n+1=n^2m$ denklemini $m$ tam sayıları ve $n>1$ tam sayıları için çözünüz.(IMO $1990$)

$66)$ $\dfrac{n^3+1}{mn-1}$ sayısının bir tam sayı olmasını sağlayan tüm $(m,n)$ sıralı pozitif tam sayı ikililerinin sayısını bulunuz.(IMO $1994$)

$67)$ $\dfrac{a^2}{2ab^2-b^3+1}$ bir pozitif tam sayı olacak şekilde kaç $(a,b)$ pozitif tamsayısı vardır?(IMO $2003$)

$68)$ $1+2^x+2^{2x+1}=y^2$ denklemini tam sayılarda çözünüz.(IMO $2006$)

$69)$ $k,n$ negatif olmayan tam sayılar ve $p$ bir asal sayı ise $5^k-3^n=p^2$ denklemini çözünüz.

$70)$ $m^4+2n^3+1=mn^3+n$ eşitliğini sağlayan tüm $(m,n)$ tam sayı ikililerini bulunuz.(Avr.Kızlar.mat.olim.TSÇ $2015$)

$71)$ $k!=(2^n-1).(2^n-2).(2^n-4)...(2^n-2^{n-1})$ denklemini sağlayan $(k,n)$ pozitif tam sayı ikililerini bulunuz.

$72)$ $7n^2=m^3+15m$ denkleminin tam sayılarda kaç çözümü vardır ? (İSBO $2019$ Ortaokul)

$73)$ $a>1$ ve $b>1$ tam sayıları için $a^{b^2}=b^a$ denklemini çözünüz.

$74)$  $2^x=3^y+5$ denklemini tam sayılarda çözünüz.  (IMO $1959-1966$ LongList)

$75)$ $x^2+y^2=(x-y)^3$  diyafont denkleminin tüm tam sayı çözümlerini bulunuz. (IMO $1971$ LongList)

$76)$ $1+x+x^2+x^3+x^4=y^4$ denklemini tamsayılarda çözünüz. (IMO $1972$  LongList)

$77)$ $p^3+m.(p+2)=m^2+p+1$ denklemini $p$ asal sayıları ve $m$  pozitif tam sayıları için çözünüz. ($AOPS$)

$78)$ $2a^4-2a^2=b^2-1$  denklemini pozitif tam sayılarda çözünüz. ($AOPS$)

$79)$ $ab+ac+bc=1$  eşitliğini sağlayan $a,b,c$ tam sayıları için $(a^2+1).(b^2+1).(c^2+1)=3^x-5^y$ eşitliğini sağlayan $x,y$ tam sayılarının bulunmasını sağlayan kaç $(a,b,c)$ üçlüsü vardır?

$80)$ $(x^2+1).(y^2+1)+2.(x-y).(1-xy)=n^2+4xy$ eşitliğini tam sayılarda çözüm sayısı kaçtır?

$81)$ $\dfrac{1}{x}+\dfrac{1}{y}=\dfrac{1}{n}$ eşitliğinin pozitif tam sayılarda çözüm sayısı kaçtır?

$82)$ $(xy-9)^2=x^2+y^2$ denklemini negatif olmayan tam sayılarda çözünüz (Hint Olimpiyatından değiştirilmiştir.)

$83)$ $x^2.(y-1)+y^2(x-1)=1$ denklemini tam sayılarda çözünüz. (Polonya Matematik Olimpiyatı)

$84)$ $x^4+4=py^4$ eşitliğini tam sayılarda çözümlü yapan tüm $p$ asallarını bulunuz. (Ion Cucurezeanu)

$85)$ $k_1+k_2+k_3+...+k_n=5n-4$ ve $\dfrac{1}{k_1}+\dfrac{1}{k_2}+\dfrac{1}{k_3}+...+\dfrac{1}{k_n}=1$ eşitliğini pozitif tam sayılar için çözünüz. (Putnam Matematik Olimpiyatı)

$86)$ $(1+\dfrac{1}{x}).((1+\dfrac{1}{y}).(1+\dfrac{1}{z})=2$ eşitliğini pozitif tam sayılar için çözünüz. (İngiltere matematik olimpiyatı)

$87)$ Tüm $n$ tam sayılarını bulunuz öyle ki $(x+y+z)^2=nxyz$ pozitif tam sayılarda çözümlü olsun. (American Mathematically Reformulation)

$88)$ $ x^2+84x+2008=y^2$  eşitliğini sağlayan pozitif $x,y$ tam sayıları için $x+y$ kaçtır? $(AIME)$

$89)$ $A,B,C$ rakamlar olmak üzere $(100A+10B+C).(A+B+C)=2005$ eşitliğini sağlayan $A$ değeri kaçtır? (AMC $12A$ $2005$)

$90)$ $71p+1$ bir doğal sayının tam karesi olacak şekildeki tüm $p$ asallarını bulunuz. (Purple Comet MS $2011$)

$91)$ $p,q,r$ asal sayılar olmak üzere $pqr=7.(p+q+r)$ olduğuna göre $p+q+r$ kaçtır?

$92)$ $n-76$ ve $n+76$ aynı anda bir pozitif sayının kübü olacak şekildeki $n$  tam sayılarını bulunuz. (Purple Comet HS $2004$)

$93)$ $a$ ve $b$ pozitif tam sayıları için

$$\log _{2} (\log_{2^a}(\log_{2^b}2^{1000}))=0$$ eşitliğini sağlayan tüm $a+b$ değerlerinin toplamı kaçtır? (AIME-1 $2013$)

$94)$ $4^y-615=x^2$ eşitliğini sağlayan $(x,y)$ ikililerini bulunuz. (American Math League $2005-2006$)

$95)$ $2.(x^2+y^2)+x+y=5xy$  denkleminin tüm $(x,y)$ tam sayı ikililerini bulunuz. (Awesome Math Test A)

$96)$ $x^3+y^3=z^3$ denkleminin tam sayılardaki genel çözümünü bulunuz.

$97)$ $x_0,x_1,...,x_{2011}$ negatif olmayan tam sayılar olmak üzere $m^{x_0}=\sum_{k=1}^{2011} m^{x_k}$ olmasını sağlayan kaç $m$ pozitif tam sayısı vardır? (AIME $2011$)

$98)$ $m$ tam sayısı için $x^3-2011x+m$ ifadesinin $3$ tam sayı kökü $a,b,c$ olsun. Buna göre $\mid a \mid+\mid b \mid+\mid c \mid$ ifadesinin değeri kaçtır? (AIME $2011$)

$99)$ $x^2+1=2y^4$ eşitliğini tam sayılarda çözünüz.

$100)$ $x^4=y^2+z^2+4$ eşitliğinin tam sayılarda çözümü olmadığını gösteriniz.

$101)$ $3^k=m^2+n^2+1$ eşitliğinin pozitif tam sayılarda sonsuz çözümü olduğunu gösteriniz (St. Petersburg)

$102)$  $x^3+117y^3=5$ denklemini tam sayılarda çözünüz.

$103)$ $(x^2+x+1).(y^2+y+1)=z^2+z+1$ eşitliğini tam sayılar için çözünüz.

$104)$ $m^3+6m^2+5m=27n^3+9n^2+9n+1$ eşitliğinin tam sayılarda çözümlerini bulunuz.

$105)$ $1!+2!+3!+...+x!=y^2$ eşitliğini tam sayılarda çözünüz.

$106)$ $x^2+y^2=2z^2$ eşitliğini pozitif tam sayılarda çözünüz.

$107)$ $x^2+y^2=3z^2$ eşitliğini pozitif tam sayılarda çözünüz.

$108)$ $x^3+3y^3=9z^3$ eşitliğinin pozitif tam sayılarda çözümü olmadığını gösteriniz.

$109)$ $x^3+8x^2-6x+8=y^3$ eşitliğini negatif olmayan tam sayılar için çözünüz.

$110)$ $19x^3-84y^2=1984$ eşitliğini tam sayılarda çözünüz.

$111)$ Eğer $n=a^2+b^2+c^2$ eşitliğini sağlayan $a,b,c$ pozitif tam sayıları var ise $n^2=x^2+y^2+z^2$ eşitliğini sağlayan $x,y,z$ pozitif tam sayıları da olacağını gösteriniz.

$112)$ $n$ pozitif tam sayısı için  $3n+1$ ve $4n+1$ aynı anda tam kare olduğuna göre $56\mid n$ olduğunu gösteriniz.

$113)$ $5m^2-6mn+7n^2=1985$ olacak şekildeki $(m,n)$ pozitif tam sayı ikililerini bulunuz. (İzlanda Matematik Olimpiyatı $1985$)

$114)$ $x^2=2^n+3^n+6^n$ eşitliğini pozitif tam sayılarda çözünüz.

$115)$ $x^3+2y^3+4z^3-6xyz=1$ eşitliğini pozitif tam sayılarda çözünüz.

$116)$ $0\le x,y,z,t \le 10^6$ olmak üzere $x^2-y^2=z^3-t^3$ eşitliğinin  tam sayı çözümlerinin sayısı $M$ ve $x^2-y^2=z^3-t^3+1$ eşitliğinin tam sayı çözümlerinin sayısı $N$ olmak üzere $M>N$ olduğunu gösteriniz. (İzlanda Matematik Olimpiyatı (?))

$117)$ $x^2+5y^2=z^2$ eşitliğini tam sayılarda çözünüz.

$118)$ $x^5-y^2=4$ eşitliğinin tam sayılarda çözümünün olmadığını gösteriniz.

$119)$ $x,y$ pozitif tam sayıları için $4xy-x-y$ ifadesinin  tam kare olamayacağını gösteriniz.

$120)$ $x^3+y^3+z^3+t^3=1999$ eşitliğinin sonsuz çözümü olduğunu gösteriniz.

$121)$ Eğer $x,y$ pozitif tam sayılar ise $x^2-y^2$ ile $x^2+y^2$ ifadelerinin aynı anda tam kare olamayacağını gösteriniz.

$122)$ $4$ tane aritmetik dizi oluşturan tam kare tam sayı veriliyor. Hepsinin birbirine eşit olması gerektiğini gösteriniz.

$123)$ $x^4-y^4=z^2$ eşitliğinin pozitif tam sayılarda çözümünün bulunmadığını gösteriniz.

$124)$ $x^4-x^2y^2+y^4=z^2$ denkleminin tam sayılarda çözümlerini bulunuz.

$125)$ $x^4+x^2+y^2+y^4=z^2$ denkleminin tam sayılarda çözümlerini bulunuz.

$126)$ $\dfrac{1}{x^2}+\dfrac{1}{y^2}=\dfrac{1}{z^2}$ denkleminin pozitif tam sayılarda çözümlerini bulunuz.

$127)$ $x^6+x^3+x^3y+y=147^{157}$ ve $x^3+x^3y+y^2+y+z^9=157^{147}$ olduğuna göre $x,y,z$ sayılarının tamamının aynı anda tam sayı olamayacağını gösteriniz. (USAMO $2005$)

$128)$ $a,b,c$ pozitif tam sayıları $a,b$ aralarında asal ve $c$ de $a$ veya $b$ ile aralarında asaldır. Buna göre $x^a+y^b=z^c$ eşitliğinin sonsuz sayıda çözümü olduğunu gösteriniz. (İzlanda Matematik Olimpiyatı $1997$)

$129)$ $x$ bir rastgele bir rasyonel sayı olsun. Buna göre  $\dfrac{a^3+b^3}{c^3+d^3}=x$ eşitliğini sağlayan $(a,b,c,d)$ çözümü daima vardır. Gösteriniz. (İzlanda Matematik Olimpiyatı $1999$)

$130)$ $x^3+x^2y+xy^2+y^3=8.(x^2+xy+y^2+1)$ eşitliğini tam sayılarda çözünüz. (engel)

$131)$ $(5+3\sqrt{2})^m=(3+5\sqrt{2})^n$ eşitliğini sağlayan $m$ ve $n$ tam sayılarını bulunuz. (engel)

$132)$ $y^2+y=x^4+x^3+x^2+x$ eşitliğini tam sayılarda çözünüz. (engel)

$133)$ $\dfrac{x^n-1}{x-1}$ ifadesi tam kare olacak şekilde $(x,n)$ tam sayılarını belirleyiniz. (Nagel- Ljunggren)

$134)$ her $a_i \in \{-1,1 \}$  olmak üzere $a_1a_2+a_2a_3+...+a_{n-1}a_n+a_na_1=0$ ise $4\mid n$ olduğunu gösteriniz. (İzlanda Matematik Olimpiyatı)

$135)$ $p$ asal sayısı ve $a,b,c,d,n$ pozitif tam sayıları için $P=a^2+nb^2=c^2+nd^2$ ise $a=c$ dir gösteriniz. (engel)

$136)$ $m,n$ tam sayıları için $\mid 12^m-5^n\mid$ şeklinde yazılabilen en küçük pozitif tam sayıyı belirleyiniz.

$137)$ $p$ asal sayısı için $(p-1)!+1$  $p$ nin bir tam kuvveti olacak şekilde $p$ sayılarını bulunuz.

$138)$ $(2x^2+5)^2=20y^2+29$ eşitliğini sağlayan $x,y$ doğal sayılarını bulunuz. ($AOPS$)
Başlık: Ynt: Diyafont Denklemler Çalışma Soruları (70 Tane)
Gönderen: AtakanCİCEK - Temmuz 18, 2019, 02:07:52 ös
$1)$ 
 Denklemi $x^3+y^3+z^3-3xyz=81$ şeklinde yazalım. Çarpanlarına ayırarak $(x+y+z).(x^2+y^2+z^2-xy-xz-yz)=81$ şeklinde yazalım. $x<y<z$ ve $x,y,z$ pozitif tam sayılar olduğundan $x+y+z\ge6$ olmalıdır.$x+y+z|81$ olduğundan $x+y+z\in\{9,27,81\}$ olmalıdır.

$a1)$ $x+y+z=9$ , $xy+xz+yz=24$ olur.

$a2)$ $x+y+z=27$ , $xy+xz+yz=242$ olur.

$a3)$ $x+y+z=81$ olursa $xy+xz+yz\notin Z$ olur. $a3$ olamaz.

$x<y$ olduğundan $x-y<0$ olduğunu aklımızda tutalım.

      $a)$ $x+y+z=9$ ve $xy+xz+yz=24$ için

            $x^2+y^2+z^2=(x+y+z)^2-2.(xy+xz+yz)$ formülünü kullanalım.

      $x^2+y^2+z^2=81-48=33$ olur ve $z\le5$ olur.
      
      eğer $x=y=z$ olsaydı bile $3z=9$ olur bu nedenle $z>3$ olmalıdır. $z\in\{4,5\}$ olmalıdır.

         $1)$ $z=4$ ise $x+y=5$ olur. $(x+y).z+xy=24$

                 $xy=4$ bulunur. Buradan $(x-y)^2=(x+y)^2-4xy$ olduğunu kullanarak

             $x-y=-3$ bulunur. buradan $(1,4,4)$ çözümü gelir ancak $x<y<z$ sağlanmaz.
   
         $2)$ $z=5$ ise $x+y=4$ olur $(x+y).z+xy=24$

            $x-y=0$ bulunur $x=y$ $x<y$ ile çelişir.

      $b)$ $x+y+z=27$ ve $xy+xz+yz=242$ için
      $x^2+y^2+z^2=(x+y+z)^2-2.(xy+xz+yz)$ formülünü kullanalım.
      $x^2+y^2+z^2=729-2.242=245$ elde edilir. $z\le15$ olmalıdır.

      $x=y=z$ olsaydı bile $3z=27$ $z=9$ bu nedenle $z>9$ olmalıdır.

$z\in \{10,11,12,13,14,15\}$ olabilir.
\begin{equation*}
\begin{cases}
x+y=17,
\\
xy=72,
\\
z=10
\end{cases}
\end{equation*}
\begin{equation*}
\begin{cases}
x+y=16,
\\
xy=66,
\\
z=11
\end{cases}
\end{equation*}

$2.$ sistemde $16^2-4.66=(x-y)^2<0$ olduğundan tam sayı çözümü yoktur.

Geriye kalan yazılabilecek sistemlerde $(x+y)^2\le225$ ve $-4xy\le-240$ olduğundan

 $(x-y)^2\le0$ çelişkisi ortaya çıkar.
$1)$ sistemini çözelim
$(x-y)^2=17^2-4.72=1$ ve $x-y\le0$ olduğundan $x-y=-1$ olur.
\begin{equation*}
\begin{cases}
x+y=17,
\\
x-y=-1,
\\
z=10
\end{cases}
\end{equation*}

denklem sisteminin çözümü diyafont denklemimizin çözümüdür ve $(8,9,10)$ olarak bulunur.
Başlık: Ynt: Diyafont Denklemler Çalışma Soruları (70 Tane)
Gönderen: AtakanCİCEK - Temmuz 18, 2019, 02:08:47 ös
$3)$ Denklemin her iki tarafına $1$ ekleyip denklemi düzenleyelim.

$y^2+1=x^3+8$

Şimdi $y^2+1$ için geçerli olan özellikler bulalım.

$y=2k$ olursa $y^2+1\equiv 1(mod4)$ olur.

$y=2k+1$ olursa $y^2+1 \equiv 2(mod4)$ olur. $(1)$

Şimdi $y$ nin $2$ dışında asal çarpanlarının tamamının $4k+1$ formunda olduğunu gösterelim.$(2)$

$p\mid y^2+1$ olsun. $y^2\equiv -1(modp)$ olur

$y^4\equiv1(modp)$

 $p\mid y^2+1$ olduğundan $(y,p)=1$ dir.

Fermat teoreminden $y^{p-1}\equiv1(modp)$

Buradan $4$'ün $p-1$ in en küçük katı olduğunu görmek mümkündür $4\mid p-1$ elde edilir.

Şimdi sorumuza geri dönelim.
$x=2m$ ,$m\in Z$
$y^2+1=(2k+2).(4k^2-4k+4)$  olur ve $(1)$ ile çelişir.

$x=2m+1$ , $m\in Z$ olsun. $x^2-2x+4\equiv3(mod4)$ olacağından ve $(2)$ den dolayı mümkün olmadığını söyleriz. Tam sayılarda çözümü yoktur.
Başlık: Ynt: Diyafont Denklemler Çalışma Soruları (70 Tane)
Gönderen: AtakanCİCEK - Temmuz 18, 2019, 02:09:56 ös
$4)$
Verilen denklemin $k$ ve $m$ negatif olmayan tam sayıları için çözümü olduğunu kabul edelim.

$k!\equiv0(mod48)$ olması gerektiği barizdir. $k\ge6$ olur.
$3.5.7.8.9.....(k-1).k+1=(k+1)^m$ şeklinde denklem yazılabilir.$(k+1)^m$ nin çarpanlarına ayrılıp ayrılamayacağını araştıralım.

$k+1$ bileşik sayı olsun ve $k+1$ sayısının asal bölenlerinden biri $p$ olsun. $p\mid k+1$ olduğundan ve $p=k+1$ olamayacağından $p<k+1$ yani $p\le k$ elde edilir. sol tarafın tek olduğunu göstermek için $k\ge8$ olduğunu gösterelim.

$k=6$ için denklem $\frac{720+48}{48}=16=7^m$ olur çözümsüzdür.

$k=7$ için denklem $106=8^m$ olur çözümsüzdür. $k\ge8$ olur.

Buradan $p$ nin tek sayı olduğu ve $k$ dan küçük eşit olduğu da açık olduğundan $p\mid 3.5.7.8.....(k-1).k$ olduğundan ve $p\mid(k+1)^m$ olduğundan $p|(k+1)^m-3.5.7.8.....(k-1).k$ yani $p\mid1$ çelişkisi ortaya çıkar. $k+1$ bir asal sayıdır.

Wilson teoreminden $k!\equiv(p-1)!\equiv-1(modp)$  yani $p\mid k!+1$ olur.

Başlangıçtaki denklem $k!+48=48p^m$ denklemini $47=48p^m-k!-1$ şeklinde yazarsak sağ tarafın $p$ ye bölünür yani $p\mid47$ elde edilir. Buradan $p=47$ olduğunu buluruz. Denklemi $p=47$ için yazalım.

$46!+48=48.47^m$ olur $46!=48.(47^m-1)$ $46!$ in asal bölenlerinden $48$ ile aralarında asal olacak şekilde seçersek $13$ $17$ seçilebilir. $47^m\equiv1(mod13)$ ve $47^m\equiv1(mod17)$ olabileceğinden Fermat teoremi yardımıyla $m\ge48$ olur. 

$48.47^m\ge48.47^{48}>46!+48$ olduğu açıktır dolayısıyla tam sayı çözümü olamaz.
Başlık: Ynt: Diyafont Denklemler Çalışma Soruları (70 Tane)
Gönderen: AtakanCİCEK - Temmuz 18, 2019, 02:11:23 ös
$5)$
$2^m-3^n=7$ ise $2^m\equiv1(mod3)$ ve $m=2x$ , $x\in Z^+$ için çözümü vardır. $m=2$ için $-3^n=3$ çözüm yoktur.

$m>2$  çift sayıları için $2^m\equiv0(mod8)$ dolayısıyla $3^n\equiv1(mod8)$ olmalıdır. dolayısıyla $n$ de çift olmalıdır. $n=2y$, $y\in Z^+$ vardır.

$2^{2x}-3^{2y}=7$

$(2^x-3^y).(2^x+3^y)=7$

$2^x+3^y\ge2$ olduğunu kullanarak sistemi yazalım.

\begin{equation*}
\begin{cases}
2^x+3^y=7,
\\
2^x-3^y=1
\end{cases}
\end{equation*}

$2^x=4$ ve $3^y=3$ bulunur. $x=2$ , $y=1$ yani $(m,n)=(4,2)$ elde edilir.
Başlık: Ynt: Diyafont Denklemler Çalışma Soruları (70 Tane)
Gönderen: AtakanCİCEK - Temmuz 18, 2019, 02:12:10 ös
$6)$

$a^2=b.(b^{1998}-1)$ yazalım. $b\ge2$ için $(b,b^{1998}-1)=1$ olduğundan dolayı $b^{1998}-1$ ifadesi tamkare olmalıdır.

Fakat $b^{1998}$ tamkare olduğu için ve $1$ den büyük tamkare sayılar arasındaki fark 3'ten büyük olduğundan dolayı $b^{1998}-1$ tamkare olamaz.

$b\le-2$ de olamaz çünkü $a^2<0$ olur. $b\in{-1,0,1}$ elde edilir.

$1)$ $b=-1$ ise $a=0$

$2)$ $b=0$ ise $a=0$

$3)$ $b=1$ ise $a=0$

Çözümleri bulunur.
Başlık: Ynt: Diyafont Denklemler Çalışma Soruları (70 Tane)
Gönderen: AtakanCİCEK - Temmuz 18, 2019, 02:12:48 ös
$7)$

$13^3>2001$ olduğundan sayılar $13$ ten küçüktür.

$8^3+8^3+8^3<2001$ olduğundan en az biri de $8$ den büyüktür.

Genelliği bozmadan $a\ge b\ge c$ alalım.

buradan $a\in 9,10,11,12$ olabilir.

$1)$ $a=9$ için $b^3+c^3=1272$ olur. $b=c=8$ olsa $1024<1272$ $b>8$ ve $a=9\ge b$ olduğundan $b=9$ olmak zorundadır. $1272-729=543$ olur $c^3=543$ denkleminin çözümü yoktur.

$2)$ $a=10$ için $b^3+c^3=1001$ olur. $b=c=7$ olsa $686<1001$ olacağı için $b\ge8$ $b\le10$ sağlanmalıdır. $b\in 8,9,10$ olabilir.

$b=8$ ise $c^3=489$ çözümü yoktur.

$b=9$ ise $c^3=272$ çözümü yoktur.

$b=10$ ise $c^3=1$ buradan $c=1$ olur.
 
$3)$ $a=11$ için $b^3+c^3=670$ olur. $b=c=6$ için $432<670$ olduğundan $b\ge7$ olmalıdır. $9^3=729$ olduğundan $b<9$ da olmaldıır. $b\in 7,8$ olabilir.

$b=7$ ise $c^3=327$ çözümü yoktur.      

$b=8$ ise $c^3=158$ olacağından çözüm yoktur.

$4)$ $a=12$ için $b^3+c^3=273$ olur. $b=c=5$ için $250<273$ olduğundan $b\ge6$ olmaldıır. $7^3=343$ olduğundan $b<7$ olur. $b=6$ olmalıdır.

$b=6$ ise $c^3=273-216=57$ çözümü yoktur.

Dolayısıyla denklemin $a\ge b\ge c$ için tek çözümü $(10,10,1)$ olduğundan çözüm sayısı bunların sıralanışları kadardır. $\frac{3!}{2!}=3$ olarak bulunur.
Başlık: Ynt: Diyafont Denklemler Çalışma Soruları (70 Tane)
Gönderen: AtakanCİCEK - Temmuz 18, 2019, 02:14:05 ös
$8)$ Denklemi $\frac{1}{x}+\frac{1}{y}+\frac{1}{z}=\frac{3}{5}$ olarak yazalım.
 Genelliği bozmadan $1< x \le y \le z$ alalım.

$x\ge6$ için eşitsizlik oluşturalım. $1\ge \frac{1}{x} \ge \frac{1}{y} \ge \frac {1}{z}$ olduğundan $\frac{1}{x}+\frac{1}{y}+\frac{1}{z}\le \frac{1}{2}$  olduğundan

$\frac{1}{x}+\frac{1}{y}+\frac{1}{z}=\frac{3}{5}$ sağlanmaz. 

$1)$ $x=5$ için ise tek çözüm $x=y=z=5$ olmalıdır.

şimdi burada $x$ in değerini seçeceğimiz için $\frac{1}{y}+\frac{1}{z}=\frac{a}{b}$ olduğunu kullanıp bir şeyler elde edelim.

öncelikle denklemi $ayz=by+bz$ şeklinde yazalım. Ardından $ayz-bz=by$, $z.(ay-b)=by$  $z=\frac{by}{ay-b}\in Z$ elde edilir.

$\frac{aby}{ay-b}\in Z$ de olması gerekir. Hatta $\frac{aby}{ay-b}+\frac{-aby+b^2}{ay-b} \in Z$ de sağlanmalıdır. Buradan $\frac{b^2}{ay-b}\in Z $ $(1)$  elde edilir.



$2)$ $x=4$ için ise $\frac{1}{y}+\frac{1}{z}=\frac{7}{20}$ olduğunu $(1)$ e uygulayalım.

$\frac{400}{7y-20}\in Z$ olduğunu kullanırsak ve  benzer şekilde $\frac{400}{7z-20}\in Z$ olduğundan ve $y\le z$ olduğundan $7y-20$ için  $\sqrt{400}$ e kadarki çarpanlarına bakmamız yeterlidir.

$7y-20$ $\in$ $A$ ,  $A\in1,2,4,5,8,10,16,20$ olmalıdır. $A$ kümesinin bir elemanı $a$ için  $7y\in 20+a$ olduğundan $a\equiv1(mod7)$ olacak şekilde seçilmelidir.

dolayısıyla $a$ nın alabileceği değerler $1,8$ dir.  Yani $y\in 3,4$ olur. $x\le y$ olduğundan yalnızca $y=4$ olmalıdır. $\frac{7}{20}-\frac{5}{20}= \frac{1}{10}$ olduğundan  $z=10$ bulunur. Çözüm $(4,4,10)$ olarak bulunur.



$3)$ $x=3$ için ise $\frac{1}{y}+\frac{1}{z}=\frac{4}{15}$ elde edilir. $(1)$ gereğince $\frac{225}{4y-15}\in Z$  olduğunu kullanıp $4y-15$ in $\sqrt{225}=15$ e kadar (dahil) olacak şekilde çarpanlarıdır. $4y-15\in 1,3,5,9,15$ olur. $y\in 4,5,6$ olduğunu görebiliriz.

$y=4$ için $z=60$ olur,

$y=5$ için $ z=15$ olur.

$y=6$ için $ z=10$ olur.




$4)$  $x=2$ için ise $\frac{1}{y}+\frac{1}{z}=\frac{1}{10}$  olur ve $(1)$ gereğince $\frac{100}{y-10}\in Z$ olmalıdır. $100$ un $10$ a kadar çarpanları $y-10$ un alabileceği değerlerdir. $y\in 11,12,14,15,20$ olabilir.

$y=11$ için $z=110$ olur.

$y=12$ için $z=60$ olur.

$y=14$ için $z=35$ olur.

$y=15$ için $z=30$ olur.

$y=20$ için $z=20$  olur.
   
$5)$ $x=1$ olamaz çünkü $\frac{1}{y}+\frac{1}{z}\le 0$ olur.

Gerçel çözümlerinin sayısı bu çözümlerin permütasyonları kadardır.

$3!+3!+3!+3!+\frac{3!}{2!}+3!+3!+3!+\frac{3!}{2!}+\frac{3!}{3!}=49$ olarak bulunur.
Başlık: Ynt: Diyafont Denklemler Çalışma Soruları (70 Tane)
Gönderen: AtakanCİCEK - Temmuz 18, 2019, 02:17:39 ös
$9)$

$Ilk$ $Yol$


 
$(x-y).(x^2+xy+y^2)=xy+61$ 
\begin{equation*}
\begin{cases}
x-y=a,
\\
xy=b
\end{cases}
\end{equation*}

$a.(a^2+3b)=b+61$ $b>0$ olduğundan $a>0$ da olmalıdır.

$a^3+3ab=b+61$

$a^3-61=b.(1-3a)$

$b=\frac{a^3-61}{1-3a}>0$  olduğunu kullanırsak  $a^3<61$ elde edilir.  $a\in 1,2,3$ olmalıdır.

$a=1$ ise $b=30$

$a=2$ ise $b\notin Z$

$a=3$ ise $b\notin Z$

Yani $x-y=1$ $xy=30$ elde ederiz  $(x+y)^2=(x-y)^2+4xy=121$ yani $x+y=11$ olur. buradan denklemin çözümü

\begin{equation*}
\begin{cases}
x=6,
\\
y=5
\end{cases}
\end{equation*}

olarak bulunur.

$Ikinci$ $Yol$

Denkleme bakıldığında $x-y>0$ olduğu açık olduğundan $x=y+z$ ,$z\in Z^+$ dönüşümü yapılabilir.

$(y+z)^3-y^3=y.(y+z)+61$

$(3z-1).y^2+(3z^2-z).y+z^3-61=0$
 
ve $z\ge1$ olduğundan $3z-1\in Z^+$ hatta $3z^2-z\in Z^+$ olduğu da yazılır.Tüm terimlerin katsayıları denklemin pozitif sayılarda çözümü olamaz. $z^3<61$ olduğunu yine söyleriz.

$x-y=z\in 1,2,3$ olduğunu yine elde ederiz.
Başlık: Ynt: Diyafont Denklemler Çalışma Soruları (70 Tane)
Gönderen: AtakanCİCEK - Temmuz 18, 2019, 02:18:58 ös
$10)$

$y-x=a$  $9x.(x+a)+127=(x+a)^3-x^3=a^3+3a^2x+3ax^2$

$(9-3a).x^2+(9a-3a^2).x+127-a^3=0$  $a=3$ olmadığı açık olduğundan

$x^2+ax+\frac{127-a^3}{9-3a}=0$

$9-3a\mid127-a^3$ ü $3-a\mid 127-a^3$ için çözelim.
Basit bir polinom bölmesiyle
$\frac{127-a^3}{3-a}=a^2+3a+9+\frac{100}{3-a}$ olur. buradan

 $a-3\in \{-100,-50,-25,-20,-10,-5,-4,-2,-1,1,2,,4,5,10,20,25,50,100\}$ olabilir. $3\mid 127-a^3$  ten dolayı $a\equiv 1(mod3)$ olur.

 $a\in \{-97,-47,-22-17,-7,-2,-1,1,2,4,5,7,8,13,23,28,53,103\}$  olur $a\equiv1(mod3)$ ten dolayı $a\in\{-47,-17,-2,1,4,7,13,28,103\} $ olabilir.

$a=-47$ için $x^2-47x+693=0$ olur $\bigtriangleup<0$ olduğundan çözüm yoktur.

$a=-17$ için $x^2-17x+84=0$ olur. $\bigtriangleup<0$ olduğundan çözüm yoktur.

$a=-2$ için $x^2-2x+9=0$ olur. $\bigtriangleup<0$ olduğundan çözüm yoktur.

$a=1$ için $x^2+x+21=0$ olur. $\bigtriangleup<0$ olduğundan çözümü yoktur.

$a=4$ için $x^2+4x-21=0$ olur. $x=-7$ ve $x=3$ çözümlerini buluruz. $(-7,-3)$ ve $(3,7)$ çözümlerdir.

$a=7$ için $a=x^2+7x+18=0$ olur. $\bigtriangleup<0$ olduğundan çözüm yoktur.

$a=13$ için $x^2+13x+69=0$ olur. $\bigtriangleup<0$ olduğundan çözüm yoktur.

$a=28$ için $x^2+28x+291=0$ olur. $\bigtriangleup<0$ olduğundan çözüm yoktur.

$a=103$ için $x^2+103x+3642=0$ $\bigtriangleup<0$ olduğundan çözüm yoktur. 

Dolayısıyla denklemin tam sayılarda $2$ çözümü vardır.
Başlık: Ynt: Diyafont Denklemler Çalışma Soruları (70 Tane)
Gönderen: AtakanCİCEK - Temmuz 18, 2019, 02:21:41 ös
$11)$

$n^3+7n-133=x^3,x\in Z^+$ şeklinde denklemi yazalım.

$n.(n^2+7)-133$ ifadesi $n<5$ için pozitif tam sayı olamaz. $n\ge5$ olmalıdır.

$7n-133=x^3-n^3$

$n=19$ için $x=19$ olacağından  $(19,19)$ çözümü olur. Şimdi $n>19$ ve $n<19$ durumlarını inceleyelim.

$1)$ $n>19$  $x^3>n^3$ olduğundan $x=n+d,d\in Z^+$ dönüşümü yapabiliriz.

$3d.n^2+(3d^2-7).n+d^3+133=0$  $n\in Z^+$ olabilimesi için $3d^2-7<0$ olmalıdır. $d^2<\frac{7}{3}$ yani $d=1$  olmalıdır.

$d=1$ olursa $3n^2-4n+134=0$  olur ve $\bigtriangleup<0$ olduğundan reel çözümü yoktur.

$2)$ $n<19$ için bakalım. $n^3>x^3$ olduğundan $n=x+y$ , $y\in Z^+$ dönüşümü yapabiliriz.

$7x+7y-133=x^3-(x+y)^3=-3x^2y-3xy^2-y^3$ olur.

$3y.x^2+(3y^2+7).x+(y^3+7y-133)=0$ elde edilir.

$y^3+7y-133\ge0$ olduğunda tüm katsayılar pozitif  veya nötr olacağından Descartes işaret değişim kuralı gereğince Pozitif sayılarda çözümü yoktur. $y^3+7y-133<0$ yani $y\le5$ olur.

$y=1$ ise $3x^2+10x-125=0=(3x+25).(x-5)$ olur.  $x=5$ ,$y=1$ yani $n=6$ olur.

$y=2$ ise $6x^2+19x-11=0=(x-3).(6x+37)$ olur. $x=3$ ,$y=2$ yani $n=5$ olur.

$y=3$ ise $9x^2+34x-85=0$ olur. $\bigtriangleup=4216$ yani $\bigtriangleup$ tam kare olmadığından tam sayı çözümü yoktur.

$y=4$ ise $12x^2+55x-41=0$ $\bigtriangleup=4993$ yani $\bigtriangleup$ tam kare olmadığından tam sayı çözümü yoktur.

o halde $n$ sayısının alabileceği değerler $n\in \{5,6,19\}$ olur.
Başlık: Ynt: Diyafont Denklemler Çalışma Soruları (70 Tane)
Gönderen: AtakanCİCEK - Temmuz 18, 2019, 02:22:26 ös
$12)$ $n^2-19n+99=x^2$ , $x\in Z^+$ olsun.

$n_{1,2}=\frac{19\pm\sqrt{361-4.(99-x^2)}}{2}$ tam sayı olabilmesi için  $361-4.(99-x^2)$ tam kare  yani $361-396+4x^2=4x^2-35$ tam kare olmaldıır. $4x^2-35=y^2,y\in Z^+$ alabiliriz.

$4x^2-y^2=35$ $(2x-y).(2x+y)=35$ $(2x+y)>0$ olduğundan $(2x-y)>0$ aynı zamanda $2x-y<2x+y$ olduğundan buna uygun denklem sistemlerini yazalım.

\begin{equation*}
\begin{cases}
2x+y=35,
\\
2x-y=1
\end{cases}
\end{equation*}

\begin{equation*}
\begin{cases}
2x+y=7,
\\
2x-y=5
\end{cases}
\end{equation*}

bu denklem sistemlerinden $y=17$ ve $y=1$  çözümleri elde edilir.

$n_{1,2}=\frac{19\pm\sqrt{361-4.(99-x^2)}}{2}=\frac{19\pm y}{2}$ ifadesinde yerine koyalım. 

$\frac{19\pm 17}{2}+\frac{19\pm 1}{2}=38$ elde edilir.
Başlık: Ynt: Diyafont Denklemler Çalışma Soruları (70 Tane)
Gönderen: AtakanCİCEK - Temmuz 18, 2019, 02:24:23 ös
$17)$

İlk Yol

$a^3-b^3-c^3=3abc$ ise  $ a^3=b^3+c^3+3abc $ ve $a^3>b^3+c^3$ elde edilir. $b$ ve $c$ simetrik olduğundan genelliği bozmadan $a>b\ge c$ olmalıdır. $a>b\ge c$ eşitsizliğinden $3a>a+b+c$ $6a>a^2$ elde edilir. yani $a<6$ eşitsizliğini elde ederiz.

$a^2=2.(a+b+c)$ olduğundan dolayı $a\equiv 0(mod2)$ olacağından  $a=2$ veya $a=4$ olmalıdır.

$1)$ $a=2$ için

$4=2.(2+b+c)$ yani $b+c=0$ elde edilir. Fakat $b,c\in Z^+$ olduğundan $b+c\ge2$ olmaldır. Dolayısıyla $a=4$ olmalıdır.

$2)$ $a=4$ ise $b+c=4$ elde edilir. aynı zamanda $a^3-b^3-c^3-3abc$ nin çarpanlarından biri $(a-b-c)$ olduğundan dolayı $1.$ denklemin doğru olmasını sağlar.

o halde $(4,1,3),(4,2,2),(4,3,1)$  şeklinde 3 adet çözüm bulunur . 

İkinci Yol

$a^3-b^3-c^3-3abc=0=(a-b-c).(a^2+b^2+c^2+ac+ab-bc)$

$1)$  $a=b+c$ ve $a^2=2.(a+b+c)$ bu nedenle $a^2-4a=0$ ve $a=4$ ve $b,c \in Z^+$ buradan gelen çözümler $(4,3,1),(4,2,2),(4,1,3)$

$2)$
\begin{equation*}
\begin{cases}
a^2+b^2+c^2=-ac-ab+bc,
\\
a^2=2.(a+b+c)
\end{cases}
\end{equation*} 

$n\in Z$ için $x_1,x_2,x_3,...,x_n\in R$ ve $y_1,y_2,y_3,...,y_n\in R$ için 

$\mid x_1.y_1+x_2.y_2+...+x_n.y_n \mid \le \sqrt{x_1^2+x_2^2+...+x_n^2}.\sqrt{y_1^2+y_2^2+...+y_n^2}$

Eşitlik yalnızca $\frac{x_1}{y_1}=\frac{x_2}{y_2}=...=\frac{x_n}{y_n}$ ile mümkündür (Cauchy-Schwarz)

$(-a).c+(-b).a+c.b\le \sqrt{a^2+b^2+c^2}. \sqrt{a^2+b^2+c^2}$

$-ac-ab+bc\le a^2+b^2+c^2$  olduğundan

$\frac{-a}{c}=\frac{c}{b}$ olması gereklidir.

$c^2=-ab$ yani  $c^2<0$ mümkün değildir..

Çözümleri $(4,3,1) ,(4,2,2),(4,1,3)$  olarak bulunur .

Üçüncü Yol

$a^3-b^3-c^3-3abc=0=(a-b-c).(a^2+b^2+c^2+ac+ab-bc)$

$1)$  $a=b+c$ ve $a^2=2.(a+b+c)$ bu nedenle $a^2-4a=0$ ve $a=4$ ve $b,c \in Z^+$ buradan gelen çözümler $(4,3,1),(4,2,2),(4,1,3)$

$2)$
\begin{equation*}
\begin{cases}
a^2+b^2+c^2=-ac-ab+bc,
\\
a^2=2.(a+b+c)
\end{cases}
\end{equation*} 

$(a-b-c)^2=-3ac-3ab+3bc$

$(a+b)^2+(a+c)^2+(b-c)^2=0$  olduğundan $a+b=0$ olmalıdır fakat mümkün değildir.
Başlık: Ynt: Diyafont Denklemler Çalışma Soruları (70 Tane)
Gönderen: AtakanCİCEK - Temmuz 18, 2019, 02:25:14 ös
$18)$

Denklemi $2n^3-m^3-mn^2=11$ şeklinde düşündüğümüzde Soldaki terimlerin dereceleri $3$ olduğundan ve katsayıları toplamı $2-1-1=0$ olduğundan dolayı  $m=n-k$ , $k\in Z$ dönüşümü yapalım.($m=n+k$ dönüşümü de uygundur.)

$2n^3-n^3+3n^2k-3nk^2+k^3=n^3-kn^2+11$

$4k.n^2-3k^2.n+k^3-11=0$ denklemi elde edilir.

$\bigtriangleup=(-3k^2)^2-4.(4k).(k^3-11)=-7k^4+176k$ olur.

$k<0$ olursa $-7k^4<0$ ve $176k<0$ olduğundan $\bigtriangleup<0$ olur $n$ tam sayısı bulunamaz.

$k=0$ olursa başlangıçtaki denklem $2n^3-n^3-n^3=11$ olur $0=11$ çelişkisi ortaya çıkar. $k\in Z^+$ olması gerektiği bulunur.

$k>0$ için $\bigtriangleup=k.(-7k^3+176)>0$ eşitsizliğini çözelim.  $-7k^3+176>0$ yani $7k^3<176$ olur. bu eşitsizliğin çözümü $k<3$ şeklindedir.

$1)$ $k=1$ ise $4n^2-3n-10=(4n+5).(n-2)=0$ yani $n=2$ elde edilir.

$2)$ $k=2$ ise $8n^2-12n-3=0$ denkleminin $\bigtriangleup$ değeri $240$ olduğundan yani tamkare olmadığından tam sayı çözümü yoktur.

Buradan denklemin tek çözümü $(m,n)=(1,2)$ olarak bulunur. 
Başlık: Ynt: Diyafont Denklemler Çalışma Soruları (70 Tane)
Gönderen: AtakanCİCEK - Temmuz 18, 2019, 02:41:05 ös
$20)$
İlk çözümdeki katkıları için metonstere teşekkür ediyorum.


İlk Yol

Soruda istenen ifadeye $a$ diyelim.

$\frac{13^m+p.2^n}{13^m-p.2^n}=a$ diyelim. İçler dışlar çarpımı yapıp düzenlersek ifademiz

$(1+a).p.2^n=13^m.(a-1)$     $4$ bilinmeyenli diyafont denklemi elde edilir. Bu ifadeden yola çıkarak

$p.2^n=13^m.\frac{a-1}{a+1}$ haline gelir. Sağ taraf birlikte düşünüldüğünde tamsayı olmalıdır.

$p.2^n=13^m-\frac{2.13^m}{a+1}$ haline gelir. Buradan

$a+1=2$,$a+1=1$,$a+1=-1$,$a+1=-2$,$a+1=13^k$, $ a+1=-13^k$,$a+1=2.13^k$,$a+1=-2.13^k$ ;($k\le m$)  eşitlikleri elde edilir. Bu ifadelerden eşitliği mümkün olan durumları bulalım.

Eşitliğin sol tarafı $0(mod2)$ olduğundan eşitliğin sağ tarafında da ifade çift olmalıdır.  $13^m\equiv1(mod2)$ olduğundan $\frac{2.13^m}{a+1}\equiv1(mod2)$ olmalıdır. Bunun için $a+1$ çift olmalıdır. $a>0$ olduğu da göz önüne alınırsa

$a+1=2$ veya $a+1=2.13^k$,$k\le m$  olabilir.

$1)$ $a+1=2$ olması durumunda $2.p.2^n=0$ olur ve $2^n>0$ ve $p>0$ olduğundan mümkün değildir.

$2)$ $a+1=2.13^k$,$k\le m$ olması durumunda $a=2.13^k-1$  olur ve

$p.2^n=13^m-13^{m-k}$ elde edilir. $m\neq k$ için ifade  $13^g.(13^t-1)$,$g\in N$ ve $t\in N$ olması gerektiğinden daima $12$ ve $13$ ile bölünür. dolayısıyla $p$ asal olamaz. $m=k$ olmalıdır.

$p.2^n=13^m-1$ Bu ifadenin içinde daima $12$ çarpanı bulanacağından ve başka tek çarpan daha  bulunacağından $m>1$ için $p$ asal olamaz

İddia: $m>1$ için $p.2^n=13^m-1$ ifadesine $13^m-1$ sayısının $2$ asal carpanı olan $p$ asal sayısı yoktur.

ispat:

Öncelikle $13^m-1$ sayısının her $m$ için $2$ ve $3$ e bölünebileceği aşikardır. Eğer $m=2n$ ise

\begin{equation*}
13^{2n}-1\equiv 169^n-1\equiv 1^n-1\equiv 0 (mod 7)
\end{equation*}

olduğundan $m$ çift ise $42$ ile bölünür ve en az $3$  asal böleni olmuş olur.

Eğer $m=2n+1$ ise farzedelim ki $13^m-1$ sayısı sadece $2$ ve $3$ ile bölünsün.

\begin{equation*}
13^{2n+1}-1\equiv 13\cdot 169^n-1\equiv 5\cdot 1^n-1\equiv 4(mod8)
\end{equation*}

olduğundan $13^m-1$ sayısı $4$ ile bölünür fakat $8$ ile bölünmez .Dolayısıyla $13^m-1=4.3^a$ formatında olmaldır. Kuvvet Kaydırma Teoremi uygulayalım,

\begin{equation*}
v_{3}(4\cdot 3^a)=a=v_{3}(13^m-1)=v_{3}(13-1)+v_{3}(m)=v_{3}(m)+1
\end{equation*}

Buradan $v_3(m)=a-1$ bulunur. Dolayısıyla $m=k.3^{a-1}$ formatında olmalıdır.

\begin{equation*}
4\cdot 3^a=13^m-1=13^{k\cdot 3^{a-1}}-1\geq 13^{3^{a-1}}-1
\end{equation*}

$a>1$ olduğunda $13^{3^a-1}-1>4.3a$ olacağı aşikardır. Dolayısıyla kabulumuz yanlıştır. Yani $m>1$ için $13^m-1$ sayısının en az $3$ asal böleni vardır.

 Bu nedenle $m>1$ için $p.2^n=13^m-1=2^n.3^{d_1}.q^{d_2}$ şeklinde yazılabileceği için $p$ asal olamaz.


$m=1$ alınmalıdır.

$p.2^n=12$ olur. $n=2$ ve $p=3$ elde edilir.

$(m,n,p)=(1,2,3)$ elde edilir.

İkinci Yol

 $a=\frac{13^m+p.2n}{13^m-p.2^n}=1+\frac{p.2^{n+1}}{13^m-p.2^n}$
$13^m-p.2^n$ olduğundan $p = 13$ olursa $p$ ye
sadeleştirdikten sonra payda tek, pay çift oluyor. $p\neq13$ ise payda $p$ ye
bölünmez ve tektir. Yani $a = 13m - p.2^n = 1$ dir. $(mod 3)$ ten p = 3
olur. $m = 1$ ve $ n = 2$  sağlar. $m \ge 2$ ve $n \ge 3$ ise
$(mod8)$ ifadesinden $m$ çift olmalıdır. Fakat bu durumda da $(mod7)$ den çelişki gelir.
Başlık: Ynt: Diyafont Denklemler Çalışma Soruları (70 Tane)
Gönderen: AtakanCİCEK - Temmuz 18, 2019, 02:42:28 ös
$21)$

$x-y^4=4$

$x=y^4+4$

$x=y^4+4y^2+4-4y^2=(y^2+2)^2-(2y)^2$ yani $(y^2-2y+2).(y^2+2y+2)=x$ olur. $y^2+2y+2$ için $\bigtriangleup<0$ ve başkatsayı pozitif olduğundan dolayı daima pozitiftir. dolayısıyla $y^2-2y+2$ de pozitiftir. Bu çarpanlardan herhangi biri $1$ olmazsa $x$ asal sayı olamaz.

$y^2-2y+2=1$ olursa $y=1$ $x=5$

$y^2+2y+2=1$ olursa $y=-1$ $x=5$ çözümleri elde edilir. $(5,1),(5,-1)$ çözümleri elde edilir.
Başlık: Ynt: Diyafont Denklemler Çalışma Soruları (70 Tane)
Gönderen: AtakanCİCEK - Temmuz 18, 2019, 02:43:13 ös
$22)$

$x>1$ sayıları için $x^6+2x^3+1<x^6+3x^3+1<x^6+4x^3+4$ olduğundan dolayı

 $(x^3+1)^2<x^6+3x^3+1<(x^3+2)^2$ olduğu için tam sayı çözümü yoktur.

$x^3<-3$ sayıları için $x^6+4x^3+4<x^6+3x^3+1<x^6+2x^3+1$  olduğundan dolayı

$(x^3+2)^2<x^3+6x+1<(x^3+1)^2$ olduğundan dolayı çözüm yoktur. Dolayısıyla $x$ $-1,0,1$ değerlerini alabilir.

$1)$ $x=-1$ ise $y^4=-1$ $y$ tam sayı olamaz

$2)$ $x=0$ ise $y^4=1$ yani $y=\pm1$ olur.

$3)$ $x=1$ ise $y^4=5$ olursa $y$ tam sayı olamaz. Dolayısıyla çözümler $(0,1)$ ve $(0,-1)$ olur.
Başlık: Ynt: Diyafont Denklemler Çalışma Soruları (70 Tane)
Gönderen: AtakanCİCEK - Temmuz 18, 2019, 02:44:10 ös
$23)$

denklemde çıkarma işleminden  kurtulmak için bir dönüşüm yapalım. $a-1=x$ $b-1=y$ $c-1=z$ $xyz\mid (x+1).(y+1).(z+1)-1$ $x<y<z$ ve $x,y,z\in Z^+$ olur.

$xyz\mid (xy+x+y+1).(z+1)-1$

$xyz\mid xyz+xy+xz+yz+x+y+z+1-1$

$xyz\mid xy+xz+yz++x+y+z$ elde edilir. Buradan

$\frac{xy+xz+yz+x+y+z}{xyz}\in Z^+$ elde edilir. İfadeyi düzenlersek $\frac{1}{x}+\frac{1}{y}+\frac{1}{z}+\frac{1}{xy}+\frac{1}{xz}+\frac{1}{yz}$  olur. $x<y<z$ olduğundan

 $\frac{1}{x}>\frac{1}{y}>\frac{1}{z}>\frac{1}{xy}>\frac{1}{xz}>\frac{1}{yz}$ olur. Bu da bize $x$ yerine değer seçerek  İfadenin maksimum değerini bulma şansı verir. yani seçebileceğimiz en küçük üçlüyü seçerek işe koyulalım.

$(x,y,z)=(1,2,3)$ ise $1+\frac{1}{2}+\frac{1}{3}+\frac{1}{6}+\frac{1}{2}+\frac{1}{3}$ olur. Bu ise $2+\frac{5}{6}$ yani  ifadenin $3$ ten kesinlikle küçük olacağını söyler.

$(x,y,z)=(2,3,4)$ ise $\frac{1}{2}+\frac{1}{3}+\frac{1}{4}+\frac{1}{6}+\frac{1}{8}+\frac{1}{12}=\frac{35}{24}<2$ olur. Yani $x=2$ için ifadenin değeri $1$ dir.

$x\ge3$ için ise bu ifade en yüksek değerini $(3,4,5)$ için alır.

$\frac{1}{3}+\frac{1}{4}+\frac{1}{5}+\frac{1}{12}+\frac{1}{20}+\frac{1}{15}=\frac{59}{60}<1$   olur. bu da $x\ge2$  için çözüm olmadığını söyler.  Dolayısıyla $x=1$ için ifade $1$ veya $2$  olabildiğinden ve $x=2$ için ifadenin değeri $1$ olduğundan $3$ farklı durumda incelemek yeterlidir.

$1)$ $x=1$ için ifadenin değeri $1$ ise

$2y+2z+1+yz=yz$ 

$2y+2z+1=0$ olur. ki $2y+2z>0$ olduğundan doalyı mümkün değildir.

$2)$ $x=1$ için ifadenin değeri $2$ ise

$2y+2z+1=yz$ 

$2z+1=y.(z-2)$

$y=\frac{2z+1}{z-2}$

$\frac{2z+1+4-2z}{z-2}\in Z$ yani $z-2\mid 5$ olur.  $z>2$ olduğundan dolayı  $z\in \{ 3,7 \}$ olur.

$z=3$ ise $y=7$ olur. $z>y$ koşulu sağlanmadığından dolayı çözümü yoktur.

$z=7$ ise $y=3$ olur.  bu bir çözümdür. $(x,y,z)=(1,3,7)$ olduğundan $(a,b,c)=(2,4,8)$

$3)$ $x=2$ için ifadenin değeri 1 ise

$3z+3y+2+yz=2yz$

$3y+3z+2=yz$

$3z+2=y.(z-3)$

$\frac{3z+2}{z-3}\in Z$

$\frac{3z+2+9-3z}{z-3}\in Z$

 $z>2$ olduğundan ve $z\not\equiv 3$ olduğu için $z-3>0$ olmalıdır.

$\frac{11}{z-3}\in Z^+$ ve $Z\in \{4,14\}$ olmalıdır.

$z=4$ ise $y=14$ olur bu $z>y$ ile çelişir.

$z=14$ ise $y=4$ olur bu durumda $(x,y,z)=(2,4,14)$ bir çözümdür ve $(a,b,c)=(3,5,15)$ olarak bulunur.

bu bölme işlemini sağlayan üçlüler $(2,4,8)$ ve $(3,5,15)$  olmak üzere $2$ tanedir.
Başlık: Ynt: Diyafont Denklemler Çalışma Soruları (70 Tane)
Gönderen: AtakanCİCEK - Temmuz 18, 2019, 02:45:12 ös
$24)$

$(x+1)^4-(x-1)^4=y^3$

$8x^3+8x=y^3$  yani $x^3+x$ tam küp olmalıdır. 

$x\ge 1$ için $x^3<x^3+x<x^3+3x^2+3x+1$ yani $x^3<x^3+x<(x+1)^3$ olduğundan çözüm yoktur.

$x\le -1$ için $x^3+3x^2+3x+1<x^3+x<x^3$ yani $(x+1)^3<x^3+x<x^3$ olduğundan dolayı  çözüm yoktur.

$x=0$ için $y=0$ çözümü bulunur. Denklemin tek çözümü $(0,0)$ olur.
Başlık: Ynt: Diyafont Denklemler Çalışma Soruları (70 Tane)
Gönderen: AtakanCİCEK - Temmuz 18, 2019, 02:45:53 ös
$25)$

Varsayalım ki $p,q$ değerlerinin her ikisi de $3$ ten farklı olsun.

$3\mid p+q$ ancak ve ancak  $3\mid p^3-q^5$ olmalıdır.

$p,q\equiv\pm1(mod3)$ olur.

$1)$ $p\equiv q(mod3)$ ise $3\mid p^3-q^5$ olur ancak $3\mid p+q$ olmaz.

$2)$ $p\not\equiv q(mod3)$ ise $3\mid p+q$ olur ancak $3\mid p^3-q^5$ olmaz.

O halde $p$ veya $q$ $3$ olmalıdır. $p=3$ ise $27-q^5>0$ olduğundan $q=1$ olmalıdır. Fakat denklemi sağlamaz.

$q=3$ ise $p^3-243=(p+3)^2$ olur. $p>7$ için $p^3-243>(p+3)^2$ olur. $p<7$ için ise $p^3-243<(p+3)^2$ olur o halde yalnızca $p=7$ olabilir.

Denklemin çözümü $(7,3)$ olarak bulunur .
Başlık: Ynt: Diyafont Denklemler Çalışma Soruları (70 Tane)
Gönderen: AtakanCİCEK - Temmuz 18, 2019, 02:46:39 ös
$26)$

$x^2-y!=2001$ denklemini  $x^2-2001=y!$ şeklinde yazalım. ardından denklemi $(mod9)$ altında inceleyelim.

$-2001\equiv 6(mod9)$ olduğunu not alalım.

$x^2\equiv \{0,1,4,7\} (mod9)$  olmalıdır.

$x^2+6\equiv y!\equiv  \{6,7,10,13\}(mod9)$ olduğu için $y!\not \equiv 0(mod9)$ olmalıdır. O halde $y<6$ olmalıdır.

$y\in \{1,2,3,4,5\}$ olur.

öncelikle $x^2\not \equiv \{2,3,7,8\}(mod10)$ ifadesine $(1)$ diyelim.

$y=1$ ise $x^2=2002$ olur $x^2\equiv 2(mod10)$ $(1)$ olamayacağından çözüm gelmez.

$y=2$ ise $x^2=2003$ olur $x^2\equiv 3(mod10)$ $(1)$ olamayacağından çözüm gelmez.

$y=3$ ise $x^2=2007$ olur $x^2\equiv 7(mod10)$ $(1)$ olamayacağından çözüm gelmez.

$y=4$ ise $x^2=2025$ olur $x=45$ olarak bulunur.

$y=5$ ise $x^2=2121$ olursa  $x^2=21.101$ olur. tam kare olamayacağından çözüm gelmez.

O halde denklemin tek çözümü $(45,4)$ olarak bulunur.
Başlık: Ynt: Diyafont Denklemler Çalışma Soruları (70 Tane)
Gönderen: AtakanCİCEK - Temmuz 18, 2019, 02:47:32 ös
$28)$

Denklemde genelliği bozmadan $x\ge y$ alalım. Ve $x+y=k$ , $k\in Z$ dönüşümü yapalım.

$x^3-4x.(k-x)+(k-x)^3=-1$

$x^3-4xk+4x^2+k^3-3k^2x+3kx^2-x^3=-1$

$(3k+4).x^2+(-3k^2-4).x+k^3+1=0$

$\bigtriangleup=(-3k^2-4)^2-4.(3k+4).(k^3+1)$

$9k^4+24k^2+16-12k^4-12k-16k^3-16$

$\bigtriangleup=-3k^4-16k^3+24k^2-12k$ olur.

$k\ge3$ için $k^2>8$ yani $3k^4>24k^2$ olduğundan dolayı $\bigtriangleup<0$ olur.

$k<3$ olmalıdır.

$k\le -7$ için ise $\bigtriangleup \le-65<0$ olur.

$k\in \{-6,-5,-4,-3,-2,-1,0,1,2\}$ olmalıdır.

$k=2$ için $\bigtriangleup =-48-128+96-24<0$ olduğundan çözüm yoktur.

$k=1$ için $\bigtriangleup =-7<0$

$k=0$ için $\bigtriangleup =0$ olur denenmelidir.

$k=-1$ için $\bigtriangleup=49$ olur tam kare olduğundan sağlayabilir.

$k=-2$ için $\bigtriangleup=200$ olur tam kare olmadığından çözüm yoktur.

$k=-3$ için $\bigtriangleup =441$ olur tam kare olduğundan sağlayabilir.

$k=-4$ için $\bigtriangleup =688$ olur tam kare olmadığından çözüm yoktur.

$k=-5$ için $\bigtriangleup =785$ olur tam kare olmadığından çözüm yoktur.

$k=-6$ için $\bigtriangleup =504$ olur tam kare olmadığından çözüm yoktur.

O halde $k\in \{0,-1,-3\}$ şeklinde kümemizi elde ederiz.

$1)$ $k=0$ ise $4x^2-4x+1=0$ $x\not\in Z$ olduğundan çözümü yoktur.

$2)$ $k=-1$ ise $x^2-7x=0$ $x\in \{0,7\}$ olabilir.

$3)$ $k=-3$ ise $-5x^2-31x-26=0$ $5x^2+31x+26=0$ olmaldıır. $x=-1$ olabilir.

Olası çözümler $x>y$ için $(0,-1),(7,-8),(-1,-2)$ olur. Verilen denklemlde denersek  sadece $(0,-1)$ ilk denklemi sağlar

Denklemin çözüm kümesi $\{ (0,-1),(-1,0)\}$ olarak bulunur.
Başlık: Ynt: Diyafont Denklemler Çalışma Soruları (70 Tane)
Gönderen: AtakanCİCEK - Temmuz 18, 2019, 02:48:23 ös
$32)$

$p>1$ olduğundan dolayı $p^3+p^2+11p+2>15$ olmalıdır. 

tam sayılar 6 ya bölündüklerinde $\{6k,6k+1,6k+2,6k+3,6k+4,6k+5\}$ olabilir.

$p=6k$ olursa $p$ asal olamaz.

$p=6k+1$  olursa sonsuz tane $p$ vardır.

$p=6k+2$ olursa $p$ asal ise $p=2$ olmalıdır.

$p=6k+3$ olursa $p$ asal ise $p=3$ olmalıdır.

$p=6k+4$ olursa $p$ asal olamaz.

$p=6k+5$ olursa sonsuz tane $p$ vardır.

$1)$   $p=6k+1$ için $q=p^3+p^2+11p+2\equiv 3(mod6)$ ve $q>3$ olduğundan çözümü yoktur.

$2)$   $p=6k+5$ için $q=p^3+p^2+11p+2\equiv 3(mod6)$ ve $q>3$ olduğundan çözümü yoktur.

$3)$  $p=2$ için $36=q$ olur $q$ asal değildir.

$4)$ $p=3$ için $71=1$ olur $q$ asaldır.

Tek çözümü $(3,71)$ olarak bulunur.
Başlık: Ynt: Diyafont Denklemler Çalışma Soruları (70 Tane)
Gönderen: AtakanCİCEK - Temmuz 18, 2019, 02:48:59 ös
$33)$

$x^3-y^3$ ifadesi görüldüğünde yapılan klasik $x=y+m$ dönüşümünü yapalım. $x^3-y^3=2y^2+1>0$ olduğundan $x^3>y^3$ yani $x>y$ olur.  Bu nedenle $m\in Z^+$ olmalıdır.

$(y+m)^3-y^3=2y^2+1$

$3my^2+3m^2y+m^3=2y^2+1$

$(3m-2).y^2+3m^2y+m^3-1=0$

$\bigtriangleup=9m^4-4.(m^3-1).(3m-2)=-3m^4+8m^3+12m-8$

$m\ge 4$ için $\bigtriangleup\le-84<0$  olur. O halde $m<3$ olmalıdır. $m\in \{1,2,3\}$

$m=1$ ise $y^2+3y=0$ olur ve $y=0$ ve $y=-3$ çözümleri oluşur. $(1,0)$ ve $(-3,-2)$

$m=2$ ise $4y^2+12y+7=0$ olur tam sayılarda çözümü yoktur.

$m=3$ ise $7y^2+27y+26=(y+2).(7y+13)=0$ olur. $y=-2$ çözümü çıkar. $(1,-2)$

Dolayısıyla denklemin çözümleri $\{(1,0),(1,-2),(-3,-2)\}$ olur.
Başlık: Ynt: Diyafont Denklemler Çalışma Soruları (70 Tane)
Gönderen: AtakanCİCEK - Temmuz 18, 2019, 02:49:44 ös
$34)$

$x=1$ için $p=5$ açık çözümdür.

Soruya baktığımızda amacımızın  $x^4+4^x=p$ ifadesini çarpanlarına ayırabilmek olabileceği akla gelmelidir.  $x>1$ için çarpanlara ayıralım.

varsayalım ki $x=2k$ ,$k\in Z^+$  olsun.

$16k^4+4^2k=p$ olur ve buradan $p$ $4$ ile bölüneceğinden dolayı $x=2k$ olamaz.

$x$ in tek olduğunu bulduk şimdi ise $4^x\equiv 4(mod5)$ ancak ve ancak $x=2k+1$  olduğuna dikkat çekelim.

$x^4\equiv\{0,1\} (mod4)$ olduğu açıktır.

$x^4+4^x\equiv 0(mod5)$ olmaması için $x^4\equiv 0(mod5)$ olur. yani $x\equiv 0(mod5)$ olmalıdır.  Aynı anda sayı tek olduğundan dolayı $x=10k+5$ olarak bulunur.

$x^4+4.4^{10k+4}$ şeklinde yazıp  $2^{5k+2}=y$ dönüşümü yaparsak

 $x^4+4y^4=x^4+4x^2y^2+4y^4-4x^2y^2=(x^2-2xy+2y^2).(x^2+2xy+2y^2)$  olur.

Dolayısıyla denklemin tek çözümü $(1,5)$ olur.
Başlık: Ynt: Diyafont Denklemler Çalışma Soruları (70 Tane)
Gönderen: AtakanCİCEK - Temmuz 18, 2019, 02:51:07 ös
$35)$

İlk Yol

denklemde bilinmeyenlerin dereceleri çift olduğundan dolayı genelliği kaybetmeden $a\ge0$ ,$b\ge0$ ve $c\ge 0$ alınabilir.

$c^2+1=(a^2-1).(b^2-1)$ denkleminde $a=b=c=0$ açık çözümü olduğu görülebilmektedir.

$a,b,c>0$ olduğunu kabul edelim.

Şimdi $c^2+1$ için geçerli olan özellikler bulalım.

$c=2k$ olursa $c^2+1\equiv 1(mod4)$ olur.

$y=2c+1$ olursa $c^2+1 \equiv 2(mod4)$ olur. $(1)$

Şimdi $c$ nin $2$ dışında asal çarpanlarının tamamının $4k+1$ formunda olduğunu gösterelim.$(2)$

$p\mid c^2+1$ olsun. $c^2\equiv -1(modp)$ olur

$c^4\equiv1(modp)$

 $p\mid c^2+1$ olduğundan $(c,p)=1$ dir.

Fermat teoreminden $c^{p-1}\equiv1(modp)$

Buradan $4$'ün $p-1$ in en küçük katı olduğunu görmek mümkündür $4\mid p-1$ elde edilir.

yani $p=4k+3$ formunda çarpan ya da sayı $4$ ün katı olamaz.

$(a^2-1).(b^2-1)$ ifadesine bakarsak $a$ ya da $b$ çift olursa $c^2+1$ ifadesinin $4k+3$ formunda çarpanı olacağı için mümkün değildir.

$a$ ya da $b$ den biri tek olursa da bu sefer sayı $4$ ün katı olacağı için mümkün değildir. O halde tek çözüm $(0,0,0)$ olarak bulunur.

İkinci Yol

denklemde bilinmeyenlerin dereceleri çift olduğundan dolayı genelliği kaybetmeden $a\ge0$ ,$b\ge0$ ve $c\ge 0$ alınabilir.

$c^2+1=(a^2-1).(b^2-1)$ $(0,0,0)$ çözümü açıktır.

$c^2+1$ ifadesi $4$ ün katı olamayacağından dolayı $a$ ve $b$ tek sayılar olamaz.

$a=2a_1$ ve $b=2b_1$ olacak şekilde $a_1\in Z^+$ ve $b_1 \in Z^+$ olmalıdır.

denklemi yazıp tekrar düzenleyelim.

$c_1^2=4a_1^2b_1^2-a_1^2-b_1^2$

Bu denklemi de $(mod4)$ altında inceleyecek olursak eğer $a_1$ tek sayı ise $a_1^2\equiv 1(mod4)$ olur.  $a_1$ veya $b_1$ den herhangi biri veya ikisi tek olursa $c^2\equiv \{2,3\}(mod4)$ olacağından dolayı mümkün değildir.

o halde $a_1=2a_2$ $b_1=2b_2$ ve $c_1=2c_2$ olur. Bunu da düzenleyecek olursak $c_2^2=16a_2^2b_2^2-a_2^2-b_2^2$ olur. ve buradan sonsuz dizi elde edilir.

$c>c_1>c_2>c_3>...>0$ şeklinde bir sonsuz dizi elde edilir. Fakat  $c$ bir tam sayı olduğu için $(0,c)$ aralığında sonlu sayıda tam sayı olacağından sonsuz tam sayı dizisi mümkün değildir.

O halde denklemin tek çözümü $(0,0,0)$ olarak bulunur.
Başlık: Ynt: Diyafont Denklemler Çalışma Soruları (70 Tane)
Gönderen: AtakanCİCEK - Temmuz 18, 2019, 02:51:32 ös
$36)$

$9^x-3^x$ ifadesini $3^x=a$ ,$a\in Z^+$ şeklinde düzenleyelim.

$a^2-a=y^4+2y^3+y^2+2y$ , $4a^2-4a+1=4y^4+8y^3+4y^2+8y+1$ şeklinde yazılabilir.

$(2a-1)^2=4y^4+8y^3+4y^2+8y+1$

$(2y^2+my+1)^2=4y^4+4my^3+(4+m^2).y^2+2my+1$ olur. yukarıdaki polinom ifade ile $y^3$ lü terimlerin katsayılarını eşitlersek $m=2$ almamız gerektiğini görürüz.

$(2y^2+2y)^2=4y^4+8y^3+4y^2$

$(2y^2+2y+1)^2=4y^4+8y^3+8y^2+4y+1$  ifadeleri $y>1$ için açıkça  $(2y^2+2y)^2<4y^4+8y^3+4y^2+8y+1<(2y^2+2y+1)^2$ olduğundan  dolayı tam kare olması mümkün değildir.

$y=0$ veya $y=1$ olmalıdır.

$1)$ $y=0$ ise $9^x-3^x=0$ $3^x.(3^x-1)=0$ yani $x=0$ çözümü elde edilir. $(0,0)$ olur.

$2)$  $y=1$ ise $a^2-a=6$ $a^2-a-6=$ $a>0$ olduğundan $a=3$,$3^x=3$, $x=1$ çözümü elde eidlir. $(1,1)$ olur.

O halde denklemin çözüm kümesi $\{(0,0),(1,1)\}$ olarak bulunur.
Başlık: Ynt: Diyafont Denklemler Çalışma Soruları (70 Tane)
Gönderen: AtakanCİCEK - Temmuz 18, 2019, 02:52:11 ös
$37)$

$(x-y).(x+y)=2^k$ olduğundan dolayı $x+y=2^m$ , $x-y=2^n$ olur.

$x=2^{m-1}+2^{n-1}$ ve $y=2^{m-1}-2^{n-1}$ olarak bulunur.

burada $y$ tek bir pozitif tam sayı olduğunu kullanırsak  $m=1$ ya da $n=1$ olmalıdır.

$m=1$ olursa $y=1-2^{n-1}\le0$ ki pozitif olmasıyla çelişir. O halde $n=1$ olmalıdır.

$x-y=2$ elde edileceğinden dolayı $x$ ve $y$ aynı anda ortak  asal çarpan içeremez

$x=3^k$, $y=3^l$ veya $x=5^k$ ,$y=3^l$ olmalıdır.

$x=3^k$ ,$y=5^l$ için $5^l=2^{m-1}-1$ olacağını görürüz. $(mod4)$ altında bakarsak

$2^{m-1}\equiv 2(mod4)$ olur. $m=2$ tek çözümüdür. $m=2$ ve $n=1$ yerine koyulduğunda $(x,y)=(3,1)$ ortaya çıkar.

$x=5^k$ ve $y=5^l$ ise $3^m\equiv\{1,3\}(mod8)$ olmalıdır.

$2^{m-1}=\equiv{2,4}(mod8)$ olduğundan dolayı $m=2$ ya da $m=3$ olabilir.

$m=2$ için $x=3$ olur fakat $x=5^k$ formunda kabul etmiştik.

$m=3$ için $x=5$ ve $y=3$ olur.  O halde $(5,3)$ te bir çözümdür.

Denklemin çözüm kümesi $\{(3,1),(5,3)\}$ olarak bulunur.
Başlık: Ynt: Diyafont Denklemler Çalışma Soruları (70 Tane)
Gönderen: AtakanCİCEK - Temmuz 18, 2019, 02:53:10 ös
$47)$

$x^3\equiv -1,0,1 (mod7)$ olduğunu kullanalım. $18+c^3-b^3\in 2,3,4,5,6(mod7)$ elde edilebilir. yani $a!\not\equiv 0(mod7)$ olduğundan dolayı $a<7$ olduğunu buluruz.

$1)$ $a=2$ için $b^3-c^3=16$ olur.  $c\ge 2$ için $b>c$ olduğundan dolayı en az $b=c+1$ olabilir. bunun için ise $3c^2+3c+1$ $c\ge2$ için çözümsüzdür. $c=1$ olması durumunda ise $7=16$ olacağından dolayı çözümü yoktur. $c+2$ olsa idi $6c^2+12c+8>16$ olacağından dolayı $a=2$ için çözümü yoktur.

$2)$ $a>2$ için $3\mid b^3-c^3$ olduğundan dolayı $9\mid b^3-c^3$ te olması gerektiğinden dolayı $a\ge6$ olması gerektiği bulunur. Dolayısıyla $a=6$  olmalıdır.

$a=6$ için $c^3-b^3=702=2.3^3.13$ olur. $(c-b).(c^2+bc+b^2)=702$ yani $(c-b).[(c-b)^2+3bc]$ $3\mid c-b$ olur. dolayısıyla $3x=c-b$ , $x\in Z^+$ vardır.  $3x.(9x^2+3bc)=2.3^3.13$ olur. ve $x.(3x^2+bc)=2.3.13$ olarak düzenlenir. $x>3$ için $3x^2+bc>27$ olur ve
$x.(3x^2+bc)>81$ yani $x.(3x^2+bc)>78$ olduğundan çözüm yoktur. $x=1$ veya $x=2$ olmalıdır.

$x=1$ için $1.(3+bc)=78$ $bc=75$ $(c-b)=3.1$ $b.(b+3)=78$ olur ancak çözüm gelmez.

$x=2$ için $2.(12+bc)=78$ $12+bc=39$ $bc=27$ olur. $c-b=3.2$ $bc=27$ $b.(b+6)=27$  $b^2+6b-27=0$ yani $b=3$ $c=9$ çözümü gelir.

O halde denklemin çözümü $(6,3,9)$ olmalıdır.   
Başlık: Ynt: Diyafont Denklemler Çalışma Soruları (70 Tane)
Gönderen: AtakanCİCEK - Temmuz 18, 2019, 02:53:40 ös
$48)$

$x^3\equiv -1,0,1(mod3)$ olduğunu ve denklemden dolayı $k>0$ için  $m^3-n^3\equiv 6(mod9)$ olmalıdır. ancak $m^3-n^3\in -2,-1,0,1,2(mod9)$ olduğundan dolayı denklem sağlanamaz. $k=0$ olmalıdır.

$m^3-n^3=124$ denklemini çözmemiz yeterlidir.

$(m-n).[(m-n)^2+3mn]=124$ 

\begin{equation*}
\begin{cases}
m-n=1,
\\
(m-n)^2+3mn=124
\end{cases}
\end{equation*}

\begin{equation*}
\begin{cases}
m-n=2
\\
(m-n)^2+3mn=62
\end{cases}
\end{equation*}

\begin{equation*}
\begin{cases}
m-n=4,
\\
(m-n)^2+3mn=31
\end{cases}
\end{equation*}

Bu sistemlerden ilkini çözmek istersek $3mn=123$ yani $mn=39$ olur $39$ iki ardışık sayının çarpımı olarak yazılamayacağından dolayı  ilk sistemin çözümü yoktur.

$2.$ sistemde $3mn=58$ olduğundan dolayı $mn\notin Z$ olur ikinci sistemin de çözümü yoktur.

$3.$ sistemde ise $3mn=45$ $mn=15$ olur. $n.(n+4)=5$ $n^2+4n-5=0$ $n=1$ çözümü olur. $m=5$ te olması gerektiğinden dolayı $(5,1,0)$  denklemin tek çözümü olur.
Başlık: Ynt: Diyafont Denklemler Çalışma Soruları (70 Tane)
Gönderen: AtakanCİCEK - Temmuz 18, 2019, 02:54:52 ös
$50)$

$t^2+1=s.(s+1)$ ifadesini $t^2=s^2+s-1$ şeklinde düşünürsek   $s^2<s^2+s-1$ olması için $0<s-1$ ,   $1<s$ olmalıdır.

$s^2+s-1<s^2+2s+1$ olması için ise $s-1<2s+1$ yani $-2<s$     olmaldıır.  O halde $s>1$ için

$s^2<s^2+s-1<s^2+2s+1$ yani $(s)^2<t^2<(s+1)^2$  olacağından ardışık iki tam sayının karesi arasında başka bir tam sayının karesi

bulunamaz.  O halde $s\le 1$ olmalıdır. Soruda verilen $s\ge 1$ ifadesinden dolayı $s=1$ olur.

$t^2+1=1.2$ yani $t^2=1$ buradan ise $t\ge 1$ olduğu için  çözüm kümemiz $\{(1,1)\}$ olarak bulunur.
Başlık: Ynt: Diyafont Denklemler Çalışma Soruları (70 Tane)
Gönderen: AtakanCİCEK - Temmuz 18, 2019, 02:55:52 ös
$52)$


Denkleme $(mod8)$ altında bakalım. $3^x+11^y\equiv \{2,4,6\}(mod8)$ , aynı zamanda $z^2\equiv \{0,1,4\} (mod8)$ olduğundan $3^x+11^y\equiv 4(mod8)$ olmalıdır.

Bu denkliğin sağlanması için $x$ veya $y$ den biri çift , diğeri tek sayı olmalıdır.

$1)$ $x=2p$,$p\in Z^+$  olsun.

$(z-3^p).(z+3^p)=11^y$ olur.

Şimdi  buradan her ikisinin de $11$ in kuvveti olacağını görelim ve $z-3^p<z+3^p$ olduğundan dolayı  $z-3^p=11^k$ olursa $z+3^p=11^{k+l}$ , $k,l\in Z^+$ olacak şekilde yazılabilir . 

o halde $z+3^p-(z-3^p)=11^{k+l}-11^k=2.3^k$ olacağından dolayı sol taraf $10$ un katı iken sağ taraf $10$ un katı değildir. O halde çözüm yoktur.

$2)$ $y=2q$ , $q\in Z^+$ olsun.

$3^x=(z-11^q).(z+11^q)$

$a,b>0$ için 

$z-11^q=3^a$

$z+11^q=3^{a+b}$

$2.11^q=3^{a+b}-3^{a}$

$2.11^q=3^a.(3^b-1)$  sol taraf $3$ ün katı olmadığı için $a=0$ olmalıdır.

$3^b\equiv1(mod11)$ olduğundan $b=5w$, $w\in Z^+$ vardır.

$2.11^q=3^b-1$  $3^b-1=(3^w-1).(3^{4w}+3^{3w}+3^{2w}+3^w+1)=2.11^q$ olur.  Ve $3^w\equiv 1(mod11)$ olduğundan $3^{w.ı}\equiv1(mod11)$ , $ı \in Z^+$ $(*)$ olmalıdır.

Aynı zamanda $3^w-1<3^{4w}+3^{3w}+3^{2w}+3^w+1$ olduğundan dolayı $3^w-1=2.11^j$, $3^{4w}+3^{3w}+3^{2w}+3^w+1=11^{h}$, $j,h \in N$ ifadesinin

 ve $(*)$ dan dolayı $3^{4w}+3^{3w}+3^{2w}+3^w+1\equiv 5(mod11)$ olur ve $j=0$ dışında çözümün olmadığı anlaşılır.

$j=0$ ise $w=1$ yani $b=5w=5$ $a=0$ bulunur.

\begin{equation*}
\begin{cases}
z-11^q=3^0=1
\\
z+11^q=3^{0+5}=243
\end{cases}
\end{equation*}


Buradan $z=122$ , $y=2m=4$ , $3^x=3^{2a+b}$ yani $x=5$  yani

$(5,4,122)$ bulunur.
Başlık: Ynt: Diyafont Denklemler Çalışma Soruları (70 Tane)
Gönderen: AtakanCİCEK - Temmuz 18, 2019, 02:56:41 ös
$58)$

Denklem katsayılarına göre simetrik olduğundan dolayı genelliği bozmadan $a\ge b\ge c\ge d$ alabiliriz.

buradan $4a^2\ge a^2+b^2+c^2+d^2=a^2b^2c^2d^2$ olur yani $4\ge b^2c^2d^2$    bu eşitsizliği $b\ge c\ge d$ den dolayı $(2,1,1)$ veya $(1,1,1)$ şeklinde çözebiliriz.

$1)$

$(b,c,d)=(2,1,1)$ için  $a^2+6=4a^2$ olur buradan $a=\sqrt{2}$ olur ve tam sayı çözümü olamaz.

$2)$

$(b,c,d)=(1,1,1)$  için $a^2+3=a^2$ yani $3=0$ çelişkisi ortaya çıkar. tam sayı çözümü olamaz.

Dolayısıyla bu denklemin pozitif tam sayılarda çözümü yoktur.
Başlık: Ynt: Diyafont Denklemler Çalışma Soruları (70 Tane)
Gönderen: AtakanCİCEK - Temmuz 18, 2019, 02:57:53 ös
$68)$

$1+2^x+2^{2x+1}=y^2$ denklemini çözerken  öncelikle kullanışlı bilgiler bulalım .

Varsayalım ki $(x_0,y_0)$ bir çözüm olsun.   O halde $(x_0,-y_0)$ da bir çözümdür.

$1+2^x+2^{2x+1}>0$ olduğundan $y^2>0$ olur. O halde genelliği kaybetmeden $y>0$ alabiliriz.

$x=0$ için çözümleri bulalım.   $1+1+2=y^2$ yani $(0,2)$ ile $(0,-2)$ çözümleri bulunur.

Buradan sonra $y>0$ ve $x>0$ için çözümleri bulalım.

$y^2-1=2^x.(2^{x+1}+1)$ olur.

Şimdi bu denklemi çözmek için hangi ifadenin hangi ifadeyi böldüğünü bulalım.

öncelikle $x$'in $1$ veya $2$ için olamayacağını görelim.

$x=1$ ise $11=y^2$  olur , sağlamaz.

$x=2$ ise $37=y^2$ olur , sağlamaz.

O halde $x\ge3$ olmalıdır.

$2\mid y-1$ ya da $2\mid y+1$ olacağı açıktır. iki durumda da $y-1$ ve $y+1$ çifttirler ve ardışık olduklarından içlerinden biri $4$ ile bölünür.

yani $x=3$ için biri $4$ ile diğeri $2$ ile bölünür.

$x=4$ için yine biri $4$ biri $2$ ile bölündüğünden ya biri $8$ diğeri $2$ ya da her ikisi de $4$ ile bölünmelidir. Fakat ardışık çift sayıların ikisi birden $4$ e bölünemez.

O halde içlerinden biri $2^3$ ile  diğeri $2$ ile bölünür.

$x\ge3$, $x=x$ için bakarsak kalanlar arası fark $2$ olması gerektiğinden dolayı içlerinden biri yalnızca $2$ ile bölünür diğeri $2^{x-1}$ ile bölünür.

$y\pm1=m.2^{x-1}$ , $y=m.2^{x-1}+k$ , $k=\pm1$ , $m\in Z^+$ olur. Bu bilgiyi alıp ilk denkleme koyalım.

$2^x.(2^{x+1}+1)=(2^{x-1}.m+k)^2-1$ yani $1+2^{x+1}=2^{x-2}.m^2+mk+k^2-1$ olur. $k=\pm1$ olduğundan dolayı $k^2=1$ yani $k^2-1=0$ olur.

$1+2^{x+1}=2^{x-2}.m+mk$

$1-mk=2^{x-2}.(m^2-8)$ olur. 

$1)$         $k=1$ için  $1-m=2^{x-2}.(m^2-8)$  bu ifadenin ise $m=2$ dışında çözümü yoktur. çünkü $m=1$ ise sağ taraf $0$ dan büyüktür.  $m>2$ için sol taraf negatifken sağ taraf pozitiftir.

$m=2$ için ise sol taraf  tek  sayı olacağından dolayı çözümü yoktur. O halde $k=-1$ olmalıdır.

$2)$         $k=-1$ için $1+m=2^{x-2}.(m^2-8)$ , $x\ge3$ olduğunu söylediğimiz için  $1+m=2^{x-2}.(m^2-8)\ge 2.(m^2-8)$ olur.

 $2m^2-m-17\le0$ eşitsizliği elde edilir.  $m=4$ için $32-4-17>0$ olur. $f(m)=2m^2-m-17$ şeklinde düşünürsek  $f'(m)=4m-1$ yani $m\ge4$ için $f(m)>0$ ve $f'(m)>0$ olduğundan daima artandır ve sıfırdan büyüktür.

Aynı zamanda $m^2-8>0$ olduğundan dolayı $m>2$ olmalıdır. $2<m<4$ elde edilir. yani $k=-1$ ,$m=3$ olmaldıır.

$1+3=2^{x-2}.1$  buradan $x=4$ elde edilir. $y=m.2^{x-1}+k$ ifadesinde $k=-1$ $m=3$ ve $x=4$ yerleştirilirse $y=23$ elde edilir.

O halde denklemin tüm çözümleri $(0,2),(0,-2),(4,23),(4,-23)$ olarak bulunur.
Başlık: Ynt: Diyafont Denklemler Çalışma Soruları (70 Tane)
Gönderen: AtakanCİCEK - Temmuz 18, 2019, 02:58:38 ös
$69)$

$5^k\equiv p^2(mod3)$ olduğundan ve $p^2\equiv 0,1 (mod3)$ olduğundan $5^k\equiv 0,1(mod3)$ olmalıdır.

 $k=2q+1, q\in Z^+$ olsun.  $5^{2q+1}\equiv 2(mod3)$ olur. dolayısıyla $k$ çift olmalıdır.

$-3^n\equiv p^2(mod5)$ olur. $p^2\equiv 0,1,4(mod5)$ olduğundan $-p^2\equiv 0,1,4(mod5)$ olur.

$3^n\equiv  0,1,4(mod5)$ denkliğini çözmemiz gereklidir.

$3^4\equiv 1(mod5)$ olduğundan denkliğinin periyodu $4$ tür.

$n=4z+1$ olursa  $3^n\equiv 3(mod5)$ olur.

$n=4z+3$ olursa $3^n\equiv 2(mod5)$ olur. ve  buradan $n$ nin de çift olması gerektiği sonucunu elde ederiz.

Şimdi ise $k=0$ olması durumunu inceleyelim. $1-3^n=p^2$ olur. $1-3^n\le 0$ olduğundan dolayı ve  $p^2\ge 4$ olduğundan çözüm yoktur.

$n=0$ olması durumunda $5^k-1=p^2$ ise Sol taraf çift olduğundan dolayı $p=2$ dışında çözüm olamaz. $5^k-1=4$ olur ve buradan $k=1$ bulunur.  $(k,n,p)=(1,0,2)$ çözümü elde edilir.

şimdi ise $x,y\in Z^+$ için $k=2x$ ve $n=2y$ dönüşümleri yapalım.

$5^{2x}-3^{2y}=p^2$   $(5^x-3^y).(5^x+3^y)=p^2$ olur. $5^x+3^y>0$ olduğundan dolayı $5^x-3^y>0$ da olmalıdır.

Buradan dolayı


\begin{equation}
\begin{cases}
5^x+3^y=p^2,
\\
5^x-3^y=1
\end{cases}
\end{equation}

\begin{equation}
\begin{cases}
5^x+3^y=p
\\
5^x-3^y=p
\end{cases}
\end{equation}

\begin{equation}
\begin{cases}
5^x+3^y=1,
\\
5^x-3^y=p^2
\end{cases}
\end{equation}

Sistemleri elde edilir.

$2$ ve $3$ nolu sistemlerin mümkün olmadığını gösterelim.

$2$ için $5^x-3^y=5^x+3^y$ olur. $-3^y=3^y$ olur ve buradan $3^y=0$ dan dolayı çelişki oluşur.

$3$ için $5^x-3^y>5^x+3^y$ yani $-3^y>3^y$ olduğu sonucu çıkar ve bu da imkansızdır.

Yani çözümü olabilecek tek sistem $1$ dir.

$5^x=\frac{p^2+1}{2}$

$3^y=\frac{p^2-1}{2}$ olur.

$p=2$ için  $5^x=\frac{5}{2}$ olduğundan çözüm yoktur.

$p=3$ için $3^y=4$  olduğundan çözüm yoktur.

$p\ge 5$ için $p=6k\pm 1$ olduğundan $p^2\equiv1(mod12)$ olmalıdır. $p^2=12s+1, s\in Z^+$ yazılabilir.

$3^y=\frac{p^2-1}{2}$  olduğundan  $3^y=6s$ yani $3^y\equiv0(mod2)$ denkliği ortaya çıkar. Bu denkliğin çözümü olmadığından $1$ sisteminin de çözümü olamaz.

Denklemin tek çözümü $(k,n,p)=(1,0,2)$  olması gerektiği bulunur.
Başlık: Ynt: Diyafont Denklemler Çalışma Soruları (70 Tane)
Gönderen: AtakanCİCEK - Temmuz 18, 2019, 02:59:35 ös
$71)$

Bu denklemde $n$ ifadesi için eşitsizlik kurmaya çalışalım. Öncelikle küçük değerler için biraz hesaplar yapalım.

$n=1$ için $k=1$ olur.
$n=2$ için $k=3$ olur.
$n=3$ için uygun $k$ değeri olmaz.


Eşitliğin sağ tarafı uzun olduğu için kısaltma amaçlı $EST$ şeklinde yazacağım.$v_2(k!)=v_2(EST)=0+1+2+...+(n-1)=\frac{(n-1).n}{2}$ olur.
$EST=2^{\frac{(n-1).n}{2}}.(2-1).(2^2-1)...(2^n-1)$ şeklinde yazılabilir. $3\mid 2^m-1$ olacak şekilde $m\in Z^+$ olması için $n\ge2$ olmalıdır.
Şimdi bu tarz sayılar için $m=2m'$ dönüşümü yapılabilir.
Kuvvet Kaydırma Teoremi ile  $v_3(4^{m'}-1)=v_3(4^{m'}-1^{m'})=v_3(4-1)+v_3(m')=1+v_3(m')$ olur.
Daha sonra \begin{equation} v_3(k!)=\sum_{n=1}^{\infty} ⌊\frac{n}{2.3^i} ⌋<\sum_{n=1}^{\infty} \frac{n}{2.3^i}=\frac{n}{2}.\sum_{n=1}^{\infty} \frac{1}{3^i} \end{equation}

Kuvvet Kaydırma Teoremi olarak söylediğim aslında şudur
https://artofproblemsolving.com/community/c2032h1011696_lifting_the_exponent

Bu ifadede ise bilinen seri toplamı olduğu için kolaylıkla $v_3(k!)<\frac{3n}{4}$ olur.
De polignac formülünden dolayı \begin{equation} v_p(k!)=\sum_{i=1}^{\infty} ⌊\frac{k}{p^i} ⌋ \end{equation}

olduğunu biliyoruz.  Bu formülü iki eşitsizlik arasına sıkıştıralım.

$⌊\frac{k}{p}⌋<v_p(k!)<\frac{k}{p}+\frac{k}{p^2}+\frac{k}{p^3}+...=\frac{k}{p}.\frac{1}{1-\frac{1}{p}}=\frac{k}{p-1}$
Şimdi \begin{equation} v_2(k!)=\frac{n.(n-1)}{2}<k \end{equation}

olur.  Aynı zamanda $\frac{k}{3}-1<v_3(k!)<\frac{3n}{4}$ olduğundan dolayı \begin{equation} k<\frac{9n}{4}+3 \end{equation}
O halde
 \begin{equation} \frac{n.(n-1)}{2}<\frac{9n}{4}+3 \end{equation}
yani $2n^2-11n+3<0$ elde edilir. $f(n)=2n^2-11n+3$ olsun. $n\ge 7$ için $f(n)=24>0$ ve $f'(n)= 4n-11\ge 17>0$ olduğundan dolayı daima pozitiftir. O halde $n\le 6$  olmalıdır.

Zaten $n\le 3$ için denemiştik. O halde

$n=4$ için $k!=7!.4$ olacağından dolayı çözümü yoktur.

$n=5$ için  $EST$ $2^5-1=31$ asal çarpanına sahip olacağından dolayı $k!\ge 31!$ olur. buradan ise eşitliğin sol tarafı  $23$ ile bölünürken sağ taraf bölünemez.

$n=6$ için de eşitliğin sol tarafı $23$ ile bölünürken $EST$ $23$ ile bölünmez.

O halde denklemin çözümleri $\{(3,2),(1,1)\}$ olarak bulunur.
Başlık: Ynt: Diyafont Denklemler Çalışma Soruları (70 Tane)
Gönderen: alpercay - Temmuz 19, 2019, 05:47:35 ös
9.sorunun çözümünde $x^3+y^3+z^3-3xyz=(x+y+z)(x^2+y^2+z^2-xy-xz-yz)$ özdeşliği de kullanılabilir. Bunun için denklemi $27$  ile genişletip $1$ çıkartmak gerekiyor. Bu durumda $$(3x)^3+(-3y^3)+(-1)^3-3(3x)(-3y)(-1)=61.27-1$$  $$(3x-3y-1)(9x^2+9y^2+1+9xy+3x-3y)=2.823$$ $$3x-3y-1=2$$   ve  $x=y+1$  değeri verilen denklemde yerine konursa $x=6$  ve  $y=5$  bulunur. 823 asal olduğundan ve diğer çarpanı modülo 9 da değerlendirince çözüm gelmiyor sanırım.
Başlık: Ynt: Diyafont Denklemler Çalışma Soruları (70 Tane)
Gönderen: AtakanCİCEK - Temmuz 19, 2019, 09:46:46 ös
9.sorunun çözümünde $x^3+y^3+z^3-3xyz=(x+y+z)(x^2+y^2+z^2-xy-xz-yz)$ özdeşliği de kullanılabilir. Bunun için denklemi $27$  ile genişletip $1$ çıkartmak gerekiyor. Bu durumda $$(3x)^3+(-3y^3)+(-1)^3-3(3x)(-3y)(-1)=61.27-1$$  $$(3x-3y-1)(9x^2+9y^2+1+9xy+3x-3y)=2.823$$ $$3x-3y-1=2$$   ve  $x=y+1$  değeri verilen denklemde yerine konursa $x=6$  ve  $y=5$  bulunur. 823 asal olduğundan ve diğer çarpanı modülo 9 da değerlendirince çözüm gelmiyor sanırım.

Bu çözüm de güzel olmuş hocam aklınıza sağlık
Başlık: Ynt: Diyafont Denklemler Çalışma Soruları (70 Tane)
Gönderen: alpercay - Temmuz 20, 2019, 12:03:09 öö
Lokman Hocamdan esinlendim; ona da teşekkürler.
Başlık: Ynt: Diyafont Denklemler Çalışma Soruları (70 Tane)
Gönderen: AtakanCİCEK - Temmuz 23, 2019, 07:57:59 ös
$65)$

$2^n+1=n^2m$ denklemini $\frac{2^n+1}{n^2}\in Z^+$  ifadesi şeklinde çözebiliriz.

$n^2\mid 2^n+1$ olduğundan dolayı $v_p(n^2)\le v_p(2^n+1)$ $(1)$

 olmalıdır.

Ayrıca $n$ çift olursa  $4\mid 2^n+1$ ve $2^n+1\equiv 1(mod2)$ olması gerektiği için çelişki oluşur.

$n$ tek iken Kuvvet Kaldırma Teoremi kullanırsak

$$v_p(x^n+y^n)=v_p(x+y)+v_p(n)$$

Yani $v_p(2^n+1)=v_p(3)+v_p(x)$ $(2)$

$p\ge 5$ için $(1)$  ile $(2)$ gereğince   $v_p(2^n+1)=v_p(n)\ge v_p(n^2)$ elde edilir.

Ancak $v_p(n^2)>v_p(n)$ olması gerektiği için $p\ge 5$ olamaz.

$2^n+1\equiv1(mod2)$ olduğundan $p=2$ de olamaz.  O halde $p=3$ olmalıdır.

Yani $2^n+1$ ifadesinin tek asal çarpanı $3$ olduğuna göre $2^n+1=3^k$ , $k\in Z^+$ olmalıdır.

$v_3(3^k)=k=v_3(2+1)+v_3(n)$ olur.  $v_3(n)=k-1$ yani $n=x.3^{k-1}$ , $x\in Z^+$ olacak şeklide dönüşüm yapalım.

$$3^k=2^{x.3^{k-1}}+1\ge 2^{3^{k-1}}+1$$

olur fakat  $k>2$ için $3^k<2^{3^{k-1}}+1$ olduğu için $k\in\{1,2\}$ olabileceği görülür.

$k=1$ için $n=1$  olur $n>1$ olmadığı için olamaz.

$n=1$ ise $m=3$ , $n=3$ ise $m=1$ olur.

O halde denklemin tek çözümü $(3,1)$ bulunur.
Başlık: Ynt: Diyafont Denklemler Çalışma Soruları (70 Tane)
Gönderen: alpercay - Temmuz 24, 2019, 05:07:34 ös
46. soruda $x^4+y^4=z^2$ denkleminin çözümünün olmadığını gösteriniz deniyor fakat $(-3,0,+-9),(-2,0,+-4),(-1,0,1)$ gibi çözümler var. Kontrol eder misin?
Başlık: Ynt: Diyafont Denklemler Çalışma Soruları (70 Tane)
Gönderen: AtakanCİCEK - Temmuz 25, 2019, 02:31:09 ös
46. soruda $x^4+y^4=z^2$ denkleminin çözümünün olmadığını gösteriniz deniyor fakat $(-3,0,+-9),(-2,0,+-4),(-1,0,1)$ gibi çözümler var. Kontrol eder misin?
Pozitif tam sayılarda  demeyi unutmuşum hocam teşekkürler
Başlık: Ynt: Diyafont Denklemler Çalışma Soruları (70 Tane)
Gönderen: AtakanCİCEK - Ağustos 01, 2019, 11:49:36 ös
$46)$
 
Varsayalım ki bu denklemin $\{(x_0,y_0,z_0)\}$ için pozitif tam sayı bir çözümü olsun. $(x_0,y_0)=d$ olsun.  O halde $d^2\mid z_0$ olacağından dolayı  $\{(\frac{x_0}{d},\frac{y_0}{d},\frac{z_0}{d^2})\}$ de bir çözüm olmalıdır.  Bu çözümdeki değişkenleri sırasıyla $a$, $b$ , $c$ olarak alırsak
                                     
                                                                                   $S=\{c\in Z^+ : a^4+b^4=c^2$ ve $(a,b)=1$ olacak şekilde bir $a,b\in Z^+$ vardır $\}$

şeklinde soruyu düşünebiliriz.

 Yukarıdaki $S$  varsayımından dolayı $S\not = \oslash$ olması gerektiğinden dolayı iyi sıralanma ilkesi gereğince  $S$ nin en küçük elemanı olmalıdır.  $(a^2,b^2,c)=1$  $(a^2)^2+(b^2)^2=c^2$ şeklinde düşünürsek   $(s,t)=1$ ve $s\not \equiv t(mod2)$  için

                                                                                              $$a^2=2st \text{    ,     } b^2=s^2-t^2 \text{   ,    }  c^2=s^2+t^2$$                                                                                               

olduğunu gösterelim.

$ispat:$ 

$Lemma:$ İspata girerken öncelikle $a$ çift olduğu için $b$ ve $c$ nin tek olduğunu gösterelim.

$a^2+b^2=c^2$  denkleminde $a $ ile $b$ ifadelerinin  ikisinin de tek olması $c^2 \equiv 2(mod4)$ olmasına neden olur.
her ikisinin de çift olması  durumu ise $(a,b,c)=1$ kabulünden dolayı çelişir


burada bizim çift olan terimimiz $a$ olduğu için  $c+b$  ile $c-b$ de çift olmalıdır.

$a=2r$, $b+c=2u$ , $b-c=2v$ olacak şekilde $ r,u,v\ge 1$ tam sayıları vardır.
$a^2=b^2-c^2$ olduğundan dolayı $4r^2=2u.2v$  yani $r^2=uv$ elde edilir.
 Şimdi $(u,v)=1$ olduğunu hızlıca gösterelim.  $d=(u,v)$  olsun.  $c-b=2u$ ve $c+b=2v$  olduğundan dolayı $c=u+v$ ve $b=u-v$ olmalıdır.  $d\mid c $  ve $d\mid b$ olmalıdır. fakat $(b,c)=1$ olduğu için $(u,v)=1$ olmalıdır.
$(u,v)=1$ olduğundan dolayı $u$ ile $v$ ayrı ayrı birer tam kare olmalıdır.
$u=s^2$ , $v=t^2$ , $s,t\ge 1$ tam sayıları bulunur. İfadeleri düzenlersek
                                                                                              $$a^2=2st \text{    ,     } b^2=s^2-t^2 \text{   ,    }  c^2=s^2+t^2$$
olduğu ispatlanmış olur.


 $2.$ eşitlikten $b^2+t^2=s^2$ bulunur. $(s,t)=1$ olduğundan dolayı $(b,s,t)=1$ olmalıdır.   Pisagor üçlülerini  $Lemma$ mızdan dolayı $b$ tek olduğu için $t$ çift $s$  tek olmalıdır.

$a^2=2st$ olduğundan dolayı $(\frac{a}{2})^2=s.\frac{t}{2}$ olur. $s=u^2$ ve $t=2v^2$ , $u,v\ge 1$ tam sayıları vardır.
$b^2+(2v^2)^2=(u^2)^2$ bulunur. $(s,t)=1$ olduğundan $(u,v)=1$ olacağı da açıktır. $s$ tek olduğundan $u$ tektir.

İspatını verdiğimiz ifadeyi tekrar kullanacak olursak
                                                                                              $$2v^2=2ef \text{    ,     } b^2=e^2-f^2 \text{   ,    }  u^2=e^2+f^2$$
ve $(e,f)=1$ olacak şekilde $e,f\ge 1$ tam sayıları vardır. $v^2=ef$olduğu için $e=q^2$ , $f=r^2$ elde edilir.
O halde $u^2=q^4+r^4$elde edilir. $(e,f)=1$ olduğu için $(q,r)=1$ de olmalıdır.  Fakat $u\le s\le a^2<c$ olduğundan dolayı yani $c>u$ olduğundan dolayı iyi sıralanma ilkesi ile yani $c$ nin en küçük olmasıyla çelişir.  O halde denklemin pozitif tam sayılarda çözümü yoktur.


 Çözüm tam sayılarda olsa idi  $x.y.z=0$ ifadesi sağlanmalıydı çünkü $x^4+y^4=z^2$ için çözümümüzü  $Z-\{0\}$ kümesine genelleyebilirdik o halde $x,y$ den biri çift olmalıdır ve  o  sayı da $0$ 'a eşit olmalıdır.  Bu nedenle $x.y=0$ elde edilebilirdi.
Başlık: Ynt: Diyafont Denklemler Çalışma Soruları (70 Tane)
Gönderen: AtakanCİCEK - Ağustos 02, 2019, 02:45:23 ös
$49)$

Öncelikle $p$ değerini bulalım.  $p=5a+b$ olsun. $p^2$ yi  bulalım.
$$ (5a+b)^2=25a^2+10ab+b^2=5.(5a^2+2ab)+b^2$$ 

Bu  ifadede $5a^2+2ab=c$ dönüşümü yapalım.  $p^2=5c+b^2$ olur.
1)   $b=0$ olarak alırsak $5$ in katı tek asal sayı $5$ olacağı için $p=5$ olmalıdır. $4p^2+1=101$ asal sayı , $6p^2+1=151$ asal sayı olduğu açıktır.
$b\not \equiv 0(mod5)$ için $b^2\equiv 1,4(mod5)$ olduğunu kullanalım.

2) $b^2\equiv 1(mod5)$ ise $4p^2+1\equiv 0(mod5)$ ve asal olması gerektiğinden dolayı $4p^2+1=5$ , $p=1$ olmalıdır fakat $p$ nin asal olmasıyla çelişir.

3) $b^2\equiv 4(mod5)$ ise $6p^2+1 \equiv 0(mod5)$ ve asal olması gerektiğinden dolayı  $6p^2+1=5$  olması gerekir fakat $p$ nin asal olmasıyla çelişir.

O halde $p=5$ olmalıdır.  Denklemde yerine koyarsak $2x^3-y^3=5$ denklemi elde edilir.

$2x^3=y^3+5$ olur .  Varsayalım ki $y$ çift olsun.  O halde
$2x^3\equiv 13(mod2)\equiv 1(mod2)$ olacağından dolayı mümkün değildir. O halde $y=2k+1$ ,  $k\in Z$ olmalıdır.

$$2x^3=8k^3+12k^2+6k+1+5=8k^3+12k^2+6k+6$$ yani

$$4k^3+6k^2+3k+3=x^3$$ denklemini çözsek yeterlidir.  $k$ nın $3$ ile bölümünden kalanları inceleyelim.
Öncelikle $x^3\equiv \{0,1,8\} (mod9)$ olduğunu belirtmekte fayda var.
1) $k\equiv 0(mod3)$ ise $x^3\equiv 3(mod9)$ olacağından dolayı çözümü yoktur.
2) $k\equiv 1(mod3)$ ise $x^3\equiv 7(mod9)$ olacağından dolayı çözümü yoktur.
3) $k\equiv 2(mod3)$ ise $x^3\equiv 2(mod9)$ olacağından dolayı çözümü yoktur.

O halde diyafont denklemimizin çözümü yoktur.
Başlık: Ynt: Diyafont Denklemler Çalışma Soruları (70 Tane)
Gönderen: AtakanCİCEK - Ağustos 02, 2019, 04:17:37 ös
$45)$

Euler-fi fonksiyonunun özelliklerinden dolayı gerek ve yeter şartın $n=2k+1$ , $k\in Z^+$ olacak şekilde sayıların seçilmesi yeterli olacaktır. Şimdi bunu gösterelim. Öncelikle Euler-Fi fonksiyonunun değerini bulduran formülü ispatlayalım.
$n=p_1^{e_1}.p_2^{e_2}.p_3^{e_3}...$ olsun. $$\phi(n)={\overset{t}{\underset{i=1}{{\displaystyle\prod}}}p_i^{e_i-1}(p_i-1)}$$ olduğunu göstermenin bir yolunu arayalım.

Bunu bulurken öncelikle bir lemma elde edelim.

$Lemma1$   $$\phi(p^m)=p^m-p^{m-1}$$ eşitliği $p$ bir asal sayı ve $m$ bir pozitif tam sayı iken geçerlidir.

$ispat:$ $1\le i\le p^m$ ve $(i,p^m)=d$ olsun. Eğer $d>0$ ise $d$ nin bir $q$ şeklinde asal böleni olmalıdır. $q\mid p^m$ olacağından dolayı $q=p$ olacağı açıktır. yani $p\mid i$ olacağını göstermiş oluruz.
$$ 1.p,2.p,3.p,...,p^{m-1}.p$$ şeklinde $p^m$ den küçük $p^m$ ile aralarında asal olmayan sayı vardır. $1$ ile $p^m$ arasında $p^m$ tane sayı olacağından dolayı    $$\phi(p^m)=p^m-p^{m-1}$$ ifademiz ispatlanmış olur.

Şimdi asıl ispatlamamız gereken ifadeye geri dönüş yapabiliriz.

İstediğimiz formül  $t=1$ için $Lemma1$ den dolayı apaçıktır.  $t>1$ için tümevarıma gidelim.
$$n_1={\overset{t-1}{\underset{i=1}{{\displaystyle\prod}}}p_i^{e_i}}$$
olsun.  O halde $n=n_1 .p_t^{e_t}$ ve $(n_1,p_t^{e_t})=1$  olduğundan dolayı  $$\phi(n)=\phi(n_1).\phi(p_t^{e_t})$$ olmalıdır.
Tümevarım hipotezinden dolayı $$\phi(n_1)={\overset{t-1}{\underset{i=1}{{\displaystyle\prod}}}p_i^{e_i-1}(p_i-1)}$$
ve $$\phi(p^t)=p^t-p^{t-1}$$ ifadelerini çarparsak
$$\phi(n)={\overset{t}{\underset{i=1}{{\displaystyle\prod}}}p_i^{e_i-1}(p_i-1)}$$ elde edilmiş olur. 

Eğer $2^a\mid \mid n$ $n=2^a.n_2$ çift olursa $\phi(2n)$ içindeki   $p_i^{e_i}$ ifadelerinden biri $2^{a+1}-2^{a}$ olacağından  $\phi(n)$ nin içindeki ise $2^{a}-2^{a-1}$ olacağı için $$\dfrac{\phi(2n)}{\phi(n)}=\dfrac{2^a.(2-1)}{2^{a-1}.(2-1)}=2$$ olacağından dolayı mümkün değildir.  $n$ tek olmalıdır.
Başlık: Ynt: Diyafont Denklemler Çalışma Soruları (70 Tane)
Gönderen: AtakanCİCEK - Ağustos 02, 2019, 09:14:16 ös
$73)$

Bu soruyu $a>b$ olup olmamasına göre iki parçada inceleyelim.


$1)$ $1\le a\le b$ için çözüme bakalım.



$a\le b$ olduğunu kullanarak eşitsizlik kurmayı hedefleyelim.
 
$(a^b)^b=b^a$ şeklinde yazabiliriz. $a\le b$ den dolayı $a^b\le b $ olmazsa çözümün olamayacağı açıktır. 
$a^b-b$ ifadesinin türevini alalım. $\dfrac {d}{db} (a^b-b)=1.a^b.lna-1$  $b>a>e$  için $lna>1$ ve $a^b>1$ olacağı için  türev $0$'dan büyüktür Mesela $b=3$ alacak olursak  $a^3-3>0$ olduğunu gösterirsek $a^b-b$ nin daima pozitif olduğunu göstermiş oluruz.
$a>e>2$ $a^3>8$  yani $a^3-3>0$ elde edilir. Yani $a=1$ ya da $a=2$ olmalıdır.

$a=1$ olsun. O halde $b=1$ olması gerektiği açıktır.

$a=2$ olsun. $2^{b^2}=b^2$ $b^2=t$ dönüşümü yapalım.  $2^t=t$,  $2^t-t=0$ Şimdi bu ifade için türeve bir kez daha bakalım.
$2^t.ln2-1$ olur. Bu ifadeyi ise $ln2^{2^t}-1$ şeklinde yazdıktan sonra $t>1$ için türevin $0$ dan büyük olacağı açıktır. Aynı zamanda $2^2-2>0$ da sağlandığından dolayı daima $t>1$ için daima pozitiftir.  $t=1$ in de sağlamadığı açıktır.



$2)$ şimdi de $b>a\ge 1$ olsun.



$a^{b^2}=b^a=(a^b)^b$ den dolayı $a>b^2$ gelir yani $\dfrac{a}{b^2}>1$ elde edilir. Acaba $a=k.b^2$ eşitliğindeki $k$ sayısı daima bir pozitif tam sayı  olabilir mi diye düşünelim.

$(ab^{-2})^{b^2}=b^{a-2b^2}$ şeklinde yazarsak $ab^{-2}>1$ olduğu için $a-2b^2>0$ olacağı açıktır. $a>2b^2$ olur.

$\dfrac{a}{b^2}=\dfrac{k}{n}$ ,$k,n\in Z^+$ ve $(k,n)=1$  olsun.  Buradan $n.a.b^{-2}=k$  elde edilir.

Aşağıdaki adımları takip edelim.

$$k^{b^2}=n^{b^2}.a^{b^2}.(b^{-2})^{b^2}=n^{b^2}.b^{a-2b^2}$$  bu ifadeden dolayı $n\mid k$ olması gereklidir. $(k,n)=1$ olduğundan dolayı $n=1$ alınmalıdır. Artık  $a=k.b^2$  $a>2b^2$ yani $k\ge3,k\in Z^+$ olduğunu biliyoruz.

Bunu başlangıçtaki denklemde yerine koyacak olursak $k^{b^2}=(b^{k-2})^{b^2}$ olmalıdır. buradan $k=b^{k-2}$ buluruz.


 $k\ge 5$ için  $b^{k-2}-k\ge 2^{k-2}-k$ olur.  $\dfrac{d}{dk}( 2^{k-2}-k)=(k-2).2^{k-2}.ln2-1$ yani $2^{k-2}.ln2^{k-2}-1>0$ $k=5$ için ise $2^3-5=3>0$ olduğundan $k<5$ olmalıdır.


$k=3$ ise $3b^2=b^3$ yani $b=3$ elde edilir. $a=kb^2=27$ olur. $(27,3)$ çözümü gelir.

$k=4$ ise $4b^2=b^4$ yani $b=2$ elde edilir. $a=16$ olur.  $(16,2)$ çözümü gelir.


O halde denklemin çözüm kümesi $\{(1,1),(27,3),(16,2) \}$ olmalıdır.   
Başlık: Ynt: Diyafont Denklemler Çalışma Soruları (70 Tane)
Gönderen: AtakanCİCEK - Ağustos 16, 2019, 01:23:01 ös
$13)$ $(n^2)^2<n^4+n^3+1$ olduğundan bir $m\in Z^+$ sayısı için $(n^2+m)^2=n^4+n^3+1$ yazılabilir.

$n^4+m^2+2mn^2=n^4+n^3+1$
$m^2+2mn^2=n^3+1$
$m^2-1=(n-2m).n^2$ 
$m>1$ için  $n^2>0$  olduğundan dolayı $n>2m$ olmalıdır. $n^2\mid m^2-1$ olduğunu görürsek $n^2\le m^2-1$ görülebilir.

$n>2m$ ile $m^2-1\ge n^2$ eşitsizliklerini ortak çözmeliyiz.  $n^2>4m^2$  olacağından dolayı $m^2-1\ge n^2 >4m^2$  yani $3m^2<-1$ gelir.  ancak bu bir çelişkidir. O halde $m=1$ olmalıdır.
$m=1$ için $0=(n-2).n^2$  olur pozitif tam sayılarda yalnızca $n=2$ için tam kare olur. Değeri ise $2^4+2^3+1=25$  tir.
Başlık: Ynt: Diyafont Denklemler Çalışma Soruları (70 Tane)
Gönderen: AtakanCİCEK - Ağustos 17, 2019, 05:47:29 ös
$14)$   

$(0,0,0,0)$ bir çözümdür.

Eşitliğin sağ tarafı $3$ ile bölünmektedir. $n$ sayısı da $3$ ile bölünmelidir.  $n=3k$, $k\in Z$ yazarsak

$5.9k^2=36a^2+18b^2+c^2$ eşitliğinden

$15k^2=12a^2+6b^2+2c^2$  bulunur. Benzer şekilde $c=3e$ , $e\in Z$ yazarsak

$5k^2-4a^2=2b^2+6e^2$ bulunur.  Bu denklemin $k$ değeri en küçük çözümü $(k,a,b,e)$ olsun.

$x^2\equiv 0,1,4,9(mod16)$  olduğunu biliyoruz.

$k^2\equiv \{0,4\} (mod16)$  olduğu açıktır.

$4a^2\equiv \{0,4\}(mod16)$

$2b^2 \equiv {0,2,8}(mod16)$

$6e^2 \equiv \{0,6,8\}(mod16)$  olduğundan dolayı

$2b^2+6e^2\equiv \{ 0,2,6,8,10,14 \}(mod16)$ ve  $5k^2-4a^2 \equiv \{ 0,4,12 \} (mod16)$ olduğundan her iki ifade de $0(mod16)$ olmalıdır. $2b^2+6e^2\equiv 0(mod16)$ olduğundan $b$ ve $e$ çifttir. $a$ çift olmamalıdır. çünkü $(k,a,b,e)$ nin yarısı da çözüm olur. minimallik ile çelişir.

$b=2b_1$ , $e=2e_1$ , $k=2k_1$ olsun.

$$2b_1^2+6e_1^2=5k_1^2-a^2$$

$a$ nın tek sayı olduğu kullanılarak $5k_1^2-a^2\equiv \{ 4,12\}(mod16)$ olmalıdır. Fakat $2b_1^2+6e_1^2\equiv \{ 0,2,6,8,10,14 \}(mod16)$ olduğundan çelişkidir.  O halde denklemin tek çözümü $(0,0,0,0)$ olmalıdır.
Başlık: Ynt: Diyafont Denklemler Çalışma Soruları (70 Tane)
Gönderen: AtakanCİCEK - Ağustos 17, 2019, 06:04:26 ös
$15)$ 
$(0,0,0)$ bir çözümdür.  Başka çözümün olmadığını göstermek için bu tarz sorularda genellikle aralarında asal çözümü (obebi 1 olan çözüm) seçmek yararlıdır. çünkü $(0,0,0)$ dışındaki diğer çözümler varsa bunlar asal çözümlerin katıdır.

Bu yüzden $14)$ numaralı soruda yaptığımız gibi $obeb(x,y,z)=1$ alalım.

$z$ nin çift olduğunu görmek çok zor değildir. $z=2z_1$ dönüşümü yapabiliriz.

$5x^3+10y^3+8xyz_1=1999.4.z_1^3$  olur. Benzer şekilde $x$ te çifttir. $x=2x_1$ dönüşümü yapılırsa

$2x^3+5y^3+8x_1yz_1=1999.2.z_1^3$  buradan dolayı $y$ de çift olmalıdır fakat $y$ çift olursa  $obeb(x,y,z)=2$ olacağından $(x,y,z)$ çözümünün asal çözüm olmasıyla çelişir.

Denklemin tek çözümü $(0,0,0)$ olmalıdır.
Başlık: Ynt: Diyafont Denklemler Çalışma Soruları (70 Tane)
Gönderen: AtakanCİCEK - Ağustos 17, 2019, 06:14:56 ös
$16)$
$(m-n)^2.(m+n-1)=4mn$ eşitliğini aşağıdaki adımlarla düzenleyelim.
$2mn-m^2+m^3-n^2+n^3-mn^2-m^2n=4mn$
$m^3+n^3-mn^2-m^n=2mn+n^2+m^2$
$m.(m^2-n^2)-n.(m^2-n^2)=(m+n)^2$ 
$(m-n)^2.(m+n)=(m+n)^2$   $m+n>0$ olduğu da göz önüne alınarak sadeleştirilirse
$(m-n)^2=m+n$ olur. $m-n=x$ $m+n=x^2$ denilirse
$0<x^2<100$ olur. aynı zamanda

sistemin çözümü $\{( \dfrac{a.(a+1)}{2},\dfrac{a.(a-1)}{2})\}$ olduğu da biliniyor. Paydanın $0$ olamayacağı da göz önüne alınırsa ($m+n\not = 1$) $a\in \{\pm 2,\pm 3 , \pm 4 ,...,\pm9\}$ değerleri için $8.2=16$  çözüm bulunur.
Başlık: Ynt: Diyafont Denklemler Çalışma Soruları (70 Tane)
Gönderen: AtakanCİCEK - Ağustos 17, 2019, 06:28:44 ös
$19)$  Çözüm için metonstere teşekkürler.
Verilen ifadelerde $x$ yerine $-x$ alırsak ifadenin tamsayı olması veya tamküp olması değişmeyeceğinden dolayı $x$ ve $y$'yi pozitif alabiliriz. Öncelikle $x=y$ için bakalım,
$$\dfrac{x^2+x^2+6}{x^2}=2+\dfrac{6}{x^2}$$
olur. Bunun tamsayı olması için $x=1$ olmalıdır. $x=1$ için de tamküp olduğu görülür. Şimdi geriye kalan çözümlerden $x>y$ olanlara bakalım.(simetrik bir ifade olduğu için $y>x$ durumlarına bakmaya gerek yok.) Bu çözümler arasında $x$ değerinin en küçük olduğu çözümü ele alalım.Bu $(x,y)$ için
$\dfrac{x^2+y^2+6}{xy}=k$ olsun.

$$x^2+y^2+6=kxy \Rightarrow x^2-kxy+(y^2+6)=0$$
olur. Bu denklemi $x$'e bağlı ikinci dereceden bir denklem olarak düşünelim ve diğer çözüm $a$ olsun.Vieta teoreminden,
$$a+x=ky,\space ax=y^2+6$$
olduğu görülür.Bu iki ifadeden $a$ sayısının pozitif bir tamsayı olduğu görülebilir. Dolayısıyla $(a,y)$ ikilisi de bir çözümdür. Kabulden dolayı $a\ge x$ olmalı.
$$x^2+6 \geq y^2+6=ax\geq x^2 \Rightarrow x^2\geq y^2\geq x^2-6$$

eşitsizliğini elde etmiş oluruz. Eğer $x\ge4$ ise  $x^2-6\geq (x-1)^2$ olacağından

$$x^2\geq y^2\geq x^2-6\geq (x-1)^2$$ olur ve ardışık tamkareler arasında tam kare olamayacağı için buradan çözüm gelmez. $4>x$ için de olası sadece $(x,y)=(3,2),(3,1),(2,1)$ ikilileri vardır fakat bunların hiçbiri ifadenin tamsayı olmasını sağlamaz. Dolayısıyla
yalnızca $(1,1)$ ikilisi için tam sayıdır ve bu durumda da $\dfrac{x^2+y^2+6}{xy}$ ifadesi tam küp olur.
Başlık: Ynt: Diyafont Denklemler Çalışma Soruları (70 Tane)
Gönderen: AtakanCİCEK - Ağustos 17, 2019, 07:59:01 ös
$30)$

$x$ ve $y$ nin birbirine göre simetrik olduğu açıktır. $x^2+axy+y^2=z^2$ ise $x^2+axy=z^2-y^2$ ve  $ x.(x+ay)=(z-y).(z+y)$

olur. $y=z$ iken $x=0$ veya $x=-ay$ olur.  Bunun dışındaki durumlarda $\dfrac{x}{z-y}=\dfrac{z+y}{x+ay}=\dfrac{m}{n}$ oranları elde edilir. Buradan ise :
\begin{equation*}
\begin{cases}
mx+ny-nz=0,
\\
nx+(n-am)-mz=0,
\\
z=k.(amn-m^2-n^2)
\end{cases}
\end{equation*}

Bu sistemin çözümünü yaparsak

\begin{equation*}
\begin{cases}
k.(an^2-2mn),
\\
k.(m^2-n^2),
\\
k.(amn-m^2-n^2)
\end{cases}
\end{equation*}

olarak bulunur.
Başlık: Ynt: Diyafont Denklemler Çalışma Soruları (70 Tane)
Gönderen: AtakanCİCEK - Ağustos 17, 2019, 10:23:01 ös
$38)$ $m^2=n^2+3^n$ eşitliğini sağlayan bir pozitif tam sayı olsun. $(m-n).(m+n)=3^n$ olduğu için $m-n=3^k$ ve $m+n=3^{n-k}$

olmasını sağlayan bir $k\ge 0 $ sayısı vardır. $m-n<m+n$ olduğundan dolayı $3^k<3^{n-k}$  $k<n-k$  $2k<n$  yani $n-2k\ge 1$ olur.

$n-2k=1$ ise $2n=(m+n)-(m-n)=3^{n-k}-3^k=3^k.(3^{n-2k}-1)=2.3^k$ olur. $3^k=2k+1$ elde edilir. Denklemi türevle çözeceğim

Fakat tümevarımla da çözülebilir.  $f(k)=3^k-2k-1$  olsun.

$f'(k)=1.3^k.ln3-2$ $k\ge2$ için $f'(k)>0$ olacağı açıktır. $f(2)=9-4-1>0$ ve $f'(2)>0$ olduğundan daima pozitiftir. $k=0$ yada $k=1$
olmak zorundadır. 
$k=0$ için $m-n=1$ $m+n=3^n$  olur. $2n+1=3^n$ $n>0$ olduğundan dolayı $n=1$ olur.
$k=1$ için $m-n=3$ $m+n=3^{n-1}$ olur. $2n+3=3^{n-1}$   $n=3$ için ifadenin kendi $0$ dan büyüktür. $1.3^{n-1}.ln3-2$ elde  edilir. $n>3$ sayıları konulduğunda pozitif çıktığından dolayı İfade daima pozitiftir, $0$ değerini almaz $1,2,3$ değerleri denenirse $n=3$ olacağı görülür.

$n=1$ için $n^2+3^n=4$
$n=3$ için $n^2+3^n=36$ olur.
Gelelim $n-2k>1$ durumuna $n-2k\ge2$ için  $k\le n-k-2$ olur. $m-n<m+n$ eşitsizliği kullanılırsa $3^k\le 3^{n-k-2}$ olur.

$2n=3^{n-k}-3^k\ge 3^{n-k}-3^{n-k-2}=3^{n-k-2}.8$  elde edilir.

Daha önceden ispatladığımız $3^x >2x+1$ , $m\ge 2$ eşitsizliğini hatırlayalım. $m=0$ ile $m=1$ iken de eşit olduğunu hatırlayarak

$8.3^{n-k-2}\ge 8.(1+2.(n-k-2))=16n-16k-24$ bulunur. $2n\ge 16n-16k-24$ yani $8k+12\ge 7n$ elde edilir. $n\ge 2k+2$ olduğundan

dolayı $7n\ge 14k+14$ olur. fakat bu sefer $8k+12>14k+14$ yani $-6k>2$  yani $k<0$  gelir. bu da $k$ nın negatif olmayan olmasıyla
çelişir.

$k=0$ ise $m-n=1$ $m+n=3^n$ denklem $3^n=2n+1$ olur bunun çözümü $n>0$ için $n=1$  olacağını daha önce göstermiştik.
$k=1$ ise $m-n=3$ $m+n=3^{n-1}$ denklem düzenlenip türev alınırsa $1.3^{n-1}.ln3-2$  $n>3$ için daima artandır, aynı zamanda $3$ konulduğunda ifadenin kendisi de pozitif olduğundan $n>3$ için daima pozitiftir.
Denenirse $n=3$ bulunur. 


$n=1$ ise $n^2+3^n=4$
$n=3$ ise $n^2+3^n=36$ bulunur.
Başlık: Ynt: Diyafont Denklemler Çalışma Soruları (70 Tane)
Gönderen: AtakanCİCEK - Ağustos 17, 2019, 10:50:25 ös
$39)$

$p,q$  tek sayılar , $r,s\ge 0$ olmak üzere $a=p.2^r$ ve $b=q.2^s$ olacak şekilde yazalım.
$$a^n+b^n=2^m$$
$$(p.2^r)^n+(q.2^s)^n=2^m$$
$$p^n.2^{nr}+q^n.2^{ns}=2^m$$
Genelliği bozmadan $r\le s$  alabiliriz.
$$p^n+(2^{s-r}.q)^n=2^{m-nr}$$
Sağ taraftaki iki terim de pozitif olduğundan dolayı sağ taraf ta çift olmalıdır. $p^n$ nin de tek sayı olduğuna dikkat edersek $2^{s-r}.q$ ifadesinin de tek sayı olması gerektiği görülür. $s-r=0$ yani $r=s$ bulunur. O halde ispatımız için $p=q$ olduğunu göstermek yeterlidir.
Denklemi düzenler isek $t=m-nr$ için $p^n+q^n=2^t$ şeklinde bir denklemin çözümü gereklidir.

$1)$ $n$ tek sayı olsun.
$(p,q)\not =(1,1)$  kabul edelim.
O halde $(p+q).(p^{n-1}-p^{n-2}.q+...+q^{n-1})=2^t$ olduğundan sağdaki çarpan da $2n+1$ tane tek sayının toplamı olduğundan dolayı tektir. $p^n+q^n>p+q$ olduğundan dolayı $ p^{n-1}-p^{n-2}.q+...+q^{n-1}>1$ olmalıdır.  Tek olmasıyla çelişir. O halde $p=q=1$ olmalıdır.

$2)$ $n$ çift sayı olsun.
$n=2w$ , $w\in Z^+$  dönüşümü yapılabilir.
$(p^w)^2+(q^w)^2=2^t$ olur. iki tek sayının karesinin toplamı $(mod4)$ altında $2$ kalanını verir $2^t=2$ yani $t=1$ olur. $p^w$ ve $q^w$ pozitif tam sayılar olduğundan alabileceği değerlerin toplamı en az $2$ olmalıdır. Bu durum ise $p^w=q^w=1$ yani $p=q=1$ durumudur.  $p=q$ olduğunu göstermiş olduk.
Başlık: Ynt: Diyafont Denklemler Çalışma Soruları (70 Tane)
Gönderen: AtakanCİCEK - Ağustos 17, 2019, 11:18:55 ös
$40)$ $m=0$ için $n=1$ , $n=0$ için ise $m=3$ olmalıdır. Fakat  $m,n>0$ şeklindeki çözümleri arıyoruz. $2^m+3^n=k^2$ olduğundan dolayı $(2,k)=(3,k)=1$ dir bu nedenle $k$ nın $2$ nin veya $3$ ün katı olmayacağı görülür. $k=6k\pm1$ olur. $k^2\equiv1(mod12)$ bulunur.
Denklem $3$ modunda incelenirse $2^m\equiv 1(mod3)$ yani $m$ çifttir.
Denklem $2^m\ge4$ olduğundan dolayı $2^m\equiv 0(mod4)$ tür.  Bunu kullanırsak $3^n\equiv 1(mod4)$ olur buradan da  $n$ çift olmalıdır.

$m=2M$ $n=2N$ dönüşümleri yapılırsa

$(2^M)^2+(3^N)^2=k^2$ ifadesine dönüşür.  $(2,3)=1$ olduğu için pisagor üçlüsünün ilkel çözümlerine eşittirler.

$2^M=2xy$, $3^N=x^2-y^2$, $k=x^2+y^2$  olur.    $2^{M-1}=xy$ ve $M>1$  olduğundan dolayı $x=2^u$, $ y=2^v$  dönüşümleri yapılabilir.  $u+v>0$ ve $u>v$ elde edilir. $2^{2u}-2^{2v}=3^N$ ifadesinden $v=0$ olduğu görülür.

$2^u+1=3^s$ ve $2^u-1=3^r$ olduğundan dolayı $2^{u+1}=3^r.(3^{s-r}+1)$ buradan ise $r=0$ olması gerektiği görülebilir. $r=0$için $u=1$ $s=1$ gelir.  yani $x=2$ $y=1$  yani $k=2^2+1^2=5$  $3^N=3$, $N=1$ , $2^M=4$, $M=2$   yani  $m=4$ , $n=2$  $k=5$ çözümü gelir. 
Pozitif tam sayılarda çözümü $(4,2,5)$ bulunur.
Başlık: Ynt: Diyafont Denklemler Çalışma Soruları (70 Tane)
Gönderen: Metin Can Aydemir - Ağustos 18, 2019, 12:58:11 öö
$2)$ $t^2=n^4+3n^2+1$ olsun. $$\Rightarrow 4t^2=4n^4+12n^2+4=(2n^2+3)^2-5$$ $$\Rightarrow (2n^2+3)^2-(2t)^2=5\Rightarrow (2n^2-2t+3)(2n^2+2t+3)=5$$ olur. Çarpanların toplamı $4n^2+6$ olduğundan sadece $1\cdot 5$ sağlar fakat $n=0$ olur, yani hiçbir pozitif $n$ için tamkare olamaz.
Başlık: Ynt: Diyafont Denklemler Çalışma Soruları (70 Tane)
Gönderen: Metin Can Aydemir - Ağustos 18, 2019, 02:03:00 öö
$27)$ Öncelikle $x=0$ durumuna bakalım. $7^y+4=3^z$ olur fakat mod $3$'de incelenirse çözüm olmadığı net bir şekilde görülebilir. Dolayısıyla $x>0$'dır. Mod $5$'de denklemi incelersek $$3^z\equiv 4(mod 5)$$ olur ve buradan $z\equiv 2(mod 4)$ bulunur. Denklemi $$(3^{\frac{z}{2}}-2)(3^{\frac{z}{2}}+2)=5^x\cdot 7^y$$ olarak düzenleyelim. Çarpanların farkı $4$ olduğundan ikisi birden $5$'e veya $7$'ye bölünemez. Dolayısıyla $3^{\frac{z}{2}}+2=7^y$ veya $3^{\frac{z}{2}}-2=7^y$ olmalıdır.

$i)$ $3^{\frac{z}{2}}+2=7^y$ ise mod $3$'de incelersek çözüm gelmez.

$ii)$ $3^{\frac{z}{2}}-2=7^y$ ise $3^{\frac{z}{2}}+2=5^x$ olur. Bu ikisini birbirinden çıkartırsak $$5^x-7^y=4$$ olur. Mod $5$'de incelersek $7^y\equiv 1(mod 5)$ olur, buradan $y$, $4$'ün katı olduğu görülür. $$5^x=7^y+4=7^y+4\cdot 7^{\frac{y}{2}}+4-4\cdot 7^{\frac{y}{2}}=(7^{\frac{y}{2}}+2)^2-4\cdot 7^{\frac{y}{2}}$$ $$\Rightarrow 5^x=(7^{\frac{y}{2}}+2)^2-4\cdot 7^{\frac{y}{2}}=(7^{\frac{y}{2}}+2\cdot 7^{\frac{y}{4}}+2)(7^{\frac{y}{2}}-2\cdot 7^{\frac{y}{4}}+2)$$ Çarpanların ikisi birden $5$'e bölünüyorsa farkı da bölüneceğinden $5|4\cdot 7^{\frac{y}{4}}$ olmalı fakat bu imkansızdır. Dolayısıyla çarpanların en az biri $1$ olmalı. $(7^{\frac{y}{2}}+2\cdot 7^{\frac{y}{4}}+2)>(7^{\frac{y}{2}}-2\cdot 7^{\frac{y}{4}}+2)$ olduğundan $$(7^{\frac{y}{2}}-2\cdot 7^{\frac{y}{4}}+2)=1\Rightarrow (7^{\frac{y}{4}}-1)^2=0\Rightarrow y=0$$ olur. $y=0$ ise $x=1$ ve $z=2$ olur. Tek çözüm $(x,y,z)=(1,0,2)$'dir.
Başlık: Ynt: Diyafont Denklemler Çalışma Soruları (70 Tane)
Gönderen: Metin Can Aydemir - Ağustos 18, 2019, 02:43:26 öö
$29)$
Denklemi $y^2$'ye bölelim. $$(\dfrac{x}{y})^2-1=2z\cdot (\dfrac{x}{y})\Rightarrow (\dfrac{x}{y})^2-2z\cdot (\dfrac{x}{y})-1=0$$ Çözüm, $$\dfrac{x}{y}=\dfrac{2z\pm \sqrt{4z^2+4}}{2}=z\pm \sqrt{z^2+1}$$ $\dfrac{x}{y}$ rasyonel olduğundan $z^2+1$ tamkare olmalı.$z$ pozitif olduğundan $$(z+1)^2>z^2+1>z^2$$ olur, yani çözüm yoktur.
Başlık: Ynt: Diyafont Denklemler Çalışma Soruları (70 Tane)
Gönderen: AtakanCİCEK - Ağustos 18, 2019, 09:46:19 öö
Denklemi $y^2$'ye bölelim. $$(\dfrac{x}{y})^2-1=2z\cdot (\dfrac{x}{y})\Rightarrow (\dfrac{x}{y})^2-2z\cdot (\dfrac{x}{y})-1=0$$ Çözüm, $$\dfrac{x}{y}=\dfrac{2z\pm \sqrt{4z^2+4}}{2}=z\pm \sqrt{z^2+1}$$ $\dfrac{x}{y}$ rasyonel olduğundan $z^2+1$ tamkare olmalı.$z$ pozitif olduğundan $$(z+1)^2>z^2+1>z^2$$ olur, yani çözüm yoktur.
bu kaç numaralı soru ekleyebilir misiniz? $29$  numaralı olan sanırım
Başlık: Ynt: Diyafont Denklemler Çalışma Soruları (70 Tane)
Gönderen: AtakanCİCEK - Ağustos 18, 2019, 10:24:17 öö
$43)$

İfadeyi adım adım düzenleyelim.

$$x^{2006}=4y^{2007}+4y^{2006}+2007y+2006$$
$$x^{2006}+1=(4y^{2006}+2007).(y+1)$$ 

Şimdi ise  $$4y^{2006}+2007\equiv 3(mod4)$$  olduğunu görelim.  Buradan $x^{2006}+1$ ifadesinin $4k+3$ formunda çarpanı olması gerektiği görülür. $x^{1003}=t$ dersek


Şimdi $t^2+1$ için geçerli olan özellikler bulalım.

$t=2k$ olursa $t^2+1\equiv 1(mod4)$ olur.

$t=2k+1$ olursa $t^2+1 \equiv 2(mod4)$ olur. $(1)$

Şimdi $t$ nin $2$ dışında asal çarpanlarının tamamının $4k+1$ formunda olduğunu gösterelim.$(2)$

$p\mid t^2+1$ olsun. $t^2\equiv -1(modp)$ olur

$t^4\equiv1(modp)$

 $p\mid t^2+1$ olduğundan $(t,p)=1$ dir.

Fermat teoreminden $t^{p-1}\equiv1(modp)$

Buradan $4$'ün $p-1$ in en küçük katı olduğunu görmek mümkündür $4\mid p-1$ elde edilir.

Buradan $t^2+1$ in $4k+3$ formunda çarpanı bulunmamalıdır. Dolayısıyla denklemin çözümü yoktur.
Başlık: Ynt: Diyafont Denklemler Çalışma Soruları (70 Tane)
Gönderen: AtakanCİCEK - Ağustos 18, 2019, 10:39:02 öö
$2)$  bu soruyu ben de klasik yöntemle çözüme kavuşturayım.
$n\ge1$ için
$$(n^2+1)^2<n^4+3n^2+1<(n^2+2)^2$$ 
olduğundan  tam kare olamaz.  $n=0$  olmalıdır. Fakat pozitif tam sayı olduğu belirtilmiş
Başlık: Ynt: Diyafont Denklemler Çalışma Soruları (70 Tane)
Gönderen: AtakanCİCEK - Ağustos 18, 2019, 12:22:29 ös
$56)$ 

$$x^3-8=3^y.7^z$$ olur.
$$(x-2).(x^2+2x+4)=3^y.7^z$$ olur.

$7\mid x-2$  olursa $7\not \mid x^2+2x+4$ olur. Dolayısıyla her ikisi birden $7$  ile bölünemez.

$1)$  \begin{equation*}
\begin{cases}
x-2=3^a.7^z,
\\
x^2+2x+4=3^{y-a}
\end{cases}
\end{equation*} sistemini kabul edelim.
İlk denklem ikinci denklemde yerine konulursa  $$(7^z.3^a+2)^2+2.3^a.7^z+4=3^{7-a}$$
$a\ge 1$ ise  Eşitliğin sol tarafı $8(mod3)$ gelir. Yani çelişki oluşur.  $a=0$ olmalıdır.

\begin{equation*}
\begin{cases}
x-2=7^z,
\\
x^2+2x+4=3^y
\end{cases}
\end{equation*}

 tekrar düzenlersek $7^{2z}+6.7^z+12=3^y$  elde edilir.  Eşitliğin sol tarafı asla $3$  ün katı olamaz.  O halde kabulümüz yanlıştır.


$2)$

\begin{equation*}
\begin{cases}
x-2=3^b,
\\
x^2+2x+4=3^{y-b}.7^z
\end{cases}
\end{equation*}

Sistemi düzenlersek $3^{2b}+6.3^b+12=7^z.3^{y-b}$ 

$b=0$ olmadığını görelim. $b=0$  için $19=7^z.3^{y-b}$ olur, çözüm yoktur.
$b\ge1$ için ise sol taraf $9$ ile bölünemez O halde $y=b+1$ olmalıdır.

$$3^{2b-1}+2.3^b+4=7^z$$
Denkleminin çözümlerini bulmalıyız.

Denklemin sol kısmı $4$ modunda incelenmelidir. 

$$3^{2b-1}+2.3^b+4\equiv (-1)^{2b-1}+2.(-1)^b\equiv -1+2.(-1)^b\equiv 1(mod4)$$

Buradan $7^z\equiv 1(mod4)$ Bu ise $z$ çift iken sağlandığı için $z=2n$ dönüşümü yapalım ve $4$ ü eşitliğin karşı tarafına atıverelim.

$$3^b.(3^{b-1}+2)=(7^n-2).(7^n+2)$$  $3\not \mid 7^n-2$ olduğu açıktır.  $7^n-2=3^{b-1}+2$ 

\begin{equation*}
\begin{cases}
7^n-2=\dfrac{3^{b-1}+2}{t} 
\\
7^n+2=3^b.t
\end{cases}
\end{equation*}

 elde edilir.  Buradan düzenleme yapıldığında $3^{b-1}=\dfrac{4t+2}{3t^2-1}$ gelir  bu ifade de ancak $t=1$ için tam sayıdır. $b=2$ gelir .

$y=b+1=3$ gelir. $b=2$ $t=1$ olduğuna göre $n=1$ gelir  $z=2$ bulunur.  İlk denklemde yerine koyarak $x=11$ elde edilir. 

Denklemin tek çözümü $(11,3,2)$  olarak bulunur.

Başlık: Ynt: Diyafont Denklemler Çalışma Soruları (70 Tane)
Gönderen: AtakanCİCEK - Ağustos 18, 2019, 12:36:40 ös
$51)$  Bu çözümü geo'dan  aldım.

$3367$ nin asal çarpanlara ayrılışı $3367=7\cdot 13\cdot 37$ dir. Eğer $x^3\equiv 2^n\ \ (mod\ 7)$ ise, uygun bir $m\in N$ için $n=3m$ olur. Böylece, $3367=2^n-x^3=\underbrace{\left(2^m-x\right)}_{a}\underbrace{\left(2^{2m}+2^mx+x^2\right)}_{b}$, $a^2<b$ ve $ab=7\cdot \ 13\cdot 37$ olduğu için aşağıdakilerden biri doğrudur:
$b-a^2=3\cdot 2^m\cdot x$ , $2^m\ge \sqrt[3]{3367}>14$ olduğu için

$(i)$ $b-a^2=3\cdot 2\cdot 561$ ve
$(iii)$  $b-a^2=90=2\cdot 3\cdot 15$ geçerli olamaz; ancak $(ii)$ durumu sözkonusu olabilir. Bu durumda $b-a^2=481-49=432=3\cdot 2^4\cdot 3^2$ olduğunda $n=12$, $x=9$ olmalı. Gerçekten $9^3+3367=2^{12}\ $'dir.
Başlık: Ynt: Diyafont Denklemler Çalışma Soruları (70 Tane)
Gönderen: AtakanCİCEK - Ağustos 18, 2019, 12:39:54 ös
$53)$  Bu çözüm de geo hocamdan

Denklemi $\bmod 4$'te incelediğimizde $1^m + (-1)^n \equiv 0,1,0,3 \pmod 4$ olacağı için $n$ tek olmalı.

$\bmod 7$'de
$5^m$ in kalan sınıfı $\{5,4,6,2,3,1\}$,
$k^3$ in kalan sınıfı $\{1,1,6,1,6, 6, 0\}$
olduğu için $m \equiv 3 \pmod 6$ yani $m=3a$ olmalı.
Bu durumda denklem $5^{3a} + 7^n = k^3$ e dönüşür.
$7^n = k^3 - (5^a)^3 \Rightarrow 7^n = (k - 5^a)\left( (k-5^a)^2 + 3\cdot k \cdot 5^a  \right )$
İkinci çarpan ilkinden büyük olduğu için $k-5^a \equiv 0 \pmod 7$ olduğunda $3\cdot k \cdot 5^a \equiv k \equiv 0 \pmod 7$ gerekeceği için $k-5^a \equiv 0 \pmod 7$ olamaz.
Bu durumda geriye sadece $k-5^a = 1$ durumu kalıyor. Yerine yazarsak $1+3\cdot k\cdot 5^a = 7^n$ elde ederiz. $a\geq 1$ için $\bmod 5$'te incelersek, $n \equiv 0 \pmod 4$ elde ederiz ki bu başlangıçta bulduğumuz $n$ tek olmalı yargısıyla çelişir. O halde $a\geq 1$ için bir çözüm yok.
Geriye sadece $a=0$ ve dolayısıyla $k-5^a = 1 \Rightarrow k=2$ durumu kalıyor. Bu durumda denklemin tek çözümü $(m,n,k)=(0,1,2)$ oluyor.
Başlık: Ynt: Diyafont Denklemler Çalışma Soruları (70 Tane)
Gönderen: AtakanCİCEK - Ağustos 18, 2019, 12:41:53 ös
$54)$  Bu çözüm de önceden geo hocamın paylaştığı çözümlerden

$k>1$ olduğundan $p\ne 7$ öncelikle $128\equiv 2^{7} \equiv 1 \pmod {127}$ olduğundan $der_{127} 2=7$ olduğunu söyleyebiliriz. Diğer taraftan $127$ bir asal sayıdır ve dolayısıyla:
$$(127,m)>1\Leftrightarrow 127|m\Leftrightarrow 127|2^{p} -1\Leftrightarrow 2^{p} \equiv 1 \pmod {127}\Leftrightarrow 7|p$$
bulunur ki bu $p>7$ olduğundan mümkün değil.

$\Rightarrow (127,m)=1$. Dolayısıyla ayrı ayrı $m|2^{m-1} -1$ ve $127|2^{m-1} -1$ olduğunu ispatlamamız yeterlidir.

$127|2^{m-1} -1\Leftrightarrow 2^{m-1} \equiv 1 \pmod {127} \Leftrightarrow 7|m-1\Leftrightarrow 7|2^{0} -2\Leftrightarrow 2^{p-1} \equiv 1 \pmod 7$ bulunur. Diğer taraftan $2^{6} \equiv 1 \pmod 7$ ve $6|p-1$ olduğundan $2^{p-1} \equiv 1 \pmod 7$ ve de $127|2^{m-1} -1$ doğrudur.

Ayrıca $m=2^{p} -1|2^{m-1} -1$ olduğunu ispatlamak için, $p|m-1$ olduğunu göstermek yeterlidir, çünkü böylece $m-1=pt$ olur ve $2^{m-1} -1=(2^{p} -1)(2^{p.(t-1)} +2^{p.(t-2)} +\dots+1)$ şeklinde yazılabilir.

Diğer taraftan $2^{p} \equiv 2 \pmod p \Leftrightarrow m\equiv 2^{p-1} \equiv 1 \pmod p$ denkliği Küçük Fermat Teoremi'nden sağlanır, ispat biter.
Başlık: Ynt: Diyafont Denklemler Çalışma Soruları (70 Tane)
Gönderen: AtakanCİCEK - Ağustos 18, 2019, 12:43:23 ös
$55)$  MATSEVER 27  nin paylaştığı çözüm

$p \mid n$ olsun. $p \neq 2$ sayısı $2^n$ yi bölmez. O halde $m!$ i de bölmez. Buradan $n$ nin her asal böleni $p$ için $p>m$ elde edilir.

(i.) $n$ nin $2$ hariç en az iki asal böleni olursa $p,q$ onlardan ikisi olsun, $n \ge pq>m^2$ elde edilir.  $m! > 2^{m^2}+m^2$ olmalıdır. Ancak  ($m$ tane $2^m$ nin çarpımı) $2^{m^2}=2^{m}.2^{m}\cdots2^{m} >1.2 \cdots m$ o halde buradan çelişki.

(ii.) $n=p^{a}.2^{b}$ olmalıdır. $a >1$ ise $p>m$ den dolayı $n > m^{a}$ dır. $a\ge 2$ ise $n >m^2$ olur ki çelişki geleceğini az önce ispatlamıştık. O halde $a=1$ olmalıdır. $n=p.2^{b}$ olur. $p \neq 2$ olduğundan $2^b $ $||$ $m!$ olur. O halde $m$ de tam olarak $b$ tane $2$ çarpanı olmalıdır. Ancak $m>2^b$ olduğundan çelişki. $n=2^{a}$ olması gerekir. $m \ge 3$ olduğundan $m!$ in çift olduğunu biliyoruz. O halde $2^{p^a}+p^a$ çift olmalı. $p=2$ olmalı. $2^{2^a}+2^a=m!$ olmalı. $m \ge 3$ için $3 \mid m!$ olduğundan $3 \mid 2^{2^a}+2^a$ o halde $a$ tek olmalı. $m \ge 5$ için $5 \mid   2^{2^a}+2^a$ olmalı. $a=1$ ise 5 ile bölünmez. $a\ge 2$ için $2^{2^a} \equiv 1 \pmod{5}$ yani $2^a \equiv 4 \pmod{5}$ olmalı. Ancak $a$ tek olduğundan çelişki! $m=3,4$ olabilir. $m=3$ için $n=2$ sağlar.

(iii.) $n=1$ olabilir. Buradan çözüm yoktur.

$(3,2)$ tek köktür. İspat biter.
Başlık: Ynt: Diyafont Denklemler Çalışma Soruları (70 Tane)
Gönderen: AtakanCİCEK - Ağustos 18, 2019, 12:45:36 ös
$57)$  Metonster 'in yaptığı bir çözüm

Soruyu üç durumda inceleyelim.
$i)$ $m=n$ ise $k=m^2$ olur.

$ii)$ $m>n$ ise $m-n=x, ~m+n=y$ diyelim. $m=\dfrac{x+y}{2}$ ve $n=\dfrac{y-x}{2}$ olur.$4|(m+n)^2$ olduğundan $y$ çifttir.Dolayısıyla $x$ de çifttir. Yerine yazarsak, $$k=\dfrac{y^2}{2(x+y)x^2+4} \Rightarrow y^2-2kx^2y-(2kx^3+4k)=0$$ olur. Diskriminantı tamkare olmalı. $$\Delta = 4k^2x^4+8kx^3+16k=4t^2 \Rightarrow t^2=k^2x^4+2kx^3+4k$$ olur. $k\geq 1$ ve $x\geq 2$ olduğunu biliyoruz buradan, $$(kx^2+x+1)^2>k^2x^4+2kx^3+4k>(kx^2+x-1)^2$$ bulunur. $k^2x^4+2kx^3+4k=(kx^2+x)^2$ olmalı buradan $k=(\dfrac{x}{2})^2$ bulunur.

$iii)$ $m<n$ ise $n-m=x,~m+n=y$ diyelim.Aynı şekilde $x$ ve $y$'yi çift bulabiliriz. Yerine yazar ve düzenlersek, $$y^2-2kx^2y+2kx^3-4k=0\Rightarrow \Delta =4k^2x^4-8kx^3+16k=4t^2\Rightarrow t^2=k^2x^4-2kx^3+4k$$ olur. $$(kx^2-x+1)^2>k^2x^4-2kx^3+4k>(kx^2-x-1)^2\Rightarrow k^2x^4-2kx^3+4k=(kx^2-x)^2\Rightarrow k=(\dfrac{x}{2})^2$$ bulunur. Yani $k$ her zaman tamkaredir.
Başlık: Ynt: Diyafont Denklemler Çalışma Soruları (70 Tane)
Gönderen: AtakanCİCEK - Ağustos 18, 2019, 01:19:36 ös
$59)$ 

Bu soruyu çözerken $c$ herhangi bir tam sayı olduğu için genelliği bozmadan pozitif tam sayılarda çözümleri bulursak geri kalan çözümleri bulmak kolaydır.

Bu soruyu $\dfrac{a^2+b^2+3}{ab}\in Z$ olarak düşünebiliriz.  $a=b$  olduğunu varsayarsak $2+\dfrac{3}{a^2}$  gelir ve $a=1$ olmalıdır. $a=b=1$ yerine koyulursa $c=5$  elde edilir.

Genelliği bozmadan $a>b$ kabul edebiliriz.

$$a^2-abc+(b^2+3)=0$$ denkleminin diğer çözümü $x$  olsun. 

$$a+x=bc$$ $$ax=b^2+3$$

elde edilir. Buradan $x$  in de tam sayı olacağı görülür.

Genelliği bozmadan $x\ge a$  olsun. O halde

$$a^2+3>b^2+3=ax\ge a^2$$ $a\ge3$  için
$$a^2>b^2\ge a^2-3 >(a-1)^2$$ olduğundan dolayı çözüm yoktur. 

$a=2$ için $a>b$ kabulünden dolayı olası tek çözüm $(2,1)$ dir ki sağlar $c=4$  bulunur.  Geri kalan çözümler ise bunların işaretlerinin ve $a$ ile $b$ nin değerlerinin yer değiştirmeleriyle gelecektir.

$(1,1)$  ikilisinden yararlanarak $(1,1,5)$ $(-1,1,-5)$  ,$(1,-1,-5)$ , $(-1,-1,5)$ çözümü gelir.

$(2,1)$  ikilisinden yararlanarak $(2,1,4)$ , $(1,2,4)$ , $(2,-1,-4)$ , $(-1,2,-4)$  , $(-2,1,-4)$ , $(1,-2,-4)$   çözümlerini söyleriz. Bu denklemin $10$  çözümü vardır.

Başlık: Ynt: Diyafont Denklemler Çalışma Soruları (70 Tane)
Gönderen: AtakanCİCEK - Ağustos 18, 2019, 01:25:28 ös
$60)$  Çözümü geo hocamızdan aldım.

$y$ yi sabit tuttuğumuzda,
$f(a,y) = f(b,y)$ ise $a^2-ay+y^2 = b^2 - by + y^2 \Rightarrow (a-b)(a+b-y)=0$ olacağı için $b=y-a$ elde edilir. Bu durumda $(a,y)$ bir çözüm ise $(y-a,y)$ de bir çözümdür.
Benzer şekilde $x$ i sabit tuttuğumuzda, $f(x,a)=f(x,b)$ ise  $x^2 - ax + a^2 = x^2 - bx+b^2 =0 \Rightarrow (a-b)(a+b-x) $ ve $b=x-a$ elde edilecek. Bu durumda $f(x, a)$ bir çözüm ise $f(x, x-a)$ da bir çözüm olacak.
Bunun haricinde simetriden dolayı $f(x,y)=f(y,x)$ ve kare ifadelerden dolayı $f(x,y)=f(-x,-y)$ olduğu görülüyor.
Tüm çözümleri birleştirirsek:
Sabit tutma sonucu $3$ tane: $(x,y)$, $(y-x, y)$, $(x, x-y)$
Yer değiştirme sonucu $3$ tane: $(y,x)$, $(x-y, x)$, $(y, y-x)$
Bunların eksileri sonucu $6$ tane çözüm geleceği için, $(x,y)$ çözümse, bunun haricinde $11$ çözüm daha vardır.
$(x,x)$ olma durumunda çözümler $(x,x)$, $(-x,-x)$, $(0,x)$, $(x,0)$, $(0,-x)$, $(-x,0)$ olacak. Çözüm sayısı yine $3$ ile bölünüyor.
$(0,0)$ çözüm ise çözüm sayısı $1$ olacak. Ama $(0,0)$ çözümse, $n=0$ olması gerekeceği ve soruda $n$ pozitif tam sayı dediği için $(0,0)$ çözüm olamaz.
Başlık: Ynt: Diyafont Denklemler Çalışma Soruları (70 Tane)
Gönderen: AtakanCİCEK - Ağustos 18, 2019, 01:28:24 ös
$61)$  Merdan97 isimli üyeden aldığım çözüm

$y$ tek olursa sağ taraf çift olur bu imkansız.
$y$ çift olmalı. o zaman sağ taraf $8$'e bölününce $1$ kalanını verir ki sol taraf da aynı kalanı vermeli $x$ çift olmalı. sol taraf tamkare olur.
Sağ taraftaki ifadeyi tamkare olmasına göre inceleyelim.
$y\ge0$ ise
$(y^2+2)^2 > y^4+4y+1>(y^2)^2$
$y^4+4y+1=(y^2+1)^2$  olmalı.
$2y^2-4y=0$
$y=0$ , $y=2$
$y=0\Rightarrow y^4+4y+1=1=5^x$ 
$x=0$ olur.
$y=2\Rightarrow y^4+4y+1=25=5^x$
$x=2$ olur.
Buradan $(x,y)=(0,0)$ ve $(x,y)=(2,2)$ bulunur.


$y< 0$ ise $a>0$ olmak üzere $a=-y$ diyelim.
$y^4+4y+1=a^4-4a+1$
$a=1,2$ için çözüm gelmez $a>2$ alabiliriz.
$(a^2+1)^2> a^4-4a+1 > (a^2-1)^2$
$a^4-4a+1=(a^2)^2=a^4$  olur. Buradan a tamsayı olmaz çözüm gelmez.
Tüm çözümler $(x,y)=(0,0)$ ve $(x,y)=(2,2)$ olur.
Başlık: Ynt: Diyafont Denklemler Çalışma Soruları (70 Tane)
Gönderen: AtakanCİCEK - Ağustos 18, 2019, 01:31:33 ös
$62)$ MATSEVER 27  den aldığım bir çözüm

Eğer $n$ bir tek sayı ise $\left(n^{\frac{n+1}{2}}\right)^2<n^{n+1}+n-1<\left(n^{\frac{n+1}{2}}+1\right)^2$ olduğundan eğer  $n \ge 2$ ise buradan çözüm gelmeyeceğini söyleyebiliriz. $n=1$ için sağlar.

Eğer $n \equiv -1$ $\text{(mod 3)}$ ise $\left(n^{\frac{n+1}{3}}\right)^3<n^{n+1}+n-1<\left(n^{\frac{n+1}{3}}+1\right)^3$ olduğundan buradan da $n \ge 2$ için çözüm gelmeyeceğini söyleyebiliriz. $n=1$ i saymıştık.

Eğer $n \equiv 0$ $\text{(mod 3)}$ ise $m^6 \equiv -1$ $\text{(mod 3)}$ olur ve buradan da çözüm gelmez.

O halde $n \equiv 4$ $\text{(mod 6)}$ diyelim. $n+1 | n^{n+1}+n+2=y^6+3$ olduğunu söyleyebiliriz. Buradan $n+1 \equiv 5$ $\text{(mod 6)}$ olur. Buradan $n+1$ i bölecek şekilde bir  $p \equiv 2 \pmod{3}$ olacak şekilde bir $p$ asalının varlığını bilebiliriz. $y^6 \equiv -3 \pmod {n+1}$ idir. O halde $ y^6 \equiv -3 \pmod{p}$ olur. Ancak bir tamkare  $p \equiv 2 \pmod{3}$ olmak üzere $\pmod{p}$ de $-3$ kalanını veremez. $p >2$ idir.

İspat: Diyelim ki bir $x$ için $ x^2 \equiv -3 \pmod{p}$ olsun. Şimdi de $2y+1 \equiv x \pmod p$ olacak şekilde bir $y$ seçelim. Buradan $y^2+y+1 \equiv 0 \pmod p \Rightarrow y^3 \equiv 1 \pmod p$ olur. O halde $y$ nin $\pmod p$ deki mertebesi $d$ olmak üzere $(d,3)=1,3$ olabilir. $=1$ ise $y \equiv 1 \pmod p$ olur. $p=3$ olması gerekir. Çelişki! $=3$ olsa $3|d|p-1$ olması gerekir. Çelişki! Kabul yanlıştır ve böyle $x$ ler yoktur.

O halde bu durumdan da çözüm gelmez ve ispat biter. Yalnızca  $n=1$ sağlar.
Başlık: Ynt: Diyafont Denklemler Çalışma Soruları (70 Tane)
Gönderen: AtakanCİCEK - Ağustos 18, 2019, 01:48:25 ös
$63)$  Mehmet Utku Özbek'in yaptığı çözümü düzenledim.

İfadeyi $m$ parantezine alalım.

$\Longrightarrow m[\ p^{2l-1}(mn+1)^2+m]=x^2$         

Eğer $m$ tam kare değilse  çarpanlarından biri $q$ olacak şekilde ve üssü tek sayı olacak şekilde bir çarpanı vardır. $q^k$ alalım.  $q$ nun ikinci parantezi bölmesi gerekir.  $q$  ile $(mn+1)^2$  aralarında asal olduğu için $q \mid p^{2l-1}$   olmalıdır.   O zaman $q=p$  olmalıdır.  Eğer $k$ nın tek olmaması gerektiğini gösterirsek ispat biter.  Şimdi  ifadede $m=p^k$  olduğunu söyleyebiliriz.

$\Longrightarrow m[\ p^{2l-1}(mn+1)^2+m]=p^k[\ p^{2l-1}(p^{k}n+1)^2+p^k]=p^{2k}[\ p^{2l-k-1}(p^{k}n+1)^2+1]=x^2$

Dolayısıyla son ifadedeki ikinci parantez de tam kare olmalıdır.

$\Longrightarrow p^{2l-k-1}(p^{k}n+1)^2+1=y^2$

Eğer $k$ tek olursa $2l-k-1$  çift olacağından ifade  aslında $c=p^{\tfrac{2l-k-1}{2}}(p^{k}n+1)$   olmak üzere   $c^2+1=y^2$  halindedir.  Bunun da pozitif tamsayılarda çözümü yoktur. Yani $k$ tek olamaz. İspat biter.
Başlık: Ynt: Diyafont Denklemler Çalışma Soruları (70 Tane)
Gönderen: AtakanCİCEK - Ağustos 18, 2019, 02:24:45 ös
$64)$ 

Kaba bir ispatla sayının $2$ basamaklı olduğunu gösterelim.

Varsayalım ki $x$ sayısı $n$ basamaklı olsun.

$x^2\ge 10^{2n-2}$ olduğundan $2n-1$ basamaklı en küçük sayıdır ve diğer $2$ terim negatif olduğu için basamaklar çarpımının minimum değeri $x^2-10x-22 > 10^{2n-3}$

Sayının basamaklar çarpımının maximum değeri ise $n$  tane $10$ çarpılmış olsaydı bile $10^n$ olurdu.

Basamaklar çarpımının maximum değeri minimum değerinden büyük veya tek değer alabiliyorsa eşit olacağından dolayı $10^n > 10^{2n-3}$  yani $n < 3$ bulunur. 

$x$ tek basamaklı ise $x=x^2-10x-22$  olacağından dolayı $\bigtriangleup$ tamkare olmadığından çözüm gelmez.

$x$ iki basamaklı ise $x=ab$  alalım.  $a.b=(10a+b)^2-10.(10a+b)-22$
 
$a\ge 2$ için  $b$  rakam olmak üzere

$$100a^2+19ab+b^2-100a-10b-22>0$$

olduğundan denklemin çözümü yoktur. $a=1$ olmalıdır.

$b^2+9b-22=0$  bulunur. Çözülürse $b=2$  bulunur. $x=12$  sağlar. 
Başlık: Ynt: Diyafont Denklemler Çalışma Soruları (70 Tane)
Gönderen: AtakanCİCEK - Ağustos 18, 2019, 03:14:39 ös
$66)$

Bölme Algoritmasını uygulayalım.

$$\dfrac{n^3+1}{mn-1}=qn+r$$ $q\ge 0 $  , $0\le r<n$

Denklemi içler dışlar çarpıp düzenlersek

$$n^3+1=qmn^2+rmn-qn-r$$

Denkleme $n$ modunda bakarsak $1\equiv -r(modn)$  yani $r\equiv -1(modn)$  bölme algoritmasını da göz önüne alırsak $r=n-1$ olmalıdır.

Denklemi tekrar düzenlersek $n^2=qmn+(n-1).m-q-1$ elde edilir.

$m^3.\dfrac{n^3+1}{mn-1}=\dfrac{m^3n^3-1}{mn-1}+\dfrac{m^3+1}{mn-1}$ olduğundan ilk $3$ terim tam sayı olduğundan $\dfrac{m^3+1}{mn-1}$  de bir pozitif tam sayıdır. Genelliği bozmadan $m\ge n$ alabiliriz.

$n=1$ ise $1=qm-q-1$  $2=qm-q$ $q\mid 2$ olduğundan $q=1$ veya $q=2$ dir.

$q=2$  ise $m=2$    $(2,1)$ ve simetriği
$q=1$ ise $m=3$ olur.  $(3,1)$ ve simetriği gelir.

$m\ge n\ge 2$ olsun. O zaman

$n^2\ge qn^2+n.(n-1)-q-1$
$n+1\ge q.(n^2-1)$
$q.(n-1)\le 1$
$n\ge 2$ olduğundan  $q\le 1$ elde edilir.

$q=1$ için $n^2\ge qn^2+n.(n-1)-q-1$  eşitsizliğine tekrar bakalım .

$n^2\ge 2n^2-n-2$
$n^2-n-2\le 0$ $n\ge 2$  olduğundan dolayı $n=2$ olası tek çözümdür. Denklemde yerine konulursa $m=2$ çıkar.

$q=0$ için

$n^2=mn-m-1$
$$n^2+1=m.(n-1)$$
$$\dfrac{n^2+1}{n-1}\in Z$$ Polinom bölmesi yardımıyla  $$\dfrac{2}{n-1}\in Z^+$$ olmalıdır.  $n=2$ ve $n=3$ çözümleri gelir.
$n=2$  için $m=5$
$n=3$ için $m=5$  olur. 

Denklemin tüm çözümleri bunların simetrikleri ile birlikte  $\{(1,3),(3,1),(1,2),(2,1),(2,2),(5,3),(3,5),(2,5),(5,2)\}$ şeklinde $9$  çözümü vardır.
Başlık: Ynt: Diyafont Denklemler Çalışma Soruları (70 Tane)
Gönderen: AtakanCİCEK - Ağustos 18, 2019, 07:42:41 ös
$67)$   $\dfrac{a^2}{2ab^2-b^3+1}=k$ , $k\in Z^+$ diyerek işe koyulalım .

İçler dışlar çarpıp ifadeleri tek tarafta toplarsak

$$f(x)=x^2-2kb^2.x+k.(b^3-1)=0$$ denkleminin çözümleri $b>1$ için iki adet pozitif tam sayı köktür.

$b=1$ durumunu özel olarak incelersek $(2n,1)$  çözümünü görürüz.

pozitif tam sayı köklerden birinin $x=a$ olduğunu görürüz. Diğer kökü $a'$ olsun.

Genelliği kaybetmeden $a'\ge a$ alınabilir. $$a^2\le a.a'=k.(b^3-1)$$  gelir.

Başlangıçtaki ifadeyi düzenleyelim.   $$k=\dfrac{a^2}{2ab^2-b^3+1}\le b.\dfrac{k^3-1}{2ab^2-b^3+1}$$   

$$b^3-1\ge 2ab^2-b^3+1$$

$$b^3-1\ge ab^2$$

$$b>b-\dfrac{1}{b^2}\ge a$$ olduğundan $b>a$ bulunur.

Aynı zamanda başlangıçtaki denklem $$b^2>a^2=k.(2ab^2-b^3+1)\ge 2ab^2-b^3+1>0$$

yazılabildiğinden dolayı $$b^2>(2a-b).b^2+1>0$$ elde edilir.  $2a-b>0$ için sol kısma bakınca $0>1$ çelişkisi gelir.  $2a-b<0$  ise  $2a-b=-x$ ,$x\in Z^+$ olmalıdır.  $-xb^2+1>0$ çelişkisi gelir.  $2a=b$ olmalıdır.   

yani $(a,b)$  ikililerinden biri daha $(n,2n)$ olarak bulunur.  Yerine konulduğunda $k=n^2$  yapar.

$$f(x)=x^2-2kb^2.x+k.(b^3-1)=x^2-8n^3x+8n^4-n=0$$  Denkleminin köklerinden biri $n$  olduğuna göre

$n.a'=8n^4-n$  $a'=8n^3-n$ elde edilir.  O halde $(8n^3-n,2n)$  de bir çözümdür.
Başlık: Ynt: Diyafont Denklemler Çalışma Soruları (70 Tane)
Gönderen: AtakanCİCEK - Ağustos 18, 2019, 08:01:59 ös
$44)$

$n=3$  için $(1,1)$ bir çözümdür.

$7x_n^2+y_n^2=2^n$  denkleminin sağlandığını kabul edelim.  Öyle ki  $7X_n^2+Y_n^2=2^{n+1}$  çözümü bulunduğunu gösterelim.
 
$$7.(\dfrac{x_n\pm y_n}{2})^2+(\dfrac{7x_n\mp y_n}{2})^2=2.(7x_n^2+y_n^2)=2^{n+1}$$  Olduğundan dolayı $x_n+y_n$  çift olacak şekilde  bir çözüm daima vardır. $n=3$ için $(1,1)$ çözümü olduğundan $n=4$ için bu özelliği kullanarak örneğin $(0,4)$  ya da $(1,3)$ çözümlerini görürüz.
Başlık: Ynt: Diyafont Denklemler Çalışma Soruları (70 Tane)
Gönderen: AtakanCİCEK - Ağustos 18, 2019, 08:27:54 ös
$42)$

$p^mq^n=(p+q)^2+1=p^2+2pq+q^2+1$ olduğu için $p\mid q^2+1$  ve $q\mid p^2+1$  olduğu açıktır.

$p=q$ olduğunu düşünelim. $p\mid p^2+1$ olur. $p\mid1$ çıkar ki bu mümkün değildir.

Genelliği bozmadan $p<q$ alalım. $p=1$  durumu $q=2$  durumunu gerektirir ki mümkün değildir. $q>p\ge2$

$$p^mq^n=(p+q)^2+1<4q^2<p^2q^2<pq^3\le p^mq^3$$ eşitsizliklerinden $n<3$  olduğu açıktır.

$n=2$  için $p^m<4$ olur. $m=1$ , $p=2$  veya $p=3$ olduğunu kullanalım. 

   $p=2$ için $$2q^2=(2+q)^2+1$$ olur ve bu $q=5$ için çözümdür.

   $p=3$ için $$3q^2=(3+q)^2+1$$ olur çözümü yoktur.

$n=1$  için  $p^m<4q$ gelir.  $q\mid p^2+1$ olduğunu biliyoruz. 

$q=p^2+1$ ise $p\mid q^2+1=p^4+2p^2+2$ bu da $p\mid2$ olduğunu gösterir. $p=2$  olduğunu gösterir bu durumda $p=5$ bulunur. Sağ taraf $25$ ile bölünürken sol taraf $5$ ile bölüneceğinden çözüm yoktur.

Bu nedenle $q\le \dfrac{p^2+1}{2}$  olur.

$p^m<2.(p^2+1)$ elde  edilir.  $p=2$ için $m\le 3$ ,  Diğer durumlarda $m\le2$ bir çözümdür.

$p=2$ olması demek $q\le \dfrac{5}{2}$ olması demektir.  fakat $q>p$ den dolayı çelişki gelir.

$m\le2$ olduğu bulunur.

   $m=1$ için $$pq=(p+q)^2+1$$ olur  çözüm gelmeyeceği açıktır.

   $m=2$  için $p^2<4q$  olur. $q\mid p^2+1$  olduğu da bilindiğinden $p^2+1=q,2q,3q,4q,$ olasılıkları ortaya çıkar.

    $p^2+1$ ifadesi $(mod3)$ veya $(mod4)$  te $0$ kalanı vermeyeceğinden ve $p^2+1=q$ durumunu zaten incelediğimiz için
     
     $p^2+1=2q$ olur. $p^4+2p^2+5=4q^2+4$ olur.

      $p\mid q^2+1$ olduğundan $4p\mid p^4+2p^2+5$ elde edilir. Buradan $p\mid 5$ gelir. $p=5$ bulunur.  $n=1$ $m=2$  durumu olduğunu da         

       göz önüne alırsak $q=13$ çözümü gelir. 

     Şimdi denklemin tüm çözümlerini sıralayabiliriz.

       $$(m,n,p,q)\in \{(2,1,5,2),(2,1,5,13),(1,2,2,5),(1,2,13,5)\}$$
Başlık: Ynt: Diyafont Denklemler Çalışma Soruları (70 Tane)
Gönderen: AtakanCİCEK - Ağustos 18, 2019, 10:12:39 ös
$72)$  Scarface hocamın youtubede paylaştığı çözümü yazıyorum.

$(0,0)$ çözümü açıktır.

artık $m>0$  olmalıdır.

$7n^2=m^3+15m$  olduğundan  denkleme $7$  modunda bakılırsa $m\equiv0(mod7)$  olduğu görülür.



Düzenlenirse $n^2=k.(49k^2+15)$ olur.

$(k,49k^2+15)=(k,15)=\{1,3,5,15\}$ olmalıdır.

$(k,49k^2+15)=1$ ise  $49k^2+15=z^2$ 

\begin{equation*}
\begin{cases}
z-7k=1,
\\
z+7k=15
\end{cases}
\end{equation*}

çözülürse $k=1$  $m=7$  gelir. $(7,8)$ ,$(7,-8)$  gelir.

$(k,49k^2+15)=3$  olursa

$k=3t$  denilip dönüşüm yapılınca

$n^2=9t.(147t^2+15)$  ayrı ayrı tamkare olmalıdırlar.  $z^2=147t^2+15$  $z^2\equiv2(mod3)$  ten çelişki gelir.

$(k,49k^2+15)=5$  olursa

$n^2=25.t.(49.5t^2+3)$  ayrı ayrı tamkare olmalıdırlar. $z^2=49.5t^2+3$  $z^2\equiv 3(mod5)$

$(k,49k^2+15)=15$  olursa

$n^2=225.t.(49.15t^2+1)$ ayrı ayrı tam kare olmalıdır.   

$49.15t^2+1=z^2$  olsun.   $z^2-735t^2=1$ pell denklemi elde edilir.  Sürekli Kesir açınımları yöntemi kullanılırsa $1$  pozitif tam sayı çözümü

bulunacaktır. Aynı zamanda $(m,n)$ çözüm ise $(m,-n)$  de bir çözüm olacağından $2$ farklı çözümü vardır.

Yani toplam $1+2+2=5$ çözümü vardır.
Başlık: Ynt: Diyafont Denklemler Çalışma Soruları (70 Tane)
Gönderen: Metin Can Aydemir - Ağustos 20, 2019, 02:37:05 ös
$70)$ Denklemi birkaç durumda inceleyeceğiz ama önce $m=0,1,2,3,4$ için incelersek $(m,n)=(0,-1),(2,17)$ çözümlerini elde ederiz.

$i)$ $m\geq 5$ ise,

$ia)$ $m\geq n$ ise, $$m^3(m-2)+m\geq n^3(m-2)+n=m^4+1\Rightarrow 2m^3+1\leq m$$ olur. Çelişki

$ib)$ $n\geq m+1$ ise, $$m^4+1=n^3(m-2)+n\geq (m+1)^3(m-2)+(m+1)\Rightarrow 0\geq m^3-3m^2-4m-2$$ olur. $m^3-3m^2-4m-2$ fonksiyonu $m\geq 5$ için pozitiftir. Çelişki

$ii)$ $m<0$ ise, $n=0$ için çözüm yoktur.

$iia)$ $n>0$ ise $mn^3+n\leq 0$ fakat $m^4+2n^3+1>0$ olacağından çözüm gelmez.

$iib)$ $n<0$ ise, $m=-a$, $n=-b$ olsun. $a,b>0$ ve $$a^4-2b^3+1=ab^3-b$$ olur. $a=1$ için $b=1$ olur ve buradan $(m,n)=(-1,-1)$ çözümü gelir. $a>1$ için, $$f(b)=b^3(a+2)-b-(1+a^4)$$ olsun. $f'(b)=3b^2(a+2)-1>0$ olur. Yani $f$ fonksiyonu artandır. $$f(a)=2a^3-a-1>0$$ ve $$f(a-1)=-a^3-3a^2+4a-2<0$$olur. Artan fonksiyon olduğu için $f$ fonksiyonun tek çözümü $(a-1,a)$ aralığında olması gerekir fakat tamsayı olamaz. Dolayısıyla buradan çözüm gelmez.

Tüm çözümler $(m,n)=(0,-1),(2,17),(-1,-1)$'dir.
Başlık: Ynt: Diyafont Denklemler Çalışma Soruları (70 Tane)
Gönderen: Metin Can Aydemir - Ağustos 21, 2019, 03:12:48 ös
$75)$ $x=y$ ise $(x,y)=(0,0)$ bulunur. $x\neq y$ ise $x>y$ olmalıdır. $x-y=k$ olsun. $k>0$'dır. $$(y+k)^2+y^2=k^3$$ denklemini düzenlersek, $$(2y+k)^2=k^2(2k-1)$$ olur. Burada $2k-1$ tamkare olmalı. $2k-1=(2t+1)^2$ olsun, $k=2t^2+2t+1$ yazarsak, $$(2t^2+2t+1)^2(2t+1)^2=(2y+2t^2+2t+1)^2$$ Buradan $y=2t^3+2t^2+t$ veya $-2t^3-4t^2-3t-1$ bulunur. $y+k=x$'den,

Tüm çözümler, $(x,y)=(0,0),(2t^3+4t^2+3t+1,2t^3+2t^2+t),(-2t^3-2t^2-t,-2t^3-4t^2-3t-1)$'dir.
Başlık: Ynt: Diyafont Denklemler Çalışma Soruları (70 Tane)
Gönderen: Metin Can Aydemir - Ağustos 21, 2019, 03:30:55 ös
$76)$ $x=0$ ise $y=\pm 1$

$x>0$ ise $$(x+1)^4>x^4+x^3+x^2+x+1=y^4>x^4$$ olduğundan çözüm yoktur.

$x=-1$ için $y=\pm 1$ olur.

$x<-1$ ise $x=-a$ olsun. $a>1$'dir. $$a^4>y^4=a^4-a^3+a^2-a+1>(a-1)^4$$ olduğundan çözüm yoktur.

Tüm çözümler $(x,y)=(0,1),(0,-1),(-1,1),(-1,-1)$'dir.
Başlık: Ynt: Diyafont Denklemler Çalışma Soruları (70 Tane)
Gönderen: Metin Can Aydemir - Ağustos 23, 2019, 01:34:15 ös
$74)$ $x=0,1,2,3,4,5$ için denersek $(x,y)=(3,1),(5,3)$ çözümleri gelir. $x\geq 6$ için $3^y+5\equiv 0(mod~64)$ olur. Bu şartı sağlayan $y$ değeri için $y\equiv 11(mod~16)$'dir. $$2^x\equiv 3^{16k+11}+5\equiv 3^{11}+5\equiv 12 (mod~17)$$ $$\Rightarrow 2^x\equiv 12 (mod~17)$$ fakat hiçbir $x$ değeri için bu sağlanamaz. Dolayısıyla tüm çözümler $(x,y)=(3,1),(5,3)$'dir.
Başlık: Ynt: Diyafont Denklemler Çalışma Soruları (70 Tane)
Gönderen: Metin Can Aydemir - Ağustos 27, 2019, 04:10:17 ös
$42)$ $b=0$ için $k=-a^2$ olur fakat $k\geq 0$ olduğundan $a=k=0$ olabilir. Benzer şekilde $a=0$ için de aynı durum gelir. $a=b>0$ için, $$k=\dfrac{3a^2}{a^2-1}=3+\dfrac{3}{a^2-1}$$ olur. Buradan tek çözüm $a=2$, $k=4$'dür. Şimdi genelliği bozmadan $a>b$ olsun.

$i)$ $b=1$ ise $$k=\dfrac{a^2+a+1}{a-1}=a+2+\dfrac{3}{a-1}$$ olur. Buradan $a=2$ ve $a=4$ çözümleri gelir ve her ikisi için de $k=7$'dir.

$ii)$ $a>b>1$ ise $$k=\dfrac{a^2+b^2+ab}{ab-1}\geq \dfrac{3ab}{ab-1}>3$$ olduğundan $k\geq 4$ olur. Ayrıca $k=4$'ün sağladığını zaten biliyoruz. Dolayısıyla $k>4$ için incelememiz yeterlidir. Şimdi bu fonksiyonu ve $m>n>1$ şartını sağlayan tamsayı çiftlerinin arasından en küçük $m$ değerli olanı alalım. $$f(x)=x^2-(k-1)nx+(n^2+k)$$ olsun. $m$ bu fonksiyonun bir çözümüdür. $n^2\geq 4$ olduğundan $$f(n)=k-(k-3)n^2<0$$ olur. Eşit olamaz çünkü eşit olursa $$n^2(k-3)=k \Rightarrow k=n^2(k-3)\geq 4k-12\Rightarrow k=4$$ olur. Çelişki.

$f$ fonksiyonu bir paraboldür ve kolları yukarı doğru bakar. Dolayısıyla $f(n)$ negatifse $n$ iki kök arasında olmalıdır. Diğer kök $p$ olsun. $m>n>p$'dir. Vieta teoreminden $$m+p=(k-1)n$$ $$mp=n^2+k$$ buradan $p$ pozitif tamsayı bulunur. Dolayısıyla $(m,n,k)$ çözüm ise $(n,p,k)$ da bir çözümdür ve $m$'nin en küçük çözüm olmasıyla çelişir. Dolayısıyla $p=1$ olmalıdır. Buradan da daha önce bulduğumuz çözümler bulunur. Yani tüm çözümler $k=0,4,7$ değerlerini alabilir.
Başlık: Ynt: Diyafont Denklemler Çalışma Soruları (70 Tane)
Gönderen: Metin Can Aydemir - Ağustos 27, 2019, 04:40:18 ös
$31)$ $xy(x^2+y^2)=\dfrac{(x+y)^4-(x-y)^4}{8}=2z^2$ Buradan $$(x-y)^4+(4z)^2=(x+y)^4$$ olur. Fermat'ın son teoreminden bu denklemin çözümü için $(x-y)=0$ veya $z=0$ olmalıdır. $a^4+b^2=c^4$ denkleminin çözümünün olmadığının ispatı için "Proof of Fermat's Last Theorem for specific exponents" 'ı incelemeniz yeterlidir.

$i)$ $z=0$ için $xy(x^2+y^2)=0$ olur. Buradan $x=0$ veya $y=0$ bulunur. $(x,y,z)=(0,k,0),(k,0,0)$ çözümleri gelir.

$ii)$ $x=y$ ise $2z^2=x^2(x^2+x^2)=2x^4$ buradan da $z=\pm x^2$ bulunur. Buradan $(x,y,z)=(k,k,k^2),(k,k,-k^2)$ çözümleri bulunur.

Tüm çözümler $(x,y,z)=(0,k,0),(k,0,0),(k,k,k^2),(k,k,-k^2)$'dir.
Başlık: Ynt: Diyafont Denklemler Çalışma Soruları (70 Tane)
Gönderen: Metin Can Aydemir - Eylül 11, 2019, 05:00:49 öö
$77)$ Verilen ifadeyi $p$ modunda inceleyelim. $$p^3+mp+2m\equiv m^2+p+1~(\text{mod}~p)\Rightarrow (m-1)^2\equiv 0~(\text{mod}~ p)$$ olur. Şimdi $m=pk+1$ olsun. $m=1$ için çözüm gelmediğinden $k$ da pozitiftir. Yerine yazarsak $$p^3+(pk+1)(p+2)=(pk+1)^2+p+1\Rightarrow p=k^2-k$$ bulunur. Yani $p$ çifttir. $p=2$ dersek $k=2$ bulunur. $m=2\cdot 2+1=5$ olur.

Tek çözüm, $(m,p)=(5,2)$'dir.
Başlık: Ynt: Diyafont Denklemler Çalışma Soruları (70 Tane)
Gönderen: AtakanCİCEK - Ocak 01, 2020, 08:58:24 ös
$79)$
$$(a^2+1).(b^2+1).(c^2+1)=a^2b^2c^2+a^2b^2+a^2c^2+b^2c^2+a^2+b^2+c^2+1$$

$$(ab+ac+bc)^2=a^2b^2+a^2c^2+b^2c^2+2.abc.(a+b+c)$$

$$(a+b+c)^2=a^2+b^2+c^2+2.(ab+ac+bc)$$

$$a^2b^2c^2+1-2abc.(a+b+c)+(a+b+c)^2-2+1=(a+b+c-abc)^2=3^x-5^y$$ bulunur.

$a+b+c-abc=p$ dönüşümü yapalım.

$p^2=3^x-5^y$

$p^2\equiv \{0,1,4\}(mod5)\equiv 3^x(mod5)$  $3^x$  ifadesi $0,1,4$ kalanlarından birini vermesi için $x$ in çift olduğu açıktır.

$x=2t$ , $t\in N$  dönüşümü yapılabilir. İki kare farkından

$$(p-3^t).(p+3^t)=-5^y$$

Varsayalım ki  $p+3^t\le 0$ olsun.  O halde $p-3^t \ge 0$ olur. Buradan $p-3^t\ge p+3^t$ yani $3^t\le  0$ elde edilir. Bu bir çelişkidir.
O halde $p+3^t$ pozitiftir.

\begin{equation*}
\begin{cases}
p+3^t=5^g
\\p-3^t=-5^{y-g}
\end{cases}
\end{equation*}
, $g \in N$ dönüşümü yapılabilir. Düzenlersek  $$2.3^t=5^g+5^{y-g}$$ elde edilir. Denklemin solu $5$ ile bölünemeyeceğinden sağ kısmı da bölünememelidir. $g=0$ dır veya $y=g$ dir.

$1)$ $g=0$ için $2.3^t=5^y+1$ Lifting The Exponent Lemma'nın $2.$ formunu uygulayalım.

$3\mid 5+1 $ olduğundan $v_3(2.3^t)=v_3(5+1)+v_3(g)$ Buradan $g=k.3^{t-1}$ , $k \in N$ dönüşümü yapabiliriz.

Yerine koyarsak $2.3^t=5^{k.3^{t-1}}+1$ olduğundan $t>1$ için çelişki gelir.

    a) $t=0$ için $y=0$ elde edilir. Buradan $p=0$ elde edilir.Fakat $$(a^2+1).(b^2+1).(c^2+1)=p^2\ge 1$$ olduğu için mümkün değildir.

   b)   $t=1$  için $y=1$  elde edilir. $p=-2$ elde edilir.

2)   a) $y=g$  ve $t=0$ için $y=0$ bulunur. $1)$ in $a$  seçeneği ile aynıdır, mümkün değildir.

       b) $y=g$ ve $t=1$ için $y=1$ bulunur. $p=2$ elde edilir.

Elimizde $2$ adet denklem sistemi oluştu

\begin{equation*}
\begin{cases}
ab+ac+bc=1
\\
abc-(a+b+c)=2
\end{cases}
\end{equation*}

\begin{equation*}
\begin{cases}
ab+ac+bc=1
\\
abc-(a+b+c)=-2
\end{cases}
\end{equation*}

Denklemlerde $c=\dfrac{1-ab}{a+b}$ yazarsak

$\dfrac{ab-a^2b^2}{a+b}-\dfrac{(a+b)^2+1-ab}{a+b}=\dfrac{a^2b^2+a^2+b^2+1}{a+b}=\pm 2$

$(ab)^2+(a\pm 1)^2+(b\pm 1)^2=1$ elde edilir. Buradan karelerin birini $1$ e diğer ikisini $0$ a eşitlersek

$(1,1,0),(1,0,1),(0,1,1),(-1,-1,0),(-1,0,-1),(0,-1,-1)$ şeklinde $6$ çözüm elde edilir.
Başlık: Ynt: Diyafont Denklemler Çalışma Soruları ($138$ Tane)
Gönderen: Metin Can Aydemir - Ekim 20, 2020, 02:43:39 ös
$80)$ Verilen ifadeyi düzenlersek, $$n^2=(x^2+1)(y^2+1)+2(x-y)(1-xy)-4xy=(x+1)^2(y-1)^2$$ bulunur. Buradan $n=(x+1)(y-1)$ veya $-n=(x+1)(y-1)$ bulunur. $n$'yi bölen her $a$ tamsayısı için $(x,y)=\left (a-1,\dfrac{n}{a}+1\right ),\left (a-1,-\dfrac{n}{a}+1\right )$ çözümleri bulunabilir. Bu çözümler çakışık olamaz çünkü olsaydı $\dfrac{1}{a}=0$ olur fakat bu sağlanamaz. Her $a$ böleni için denklemin tam olarak $2$ çözümü bulunur. Dolayısıyla toplam çözüm sayısı, $n$'nin bölen sayısının $2$ katıdır.
Başlık: Ynt: Diyafont Denklemler Çalışma Soruları ($138$ Tane)
Gönderen: Metin Can Aydemir - Ekim 20, 2020, 02:52:20 ös
$81)$ $\dfrac{1}{x}+\dfrac{1}{y}=\dfrac{1}{n}$ olduğundan $\dfrac{1}{n}>\dfrac{1}{x}$ ve $\dfrac{1}{n}>\dfrac{1}{y}$ olmalıdır. Buradan $x>n$ ve $y>n$ bulunur. Şimdi denklemi düzenlersek, $$\dfrac{x+y}{xy}=\dfrac{1}{n}\Rightarrow xy=nx+ny\Rightarrow (x-n)(y-n)=n^2$$ bulunur. $x>n$ ve $y>n$ olduğundan $(x-n)$ ve $(y-n)$ ifadeleri pozitiftir. $n^2$'nin her pozitif $a$ böleni için $(x,y)=\left ( a+n,\dfrac{n^2}{a}+n \right )$ çözümü bulunur. Dolayısıyla toplam çözüm sayısı $n^2$'nin pozitif bölen sayısıdır.
Başlık: Ynt: Diyafont Denklemler Çalışma Soruları ($138$ Tane)
Gönderen: Metin Can Aydemir - Ekim 20, 2020, 03:42:14 ös
$82)$ Denklem simetrik olduğundan genelliği bozmadan $x\geq y$ diyebiliriz. Buradan, $$2x^2\geq x^2+y^2=(xy-9)^2\Rightarrow 0\geq x^2(y^2-2)-18xy+81$$ $$\Rightarrow 0\geq x^2(y^2-2)-18xy+81\geq x^2(y^2-2)-18x^2+81$$ bulunur. Eğer $y\geq 5$ ise $$0\geq x^2(y^2-20)+81\geq 5x^2+81>0$$ bulunur ve bu bir çelişkidir. Dolayısıyla $y\leq 4$ olmalıdır.

$i)$ $y=0$ ise $81=x^2$ elde edilir, buradan $(x,y)=(9,0)$ çözümü gelir.

$ii)$ $y=1$ ise $(x-9)^2=x^2+1$ ve buradan $x=\dfrac{40}{9}$ bulunur. Tamsayı çözümü yoktur.

$iii)$ $y=2$ ise $(2x-9)^2=x^2+4$ ve buradan $3x^2-36x+77=0$ bulunur fakat tamsayı çözümü gelmez.

$iii)$ $y=3$ ise $(3x-9)^2=x^2+9$ ve buradan $4x^2-27x+36=0$ bulunur. Buradan da tamsayı çözümü gelmez.

$iv)$ $y=4$ ise $(4x-9)^2=x^2+16$ ve buradan $15x^2-72x+65=0$ bulunur. Tamsayı çözümü yoktur.

Dolayısıyla tüm çözümler $(x,y)=(9,0),(0,9)$ bulunur.
Başlık: Ynt: Diyafont Denklemler Çalışma Soruları ($138$ Tane)
Gönderen: Metin Can Aydemir - Ekim 20, 2020, 08:32:10 ös
$83)$ İfadeyi açalım, $x^2y+xy^2-x^2-y^2=xy(x+y)-(x^2+y^2)=1$ olur. $x^2+y^2=(x+y)^2-2xy$ olduğundan $$xy(x+y)-(x^2+y^2)=xy(x+y)-(x+y)^2+2xy=1\Rightarrow xy(x+y+2)=(x+y)^2+1$$ bulunur. $xy=a$ ve $x+y=b$ için $$a(b+2)=b^2+1\Rightarrow a=\dfrac{b^2+1}{b+2}=b-2+\dfrac{5}{b+2}$$ bulunur. $b+2=\pm 1,\pm 5$ olabilir. Buradan olası $b$ değerleri $\{-7,-3,-1,3\}$ bulunur. Bunlara karşılık gelen $a$ değerlerini bulabiliriz. Olası $(a,b)$ ikilileri $(-10,-7)$, $(-10,-3)$, $(2,-1)$, $(2,3)$ bulunur. $(a,b)=(-10,-3)$ ikilisi için $(x,y)=(-5,2),(2,-5)$ çözümleri bulunur. $(a,b)=(2,3)$ için $(x,y)=(2,1),(1,2)$ çözümleri bulunur. $(a,b)=(-10,-7)$ ve $(a,b)=(2,-1)$ ikilileri için çözüm gelmez. Tüm çözümler, $\boxed{(x,y)=(-5,2),(2,-5),(2,1),(1,2)}$'dir.
Başlık: Ynt: Diyafont Denklemler Çalışma Soruları ($138$ Tane)
Gönderen: Metin Can Aydemir - Ekim 24, 2020, 11:04:11 ös
$84)$ Öncelikle $x$'nin tek sayı olması gerektiğini gösterelim. Aksini varsayalım, $m\in \mathbb{Z}$ olmak üzere, $x=2m$ olsun. Ana denklem, $16m^4+4=py^4$ haline dönüşür. Sol taraf $4$ ile bölündüğünden sağ taraf da bölünmelidir, $p=2$ olsa bile $y$ çift olmak zorundadır. $y=2n$ için $16m^4+4=16pn^4$ olur. Sağ taraf $16$ ile bölünüp, sol taraf bölünmediği için çelişki olur. Dolayısıyla $x$ tek sayıdır. Şimdi ifadeyi çarpanlarına ayıralım, $$x^4+4=x^4+4x^2+4-4x^2=(x^2+2)^2-(2x)^2=(x^2+2x+2)(x^2-2x+2)=py^4$$ bulunur. $x$ tek sayı olduğundan $x^2+2x+2$ ve $x^2-2x+2$ ifadeleri de tek sayıdır. Eğer bu iki ifade aralarında asal değilse, ikisini de bölen bir $q$ tek asal sayısı vardır. $$q\mid x^2+2x+2 \text{         ve        } q\mid x^2-2x+2 \Rightarrow q\mid (x^2+2x+2)-(x^2-2x+2)\Rightarrow q\mid 4x \Rightarrow q\mid x$$ $$q|x \text{         ve        }  q|x^2-2x+2 \Rightarrow q\mid (x^2-2x+2)-x(x-2)\Rightarrow q\mid 2$$ bulunur fakat $q$ tek asal sayı olduğundan bu durum da çelişki oluşturur. Dolayısıyla $x^2+2x+2$ ve $x^2-2x+2$ ifadeleri aralarında asaldır. Bu iki ifade aralarında asal olduğundan ve çarpımları $py^4$ olduğundan $2$ durum söz konusudur.

$i)$ $x^2+2x+2=pa^4$ ve $x^2-2x+2=b^4$ ise $(x-1)^2+1=b^4$ ifadesini düzenleyebiliriz, $$b^4-(x-1)^2=1\Rightarrow (b^2-x+1)(b^2+x-1)=1$$ olur ve buradan gelen tek çözüm $x=1$ durumudur. $x=1$ için ana denklemde $py^4=5$ bulunur, buradan $p=5$ elde edilir.

$ii)$ $x^2+2x+2=a^4$ ve $x^2-2x+2=pb^4$ ise $(x+1)^2+1=a^4$ ifadesini düzenlersek $$(a^2+x+1)(a^2-x-1)=1$$ olur buradan sadece $x=-1$ çözümü gelir. $x=-1$ için ana denklemden yine $py^4=5$ elde edilir. Buradan da $p=5$ gelir.

Tüm durumlarda $p=5$ elde edildiğinden çözüm olmasını sağlayan tek asal sayı $\boxed{p=5}$ 'dir.
Başlık: Ynt: Diyafont Denklemler Çalışma Soruları ($138$ Tane)
Gönderen: Metin Can Aydemir - Ekim 25, 2020, 02:40:24 öö
$85)$ Verilen eşitlikleri Aritmetik-Harmonik ortalama eşitsizliğinde kullanırsak, ($k_1,k_2,\dots, k_n$ terimleri pozitif olduğundan bunu uygulayabiliriz.) $$\dfrac{k_1+k_2+\cdots+k_n}{n}\geq \dfrac{n}{\dfrac{1}{k_1}+\dfrac{1}{k_2}+\cdots+\dfrac{1}{k_n}}\Rightarrow \dfrac{5n-4}{n}\geq n\Rightarrow 0\geq n^2-5n+4$$ $n^2-5n+4$ bir parabol olduğundan ve başkatsayısı pozitif olduğundan negatif değer alabilmesi için $n$ sayısını, $x^2-5x+4=0$ denkleminin köklerinin arasında olması gerekir. Bu denklemin çözümleri $x=1$ ve $x=4$ olduğundan $n\in [1,4]$ olmalıdır. $n=1$ ve $n=4$ için yukarıdaki eşitsizlikte eşitlik durumu sağlanacağından terimler birbirine eşit olmalıdır.

$i)$ $n=1$ için $k_1=1$ olur. $(n,k_1)=(1,1)$ çözümü gelir.

$ii)$ $n=4$ için terimlerin eşit olması gerektiğini belirtmiştik. $k_1=k_2=k_3=k_4$ için $k_1+k_2+k_3+k_4=16$ olduğundan $(n,k_1,k_2,k_3,k_4)=(4,4,4,4,4)$ çözümü gelir.

$iii)$ $n=2$ ise $k_1+k_2=6$ ve $\dfrac{1}{k_1}+\dfrac{1}{k_2}=\dfrac{k_1+k_2}{k_1k_2}=\dfrac{6}{k_1k_2}=1$ olduğundan $k_1k_2=6$ elde edilir. Bu iki denklemin tamsayı çözümü yoktur.

$iv)$ $n=3$ ise $k_1+k_2+k_3=11$ ve $\dfrac{1}{k_1}+\dfrac{1}{k_2}+\dfrac{1}{k_3}=1$ denklemleri elde edilir. $$\dfrac{1}{k_1}+\dfrac{1}{k_2}+\dfrac{1}{k_3}=\dfrac{k_1k_2+k_1k_3+k_2k_3}{k_1k_2k_3}=1\Rightarrow k_1k_2+k_1k_3+k_2k_3=k_1k_2k_3$$ olur. $k_3=11-k_1-k_2$ dersek $$k_1k_2+(11-k_1-k_2)(k_1+k_2)=k_1k_2(11-k_1-k_2)$$ eşitliği elde edilir. $k_1k_2=a$ ve $k_1+k_2=b$ için $a+(11-b)b=a(11-b)$ elde edilir. $a$'yı yalnız bırakırsak, $$a+(11-b)b=a(11-b)\Rightarrow a=\dfrac{b^2-11b}{b-10}=b-1-\dfrac{10}{b-10}$$ bulunur. Buradan olası pozitif $b$ değerlerini bulabiliriz, $b=5,8,9,11,12,15,20$ bulunur. Bu değerler için $a$ değerlerini de hesaplayabiliriz. $(a,b)=(6,5),(12,8),(18,9),(0,11),(6,12),(12,15),(18,20)$ ikilileri bulunur fakat bunlardan sadece $(a,b)=(6,5),(12,8),(18,9)$ için $k_1$ ve $k_2$ pozitif tamsayıları bulunabilir. Bu $(a,b)$ ikilileri için $(k_1,k_2,k_3)=(2,3,6)$ ve permütasyonları bulunur.

Tüm çözümler, $(n,k_1,k_2,\dots,k_n)=(1,1),(3,2,3,6),(3,2,6,3),(3,3,2,6),(3,3,6,2),(3,6,2,3),(3,6,3,2),(4,4,4,4,4)$ bulunur.
Başlık: Ynt: Diyafont Denklemler Çalışma Soruları ($138$ Tane)
Gönderen: Metin Can Aydemir - Ekim 25, 2020, 03:10:42 öö
$86)$ Öncelikle $x,y,z$ sayılarının farklı olduğu duruma bakalım, genelliği bozmadan $x<y<z$ olsun.

$i)$ $x=1$ için $\dfrac{y+1}{y}\dfrac{z+1}{z}=1$ olur fakat $\dfrac{y+1}{y}>1$ ve $\dfrac{z+1}{z}>1$ olduğundan çözüm yoktur.

$ii)$ $x=2$ ise $\dfrac{y+1}{y}\dfrac{z+1}{z}=\dfrac{4}{3}$ olur. $\dfrac{y+1}{y}>\dfrac{z+1}{z}$ olduğundan $\left (\dfrac{y+1}{y} \right )^2>\dfrac{4}{3}$ olur. Buradan, $y\leq 6$ bulunur. Bu değerler denenirse $(x,y,z)=(2,6,7),(2,5,9),(2,4,15)$ çözümleri bulunur.

$iii)$ $x=3$ ise $\dfrac{y+1}{y}\dfrac{z+1}{z}=\dfrac{3}{2}$ olur. Aynı şekilde $\dfrac{y+1}{y}>\dfrac{z+1}{z}$ dersek $\left (\dfrac{y+1}{y} \right )^2>\dfrac{3}{2}$ olur ve buradan $y\leq 4$ bulunur. $y>x$ olduğundan $y=4$ olmalıdır. yerine yazılırsa $(x,y,z)=(3,4,5)$ çözümü bulunur.

$iv)$ $x>3$ ise $y>4$ ve $z>5$ olur. $\left (1+\dfrac{1}{x}\right )\left (1+\dfrac{1}{y}\right )\left (1+\dfrac{1}{z}\right )<\left (1+\dfrac{1}{3}\right )\left (1+\dfrac{1}{4}\right )\left (1+\dfrac{1}{5}\right )=2$ olur. Çelişki.

Eğer $x,y,z$'den en az ikisi eşitse, genelliği bozmadan $y=z$ olsun. $\left (\dfrac{x+1}{x}\right )\left( \dfrac{y+1}{y}\right )^2=2$ olur. $EBOB(x,x+1)=EBOB(y,y+1)=1$ olduğundan $\dfrac{x+1}{y^2}$ ve $\dfrac{(y+1)^2}{x}$ ifadeleri tamsayı olmalıdır. Bu iki ifadenin çarpımları $2$ olduğundan iki durum söz konusudur.

$a)$ $\dfrac{x+1}{y^2}=1$ ve $\dfrac{(y+1)^2}{x}=2$ ise $y^2=x+1$ ve $y^2+2y+1=2x$ bulunur. İlk denklemdeki eşitliği ikincide yazarsak $x=2y+2$ bulunur. Bunu tekrar ilk denkleme yazarsak $2y+3=y^2$ elde edilir. Bu ikinci derecen denklemi çözersek $y=3$ ve $y=-1$ bulunur. Pozitif tamsayılarda çözdüğümüz için $y=3$ olmalıdır. Buradan $(x,y,z)=(8,3,3)$ çözümü gelir.

$b)$ $\dfrac{x+1}{y^2}=2$ ve $\dfrac{(y+1)^2}{x}=1$ ise $2y^2-1=x$ ve $y^2+2y+1=x$ bulunur. Bu iki denklemden $2y^2-1=y^2+2y+1$ bulunur fakat buradan tamsayı çözüm gelmez.

Tüm çözümler, $\boxed{(x,y,z)=(2,6,7),(2,5,9),(2,4,15),(3,4,5),(8,3,3)}$ ve permütasyonlarıdır.


------------------------------------------------------------------------------------------------------------------------------------
Not: Buradaki (http://geomania.org/forum/index.php?topic=6805) gönderide $x,y,z\in \mathbb{Z}^{+}$ için $\left (1+\dfrac{1}{x}\right )\left (1+\dfrac{1}{y}\right )\left (1+\dfrac{1}{z}\right )$ ifadesinin tamsayı olduğu tüm $(x,y,z)$ üçlüleri bulunmuştur.
Başlık: Ynt: Diyafont Denklemler Çalışma Soruları ($138$ Tane)
Gönderen: Metin Can Aydemir - Ekim 26, 2020, 12:26:08 ös
$87)$ Genelliği bozmadan $x\geq y\geq z$ olsun. Verilen eşitliği sağlayan $(x,y,z)$ pozitif tamsayı üçlüleri arasından toplamı en küçük olan $(x,y,z)$ alalım. Şimdi verilen eşitliği ikinci dereceden olarak yazalım, $$x^2-x(nyz-2y-2z)+(y+z)^2=0$$ Bu denklemin $x$ dışındaki kökü $a$ olsun. Vietta'dan $x+a=nyz-2y-2z$ ve $xa=(y+z)^2$ bulunur. Bu iki eşitliğinden anlaşılır ki $a$ pozitif bir tamsayıdır. $a\geq x$ olmalıdır çünkü aksi takdirde $(a,y,z)$ bir çözüm olur ve $a+y+z<x+y+z$ olduğundan kabulumuz ile çelişir. Dolayısıyla $a\geq x$'dir. $$(y+z)^2=ax\geq x^2\Rightarrow y+z\geq x$$ olur. Ayrıca $2x\geq y+z$ olduğundan $$(y+z)^2=ax\geq a\left (\dfrac{y+z}{2}\right )\Rightarrow 2(y+z)\geq a$$ olur. Buradan, $$3(y+z)\geq x+a\Rightarrow 3(y+z)\geq nyz-2y-2z\Rightarrow 0\geq nyz-5y-5z\Rightarrow 25\geq n^2yz-5ny-5nz+25=(ny-5)(nz-5)$$ olur. Eğer $n>10$ ise $ny-5> 10y-5>5$ ve benzer şekilde $nz-5>5$ olacağından $(ny-5)(nz-5)>25$ olur ve bu bir çelişkidir. Dolayısıyla $n\leq 10$ olmalıdır. $n=1,2,3,4,5,6,8,9$ için çözüm vardır (çözümün sonunda belirteceğim) fakat $n=10$ ve $n=7$ için çözüm yoktur.

$i)$ $n=10$ ise $25\geq (10y-5)(10z-5)\geq (10-5)(10-5)=25$ olduğundan $y=z=1$ olmalıdır. Ana denklem $(x+2)^2=10x$ haline gelir. $x^2-6x+4=0$ denkleminin tamsayı çözümü yoktur.

$ii)$ $n=7$ ise $y,z\geq 2$ ise $25\geq (7y-5)(7z-5)\geq (7\cdot 2-5)^2=81$ olacağından çelişki olur. Dolayısıyla $z=1$ olmalıdır. $25\geq 2(7y-5)$ olur. $y$'yi yalnız bırakırsak $\dfrac{5}{2}\geq y$ olur ve buradan $y=1$ veya $y=2$ olabileceği görülür.

$iia)$ $y=1$ ise ana denklemde $(x+2)^2=7x$ denklemi elde edilir fakat bu denklemin tamsayı çözümü yoktur.

$iib)$ $y=2$ ise ana denklemde $(x+3)^2=14x$ denklemi elde edilir fakat bu denklemin de tamsayı çözümü yoktur. Dolayısıyla $n=7$ için denklemin çözümü yoktur.

Soruda bizden çözüm olan $n$ değerleri istendiğinden $n=1,2,3,4,5,6,8,9$ için çözüme örnek vermemiz yeterlidir. $(n,x,y,z)=(1,9,9,9)$, $(2,8,4,4)$, $(3,3,3,3)$, $(4,4,2,2)$, $(5,5,4,1)$, $(6,3,2,1)$, $(8,2,1,1)$, $(9,1,1,1)$ örnek çözümleri vardır. Dolayısıyla çözümü mümkün kılan $n$ değerleri $n=1,2,3,4,5,6,8,9$ bulunur.
Başlık: Ynt: Diyafont Denklemler Çalışma Soruları ($138$ Tane)
Gönderen: Metin Can Aydemir - Ekim 26, 2020, 12:36:04 ös
$88)$ $x^2+84x+2008$ ifadesini düzenlersek $$y^2=x^2+84x+2008=(x+42)^2+244\Rightarrow (y-x-42)(y+x+42)=244=2^2\cdot 61$$ olur. $(y-x-42)+(y+x+42)=2y$ olduğundan ya iki çarpan da tektir ya da çifttir. Çarpımları çift olduğundan iki terim de çift olmalıdır. Ayrıca $y+x+42\geq y-x-42$ olduğundan $y+x+42=122$ ve $y-x-42=2$ olmalıdır. Bu iki ifadeyi toplarsak $y=62$ ve bunu denkleme koyarsak $x=18$ bulunur. $\boxed{x+y=80}$ bulunur.
Başlık: Ynt: Diyafont Denklemler Çalışma Soruları ($138$ Tane)
Gönderen: Metin Can Aydemir - Ekim 26, 2020, 12:45:06 ös
$89)$ Öncelikle $A=0$ durumuna bakalım. Denklem $(10B+C)(B+C)=2005$ durumuna gelir. $(10B+C)(B+C)\leq (10\cdot 9+9)(9+9)=1782$ olduğundan $(10B+C)(B+C)=2005$ denkleminin çözümü yoktur.

$A,B,C$ rakam olduğundan $A\neq 0$ ise $\overline{ABC}$ bir üç basamaklı sayı belirtir. $\overline{ABC}(A+B+C)=2005=5\cdot 401$ olduğundan $\overline{ABC}$, $2005$ sayısının bir bölenidir ve tüm bölenler $1,5,401,2005$ olduğundan sadece $\overline{ABC}=401$ olabilir. Bu durumda $A+B+C=5$ sağladığından $(A,B,C)=(4,0,1)$ çözümü bulunur. $\boxed{A=4}$'dür.
Başlık: Ynt: Diyafont Denklemler Çalışma Soruları ($138$ Tane)
Gönderen: Metin Can Aydemir - Ekim 26, 2020, 12:51:11 ös
$90)$ $x\in\mathbb{N}$ olsun ve $x^2=71p+1$ sağlasın. $x^2-1=(x-1)(x+1)=71p$ bulunur. $x+1>x-1$ olduğundan ve $71$ de bir asal sayı olduğundan $((x-1),(x+1))=(1,71p),(71,p),(p,71)$ olabilir. $x-1=1$ ise $x+1=3=71p$ bulunur fakat çözüm yoktur.

Eğer $x-1=71$ ise $x+1=p=73$ bulunur. Gerçekten de $p=73$ için $71\cdot 73+1=72^2$ sağlar.

Eğer $x+1=71$ ise $x-1=p=69$ bulunur fakat $69$ asal sayı değidir. Dolayısıyla şartı sağlayan tek asal sayı $\boxed{p=73}$'dür.
Başlık: Ynt: Diyafont Denklemler Çalışma Soruları ($138$ Tane)
Gönderen: Metin Can Aydemir - Ekim 26, 2020, 06:03:35 ös
$91)$ Verilen eşitlikte sağ taraf $7$ ile bölündüğünden sol taraf da bölünmelidir. Dolayısıyla $p,q,r$'den en az biri $7$ olmalıdır. Genelliği bozmadan $r=7$ olsun. Denklemi düzenlersek, $$pq=p+q+7\Rightarrow pq-p-q+1=8\Rightarrow (p-1)(q-1)=8$$ bulunur. Bu eşitliği $8$'in çarpanlarına ayırarak çözebiliriz. Genelliği bozmadan $p\geq q$ olsun dersek $(p,q)=(9,2),(5,3)$ bulunur. $9$ asal sayı olmadığından $(p,q)=(5,3)$ olmalıdır. Buradan $\boxed{p+q+r=3+5+7=15}$ bulunur.
Başlık: Ynt: Diyafont Denklemler Çalışma Soruları ($138$ Tane)
Gönderen: Metin Can Aydemir - Ekim 26, 2020, 06:16:17 ös
$92)$ $n-76=a^3$ ve $n+76=b^3$ yazalım. $b^3-a^3=152$ bulunur. $$b^3-a^3=(b-a)(a^2+b^2+ab)=152=2^3\cdot 19$$ bulunur. $b^3-a^3=152$ olduğundan $a$ ve $b$'nin pariteleri aynı olmalıdır. Dolayısıyla $b-a$ çift olacaktır. Ayrıca $a^2+b^2+ab=(b-a)^2+3ab$ olduğundan $a^2+b^2+ab>(b-a)^2$ olacaktır. Bu şartlar altında olası durumlar şunlardır; $(b-a, a^2+b^2+ab)=(2,76),(4,38)$ bulunur.

$i)$ $b-a=2$ ve $a^2+b^2+ab=76$ ise $b=a+2$ yazarsak $$a^2+(a+2)^2+a(a+2)=76\Rightarrow a^2+2a-24=(a+6)(a-4)=0$$ bulunur. $a$ pozitif olduğundan $a=4$ ve $b=a+2=6$ olacaktır. Bu değerler için $n=140$ bulunur.

$ii)$ $b-a=4$ ve $a^2+b^2+ab=38$ ise $b=a+4$ için $$a^2+(a+4)^2+a(a+4)=38\Rightarrow 3a^2+12a-22=0$$ bulunur fakat buradan tamsayı çözümü gelmez.

Şartı sağlayan tek sayı $\boxed{n=140}$'dır.
Başlık: Ynt: Diyafont Denklemler Çalışma Soruları ($138$ Tane)
Gönderen: Metin Can Aydemir - Ekim 26, 2020, 11:52:57 ös
$93)$ Herhangi bir $a>0$ ve $a\neq 1$ reel sayısı için $\log_{a}{x}=0$ ise $x=1$'dir ve $\log_{a}{y}=1$ ise $y=a$'dır. Dolayısıyla $$\log_{2}{\left (\log_{2^a}{\left (\log_{2^b}{2^{1000}}\right )}\right )}=0\Rightarrow \log_{2^a}{\left (\log_{2^b}{2^{1000}}\right )}=1\Rightarrow \log_{2^b}{2^{1000}}=2^a$$ bulunur. $\log_{a^x}{b^y}=\dfrac{y}{x}\log_{a}{b}$ olduğundan $\log_{2^b}{2^{1000}}=\dfrac{1000}{b}\log_{2}{2}=\dfrac{1000}{b}=2^a$ bulunur. $1000=2^3\cdot 5^3$ olduğundan ve $2^a$ sayısının da $1000$'in bir böleni olmasından dolayı $a=1,2,3$ olabilir. Bu değerler için $b$ değerlerini de hesaplayabiliriz.

Tüm çözümler $(a,b)=(1,500),(2,250),(3,125)$'dir. Bu çözümler için $a+b$ ifadesi $501,252,128$ değerlerini alır. Bu değerlerin toplamı $501+252+128=\boxed{881}$ bulunur.
Başlık: Ynt: Diyafont Denklemler Çalışma Soruları ($138$ Tane)
Gönderen: Metin Can Aydemir - Ekim 27, 2020, 12:06:01 öö
$94)$ Öncelikle şu görülebilir ki $5>y$ ise denklemin sol tarafı negatif olmaktadır. Dolayısıyla negatif $y$ değerleri için çözüm yoktur. Ayrıca $(x,y)$ bir çözümse $(-x,y)$ de bir çözüm olacağından genelliği bozmadan $x$'i pozitif alabiliriz. ($x=0$ için denklemin çözümü olmadığı barizdir.) $$4^y-615=x^2\Rightarrow 4^y-x^2=(2^y-x)(2^y+x)=615=3\cdot 5\cdot 41$$ elde edilir. $2^y-x+2^y+x=2^{y+1}$ olduğundan $615$'in çarpanları arasından toplamı $2$'nin kuvveti olan ve çarpımları $615$ olan ikilileri bulmalıyız. Çarpımları $615$ olan tüm ikililer $(1,615)$, $(3,205)$, $(5,123)$, $(15,41)$'dir. Bunlardan toplamı $2$'nin kuvveti olan sadece $(5,123)$ ikilisidir. Dolayısıyla $2^y-x=5$ ve $2^y+x=123$ olmalıdır, bu iki denklemden $(x,y)=(59,6)$ bulunur. Başta da belirttiğimiz gibi $(x,y)=(59,6)$ çözümse $(x,y)=(-59,6)$ da bir çözümdür. Tüm çözümler, $\boxed{(x,y)=(59,6),(-59,6)}$ bulunur.
Başlık: Ynt: Diyafont Denklemler Çalışma Soruları ($138$ Tane)
Gönderen: Metin Can Aydemir - Ekim 28, 2020, 11:25:21 ös
$95)$ Verilen ifadeyi düzenleyelim, $$2(x^2+y^2)+x+y=5xy\Rightarrow 2(x+y)^2+x+y=9xy$$ $x+y=a$ ve $xy=b$ için $2a^2+a=9b$ bulunur. Ayrıca, $$(x+y)^2-4xy=(x-y)^2\Rightarrow a^2-4b=(x-y)^2\Rightarrow \sqrt{a^2-4b}\in \mathbb{Z}$$ $$\sqrt{a^2-4b}\in \mathbb{Z}\Rightarrow \sqrt{9a^2-36b}\in \mathbb{Z}\Rightarrow \sqrt{9a^2-4(2a^2+a)}\in \mathbb{Z}\Rightarrow \sqrt{a^2-4a}\in \mathbb{Z}$$ bulunur. Öyleyse $a^2-4a=t^2$ olacak şekilde bir $t$ tamsayısı vardır. $$a^2-4a+4=t^2+4\Rightarrow (a-2)^2-t^2=4\Rightarrow (a-2-t)(a-2+t)=4$$ bulunur. $(a-2-t)+(a-2+t)=2a-4$ olduğundan $(a-2-t)$ ve $(a-2+t)$ ifadeleri aynı paritedelerdir ve ayrıca çarpımları çift olduğundan ikisi de çift olmalıdır. Dolayısıyla ihtimaller, $(a-2-t,a-2+t)=(-2,-2)$ ve $(a-2-t,a-2+t)=(2,2)$'dir. Buradan $a=0$ ve $a=4$ çözümleri bulunur. Bu değerler için $2a^2+a=9b$ olduğundan $b$ değerlerini de bulabiliriz. $(a,b)=(0,0)$ ve $(a,b)=(4,4)$ bulunur. $x+y=a$ ve $xy=b$ olduğundan $\boxed{(x,y)=(0,0),(2,2)}$ bulunur. Tüm çözümler bunlardır.
Başlık: Ynt: Diyafont Denklemler Çalışma Soruları ($138$ Tane)
Gönderen: Metin Can Aydemir - Mayıs 08, 2022, 01:34:54 öö
$41)$ Öncelikle $p=q$ için çözüm var mı araştıralım. Denklem $p^{m+n}=4p^2+1$ olur. Sol taraf $p$ ile bölündüğünden $4p^2+1$ ve $1$ sayısı da $p$ ile bölünür. Yani $p=1$ olmalıdır fakat denklemi sağlamaz. Şimdi genelliği bozmadan $p>q$ kabul edelim.

Eğer $m\geq 3$ ise $p^mq^n\geq p^3$ ve $(p+q)^2+1\leq (p+p)^2+1=4p^2+1$ olduğundan $$p^3\leq 4p^2+1\Rightarrow 4\geq p$$ olacaktır. $p=1$ olamaz çünkü $p>q\geq 1$'dir.

$p=2$ ise $q=1$ olmalıdır fakat $2^m=(2+1)^2+1$ denkleminin çözümü yoktur.

$p=3$ ise $q=1$ veya $q=2$ olmalıdır. İkisi için de denklemin çözümü yoktur.

$p=4$ ise $q=1$ veya $q=2$ veya $q=3$ olmalıdır. Üçü için de denklemin çözümü yoktur.

i) $m=2$ ise $q=1$ için denklem sağlanmaz çünkü $p^2=(p+1)^2+1$ denkleminin çözümü yoktur. Yani $p>q\geq 2$ diyebiliriz. Denklemi açıp düzenlersek $p^2(q^n-1)-2pq-(q^2+1)=0$ olur. Bu ikinci dereceden bir denklemdir ve çözümünü yapabiliriz. Denklemin köklerinin çarpımı, Vieta teoreminden, $-\dfrac{q^2+1}{q^n-1}$ olduğundan köklerden biri pozitif, diğeri negatiftir. Bu yüzden $$p=\dfrac{q+\sqrt{q^{n+2}-q^n+1}}{q^n-1}$$ olacaktır. İfade $1$'den büyük olduğundan $$q+\sqrt{q^{n+2}-q^n+1}>q^n-1\implies q^{n+2}-q^n+1\geq (q^n-q)^2\implies q^{n+2}+2q^{n+1}+1\geq q^{2n}+q^n+q^2$$ $$3q^{n+2}+q^2\geq q^{n+2}+2q^{n+1}+1\geq q^{2n}+q^n+q^2\implies 3q^{n+2}\geq q^{2n}+q^n\implies 3q^2\geq q^n+1$$ Buradan $n\geq 4$ için çelişki gelir. $n=3$ için ise $q\leq 3$ olmalıdır aksi taktirde $3q^2< q^3+1$ olur. $n=3$ için de $q=2$ ve $q=3$'ten çözüm gelmez.

$n=2$ ise $(pq)^2=(p+q)^2+1$ olur fakat aralarında $1$ fark olan tek olası tamkareler $1$ ve $0$'dır. Bu bir çelişkidir.

$n=1$ ise denklem $p^2q=(p+q)^2+1$ şekline gelir. Bu denklemi de düzenlersek $q^2+q(2p-p^2)+p^2+1=0$ olur. $$x^2+x(2p-p^2)+p^2+1=0$$ denkleminin kökleri $q_1>q_2$ olsun (eşitlik durumunda $q^2=p^2+1$ olur). Vieta teoreminden $$q_1+q_2=p^2-2p$$ $$q_1q_2=p^2+1$$ Yani $p^2q=(p+q)^2+1$ denkleminin istediğimiz koşullarda çözümü varsa hem $q_1$ hem de $q_2$ pozitif tamsayıdır. En az biri $p$'den küçük olduğundan ve çarpımları $p^2$'den büyük olduğundan $q_1>p>q_2$ olacaktır ($q_1,q_2\neq p$ olmalıdır çünkü $q_1q_2=p^2+1$ sayısı $p$ ile bölünmez). Burada $(p,q_2)$ denklemimizin aradığımız çözümüdür. Vieta'dan elde ettiğimiz eşitlikleri birbirinden çıkartırsak $$(q_1-1)(q_2-1)=2p+2\geq (p-1)(q_2-1)$$ elde edilir. $q_2\geq 4$ ise $2p+2\geq 3p-3$ elde edilir yani $5\geq p$ olur. Tek olası durum $(p,q_2)=(5,4)$'dür ama denkleme çözüm değildir. O halde $q_2\leq 3$ olmalıdır. Buradan $q_2=2$ için $p=5$ çözümü gelir.

ii) Eğer $m=1$ ise $q$ çift olamaz çünkü aksi taktirde $(p+q)$ tek olacaktır ama $(p+q)^2+1\equiv 2\pmod{4}$ olduğundan $n=1$ olacaktır. Buradan da çözüm gelmez. Dolayısıyla $n\geq 2$ ve $q\geq 3$ diyebiliriz. Denklemi düzenlersek $$p^2-p(q^n-2q)+q^2+1=0$$ olur. $x^2-x(q^n-2q)+q^2+1=0$ denkleminin kökleri  $p_1>p_2$ olsun ($p_1=p_2$ durumunda $p_1^2=q^2+1$ olacaktır). Denklemin aradığımız şartlarda bir çözümü varsa $p_1>q>p_2\geq 2$ olmalıdır çünkü $p_1>p_2>q$ durumunda $p_1p_2>q^2+1$ olacaktır ($p_1=q$ veya $p_2=q$ veya $p_2=1$ durumunda $q$ bulunamadığı görülebilir). Buradan $$p_1+q>p_1+p_2=q^n-2q\implies p_1>q^n-3q\implies q^2+1=p_1p_2>p_1>q^n-3q$$ $n>2$ için son eşitsizlik sağlanmaz. O halde $n=2$ olmalıdır. Bu durumda Vieta teoreminden $(p_1-1)(p_2-1)=2q+2$ elde edilir. $p_2=2$ ise $p_1=2q+3$ olur ve denklemde yerine yazarsak $(p,q)=(13,5)$ çözümü gelir. $p_2>2$ ise $$ 2q+2\geq2(p_1-1)\implies q+2\geq p_1$$ elde edilir. Yani $p_1=q+1$ veya $q+2$ olacaktır. İki durumda da denklemde yerine yazınca çözüm gelmez. O halde buradan gelen tek çözüm $(p,q)=(13,5)$'dir.

Tüm çözümler, $(p,q,m,n)=(13,5,1,2),(5,13,2,1),(5,2,2,1),(2,5,1,2)$'dir.
Başlık: Ynt: Diyafont Denklemler Çalışma Soruları ($138$ Tane)
Gönderen: Metin Can Aydemir - Haziran 09, 2022, 08:37:43 ös
96) $x^3=1$ denkleminin karmaşık sayılarda üç tane çözümü vardır. Bunlar, $\omega=e^{\frac{2\pi i}{3}}$ olmak üzere $1$, $\omega$ ve $\omega^2$'dir ve $\omega^2+\omega+1=0$ sağlanır. Biz $x^3+y^3=z^3$ denkleminin tamsayılardan daha büyük bir küme olan $\mathbb{Z}[\omega]=\{a+b\omega\mid a,b\in \mathbb{Z}\}$'de çözümümlerine bakacağız. Buradaki (https://geomania.org/forum/index.php?topic=7528) gönderide $\mathbb{Z}[\omega]$'nin birim veya $0$ olmayan her elemanının tek şekilde çarpanlarına ayrılabildiğini göstermiştik ($\mathbb{Z}[\omega]$'de birim elemanlar $1,-1,\omega,-\omega,1+\omega,-1-\omega$'dır).

$x^3+y^3=z^3$ denkleminde eğer $\text{EBOB}(x,y,z)=d$ ise herhangi bir $(x,y,z)$ çözümü için $\left(\dfrac{x}{d},\dfrac{y}{d},\dfrac{z}{d}\right)$ de çözüm olacağından genelliği bozmadan $\text{EBOB}(x,y,z)=1$ kabul edebiliriz. Tüm çözümleri bulmak için elde edeceğimiz ilkel çözümleri $d$ ile çarpmamız yeterli olacaktır. Bu denklem için $x,y,z$'nin aralarında asal olması ikişerli olarak da aralarında asal olması demektir. Şimdi $x$ ve $y$'nin $\mathbb{Z}[\omega]$'de aralarında asal olup olmadığını kontrol edelim. Şimdilik $xyz\neq 0$ olarak kabul edelim.

$\alpha\in\mathbb{Z}[\omega]$ olmak üzere $\alpha\mid x$ ve $\alpha \mid y$ olsun. $\alpha=a+b\omega$ ve $\text{EBOB}(a,b)=d$ olsun. $\alpha\beta=x$ ve $\alpha=d\theta$ olacak şekilde bir $\beta,\theta\in \mathbb{Z}[\omega]$ vardır. Dolayısıyla $d\cdot (\beta\theta)=x$ olur. $\beta\theta\in\mathbb{Z}[\omega]$ ve $d,x\in\mathbb{Z}$ olduğundan $\beta\theta\in\mathbb{Z}$ olmalıdır ve buradan tamsayılar için $d\mid x$ bulunur. Benzer şekilde $y$ için de aynısını yapabiliriz. $x$ ve $y$ aralarında asal olduğundan $d=1$ ve $a$ ve $b$ aralarında asal olmalıdır. $\alpha \neq 0$ olduğundan $\text{EBOB}(a,b)=1$ olmasında istisna olabilecek durumlar $(a,b)=(0,\pm 1)$ ve $(a,b)=(\pm 1,0)$'dır. İki durumda da $\alpha$ birim çıkar. O yüzden $a,b\neq 0$ diyebiliriz. $\beta=c+d\omega$ dersek $$\alpha\beta=(a+b\omega)(c+d\omega)=(ac-bd)+(ad+bc-bd)\omega\in \mathbb{Z}\implies ad+bc=bd$$ $a$ ve $b$'yi aralarında asal bulduğumuzdan son eşitlikten $b\mid d$ elde edilir. $t\in \mathbb{Z}$ için $d=bt$ yazıp $b$'leri sadeleştirirsek $at+c=bt$ olur. $c=(b-a)t$ ve $d=bt$ olduğundan $$\alpha\beta=ac-bd=t(ab-a^2-b^2)=x$$ elde edilir. $a^2+b^2-ab\geq 1$ olduğunu yukarıda bağlantısını bıraktığım gönderide 3. teoremin ispatından görebilirsiniz. $a^2+b^2-ab$ ifadesi hem $x$ hem de $y$'yi böldüğünden $1$'e eşit olmalıdır. $$1=a^2+b^2-ab\geq a^2+b^2-|ab|\geq |ab|\geq 0$$ olmasını kullanarak bu denklemi çözebiliriz. Her durumda $\alpha$ birim olarak bulunur. Yani $x$ ve $y$ için $\mathbb{Z}[\omega]$'da da aralarında asal diyebiliriz.

Ana denkleme dönersek bu denklemi $\mathbb{Z}[\omega]$'da çarpanlarında ayırırsak $$x^3+y^3=(x+y)(x+\omega y)(x+\omega^2 y)=(x+y)(x+\omega y)(x-y-\omega y)=z^3$$ olarak buluruz. Şimdi $(x+y)$, $(x+\omega y)$ ve $(x-y-\omega y)$ ifadelerinin ortak bölenlerini inceleyelim. Aralarında asal olmaları durumunu kenarda tutalım ve farz edelim ki yukarıdakinden farklı bir sıfırdan farklı ve birim olmayan $\alpha$ hem $x+y$ hem de $x+\omega y$'yi bölüyor. Dolayısıyla ikisinin farklı olan $(\omega -1)y$'yi de bölecektir. $\omega -1$ indirgenemezdir (aksini kabul edip kolayca çelişki bulabilirsiniz veya norm kavramını bilen kişiler normunun $3$ olmasından dolayı bunu kolaylıkla görecektir). Eğer $\alpha$ ile $y$ aralarında asal değilse herhangi bir ortak bölenleri $x$ de böleceğinden çelişki çıkar. Dolayısıyla $\alpha=\omega-1$ olmalıdır. Eğer $\omega-1\mid x+y$ ise diğer çarpanları da böldüğünü görebiliriz. O halde elimizde iki ihtimal vardır. $\mathbb{Z}[\omega]$ bir UFD olduğundan ikişerli olarak aralarında asal $u_1,u_2,u_3\in\mathbb{Z}[\omega]$ için

$(x+y,x+\omega y, x-y-\omega y)=(u_1^3,u_2^3,u_3^3)$ veya $(x+y,x+\omega y, x-y-\omega y)=((\omega-1)u_1^3,(\omega-1)u_2^3,(\omega-1)u_3^3)$ olacaktır. İkinci durumu önce inceleyelim. $u_1=a+b\omega$ dersek $(\omega-1)(a+b\omega)^3$ tamsayı olması gerektiğinden $$(\omega-1)((a^3-3ab^2+b^3)+(3a^2b-3ab^2)\omega)=(-a^3-3a^2b+6ab^2-b^3)+(a^3-6a^2b+3ab^2+b^3)\omega\in\mathbb{Z}\implies a^3-6a^2b+3ab^2+b^3=0$$ Eğer $b=0$ ise $a=0$ bulunur. Bu durumda da $z=0$ olur. Çelişki. $b\neq 0$ için her tarafı $b^3$'e bölüp $\dfrac{a}{b}=t$ dersek $$t^3-6t^2+3t+1=0$$ elde edilir ama bu denklemin rasyonel kökü yoktur. Buradan çözüm gelmez.

Eğer $(x+y,x+\omega y,x-y-\omega y)=(u_1^3,u_2^3,u_3^3)$ ise $u_2=c+d\omega$ diyelim. Buradan $$u_2^3=x+y\omega=(c^3-3cd^2+d^3)+(3c^2d-3cd^2)\omega\implies y=3c^2d-3cd^2$$ elde edilir. Yani $3\mid y$'dir. Eğer $x^3+y^3$ ifadesini $(x+y)(y+\omega x)(y-x-\omega x)$ olarak çarpanlarına ayırsaydık, $x$ de $3$'ün katı bulunacaktı. Bu $x$ ve $y$'nin aralarında asal olmasıyla çelişir.

Yani $xyz=0$ olmalıdır. İfadede sırayla $x=0$, $y=0$ ve $z=0$'ı denersek $k\in \mathbb{Z}$ için $(x,y,z)=(0,k,k), (k,0,k),(k,-k,0)$ çözümleri elde edilir. Tüm çözümler bunlardır.
Başlık: Ynt: Diyafont Denklemler Çalışma Soruları ($138$ Tane)
Gönderen: Metin Can Aydemir - Haziran 09, 2022, 09:18:20 ös
97) Verilen eşitliği bir polinoma çevirelim. $$P(t)=t^{x_{2011}}+t^{x_{2010}}+\cdots+t^{x_1}-t^{x_0}$$ polinomunun pozitif tamsayı olan bir kökü olmasını istiyoruz. $P(1)=2010$ ve $P(m)=0$ olduğundan ve tüm katsayılar tamsayı olduğundan $$m-1\mid 2010$$ olacaktır. $2010=2\cdot 3\cdot 5\cdot 67$ olduğundan olası $16$ adet $m$ pozitif tamsayısı vardır. Bu değerlerin hepsi için şarta uygun bir $x_i$'ler elde edebileceğimizi göstersek yeterlidir. Öncelikle $$\underbrace{m^i+m^i+\cdots+m^i}_{m\text{  adet}}=m^{i+1}$$ olduğunu gözlemleyelim. $(m-1)k=2010$ dersek $$\underbrace{m^0+m^0+\cdots+m^0}_{m\text{  adet}}=m^1$$ $$\underbrace{m^1+m^1+\cdots+m^1}_{m\text{  adet}}=m^{2}$$ $$\vdots$$ $$\underbrace{m^{k-1}+m^{k-1}+\cdots+m^{k-1}}_{m\text{  adet}}=m^{k}$$ eşitliklerini toplarsak $$\underbrace{m^0+m^0+\cdots+m^0}_{m\text{  adet}}+\underbrace{m^1+m^1+\cdots+m^1}_{(m-1)\text{  adet}}+\cdots+\underbrace{m^{k-1}+m^{k-1}+\cdots+m^{k-1}}_{(m-1)\text{  adet}}=m^{k}$$ elde edilir. Son eşitliğin sol tarafında $m+(m-1)(k-1)=(m-1)k+1=2011$ adet $m$'nin kuvveti vardır. Dolayısıyla yukarıda bulduğumuz $16$ adet sayının hepsi için istenilen $x_i$ negatif olmayan tamsayılarını bulabiliriz. Cevap $\boxed{16}$'dır.
Başlık: Ynt: Diyafont Denklemler Çalışma Soruları ($138$ Tane)
Gönderen: matematikolimpiyati - Ağustos 19, 2023, 06:39:05 öö
$98)$

Vieta teoreminden $a+b+c=0$ ve $ab+bc+ca=-2011$ yazalım. Kökler toplamı $0$ olduğu için köklerin üçü birden pozitif veya üçü birden negatif olamaz. Ya iki pozitif bir negatif kök vardır ya da tam tersi durum söz konusudur. Genelliği bozmadan $a\geq b \geq 0 \geq c$ kabul edelim. $c=-a-b$ değerini diğer denklemde yerine yazarsak $$ab+b(-a-b)+a(-a-b)=-2011 \implies a^2+ab+b^2=2011 \ (*)$$ elde ederiz. Öte yandan $$b \leq a \implies 3b^2 = b^2+b^2+b^2 \leq a^2+ab+b^2 = 2011 \implies b \leq 25$$ bulunur. Şimdi $(*)$ denklemini $4$ ile çarpıp düzenlersek $$(2a+b)^2=8044-3b^2$$ olur ki buradan da $8044-3b^2$ ifadesinin bir tamkare olduğunu söyleyebiliriz. Tamkare ifadeler $\pmod 5$ te $0,1,4$ değerlerini alabilir.
Böylece $b \in \{0,1,4,5,6,9,10,11,14,15,16,19,20,21,24,25\}$ olur. $(1)$

Şimdi ifadeyi $\pmod 7$ de inceleyelim. Tamkare ifadeler $\pmod 7$ de $0,1,2,4$ değerlerini alabilir.
Buradan da $b \in \{0,3,4,7,10,11,14,17,18,21,24,25\}$ elde edilir. $(2)$

$(1)$ ve $(2)$ den ortak olanları alırsak $b \in \{0,4,10,11,14,21,24,25\}$ buluruz. Bunların içinden $8044-3b^2$ ifadesini tamkare yapan tek değer $b=10$ dur.
$(*)$ denkleminde yerine yazarsak $a=39$ buluruz ve $c=-a-b=-39-10=-49$ elde ederiz.

Sonuç olarak sorumuzun cevabı $|a|+|b|+|c|=39+10+49= \boxed{98}$ dir.
Başlık: Ynt: Diyafont Denklemler Çalışma Soruları ($138$ Tane)
Gönderen: matematikolimpiyati - Ağustos 23, 2023, 04:26:43 öö
$110)$

$5x^3 \equiv 3 \pmod 7 \implies x^3 \equiv 2 \pmod 7$ olur ki bu da imkansızdır çünkü hiçbir tam sayının küpü $7$'ye bölündüğünde $2$ kalanını vermez. (Tamküp ifadelerin $7$ ile bölümünden kalanlar $0,1,6$ olabilir) Dolayısıyla denklemin tam sayılarda çözümü yoktur.
Başlık: Ynt: Diyafont Denklemler Çalışma Soruları ($138$ Tane)
Gönderen: matematikolimpiyati - Ağustos 23, 2023, 05:18:34 öö
$109)$

$x=0$ için $y=2$ olur. $x \geq 1$ olursa
$$(x+1)^3 < \underbrace{x^3+8x^2-6x+8}_{y^3} < (x+3)^3 \implies x+1 < y < x+3$$
elde edilir. Tam sayı çözüm bulunabilmesi için $y=x+2$ olmalıdır.

$\implies (x+2)^3=y^3=x^3+8x^2-6x+8 \implies x^3+6x^2+12x+8=x^3+8x^2-6x+8 \implies 2x^2=18x \implies x=9$ ve yerine yazarsak $y=11$ bulunur.

Denklemin tüm çözümleri $(0,2)$ ve $(9,11)$ ikilileridir.
Başlık: Ynt: Diyafont Denklemler Çalışma Soruları ($138$ Tane)
Gönderen: matematikolimpiyati - Ağustos 23, 2023, 05:34:49 öö
$104)$

$m^3+6m^2+5m=m(m^2+6m+5)=m(m+1)(m+5) \equiv m(m+1)(m+2) \pmod 3$ ve ardışık üç sayıdan bir tanesi $3$'e tam bölüneceğinden eşitliğin sol tarafı $3$'ün tam katıdır. Öte yandan eşitliğin sağ tarafının $3$'e bölümünden kalan $1$ olduğu için denklemin tam sayılarda çözümü yoktur.
Başlık: Ynt: Diyafont Denklemler Çalışma Soruları ($138$ Tane)
Gönderen: matematikolimpiyati - Ağustos 23, 2023, 04:05:32 ös
$102)$

Denklemi $\pmod 9$ da incelersek $x^3 \equiv 5 \pmod 9$ olur ki bu da imkansızdır çünkü hiçbir tam sayının küpü $9$'a bölündüğünde $5$ kalanını vermez. (Tamküp ifadelerin $9$ ile bölümünden kalanlar $0,1,8$ olabilir) Dolayısıyla denklemin tam sayılarda çözümü yoktur.
Başlık: Ynt: Diyafont Denklemler Çalışma Soruları ($138$ Tane)
Gönderen: matematikolimpiyati - Ağustos 23, 2023, 04:23:37 ös
$118)$

$x \in \mathbb Z$ olmak üzere $x^5 \equiv 0, \pm 1 \pmod {11}$ ve $y \in \mathbb Z$ olmak üzere $y^2 \equiv 0,1,3,4,5,9 \pmod {11}$ yazabiliriz. Bu değerleri denklemde yerine koyup $\pmod {11}$ de incelediğimiz zaman $x^5-y^2 \not \equiv 4 \pmod {11}$ elde ederiz. Dolayısıyla denklemin tam sayılarda çözümü yoktur.
Başlık: Ynt: Diyafont Denklemler Çalışma Soruları ($138$ Tane)
Gönderen: Hüseyin Yiğit EMEKÇİ - Ağustos 23, 2023, 05:10:36 ös
$105)$

$i)$ $x\geq 4$
Bu durumda sayımızın birler basamağı $1+2+6+24$ olduğundan 3 ile biter. Fakat bir tamkare, (mod 10) da ${0,1,4,5,6,9}$ değerlerini alabilir.
$ii)$ $x<4$
Bu durumda sadece $x=1$ ve $x=3$ durumlarında y bir tamsayı olur.

Yani sorumuzun cevabı tamsayılarda $(x,y)=(1,±1),(3,±3)$ tür.
Başlık: Ynt: Diyafont Denklemler Çalışma Soruları ($138$ Tane)
Gönderen: matematikolimpiyati - Ağustos 23, 2023, 06:59:31 ös
$136)$

$m$ ve $n$'ye göre $7$ durumda inceleyelim:

$\color{red}{i)}$ $m,n<0$ olsun. $m=-a$ ve $n=-b$ diyelim ($a,b>0$). Bu durumda $|12^m-5^n|=|12^{-a}-5^{-b}|=\left|\dfrac{1}{12^a}-\dfrac{1}{5^b}\right| = \left| \dfrac{5^b-12^a}{12^a \cdot 5^b} \right|$ olur ki bu ifade tam sayı olamaz çünkü pay kısmı tek, payda kısmı ise çifttir. Buradan çözüm gelmez.

$\color{red}{ii)}$ $m>0,n<0$ (veya $m<0,n>0$) olsun. Bu durumda da $12^m$ ve $5^n$ ifadelerinden birisi tam sayı olurken diğeri tam sayı olmaz ve farkları da tam sayı olamaz. Dolayısıyla bu durumdan da çözüm gelmez.

$\color{red}{iii)}$ $m,n>0$ olsun. $12^m$ çift ve $5^n$ tek olduğu için aradığımız ifadenin sonucu tek sayı olmalıdır. Alt durumlara bakalım:

   $\color{blue}{a)}$ $12^m-5^n=1 \implies 12^m-5^n \equiv 1 \pmod 4 \implies -1 \equiv 1 \pmod 4$ çelişki

   $\color{blue}{b)}$ $12^m-5^n=-1 \implies 12^m-5^n \equiv -1 \pmod{11} \implies 5^n \equiv 2 \pmod{11}$ çelişki ( $x \in \mathbb Z$ olmak üzere $5^x \equiv 1,3,4,5,9 \pmod{11}$ dir)

   $\color{blue}{c)}$ $12^m-5^n= \pm 3 \implies 12^m-5^n \equiv \pm 3 \pmod 3 \implies 5^n \equiv 0 \pmod 3$ çelişki

   $\color{blue}{d)}$ $12^m-5^n= \pm 5 \implies 12^m-5^n \equiv \pm 5 \pmod 5 \implies 2^m \equiv 0 \pmod 5$ çelişki

   $\color{blue}{e)}$ $12^m-5^n=\boxed{7}$ yi elde edebilmek için $m=n=1$ kullanabiliriz.

$\color{red}{iv)}$ $m=n=0$ için $|12^m-5^n|=\boxed{0}$ olur.

$\color{red}{v)}$ $m=0, n>0$ olsun. Bu durumda $|12^m-5^n|=|1-5^n|=5^n-1$ olup ifade kesin artan olduğundan dolayı alabileceği en küçük değer $5^1-1=\boxed{4}$ tür.

$\color{red}{vi)}$ $n=0,m>0$ olsun. Bu durumda $|12^m-5^n|=|12^m-1|=12^m-1$ olup ifade kesin artan olduğundan dolayı alabileceği en küçük değer $12^1-1=\boxed{11}$ dir.

$\color{red}{vii)}$ $m=0,n<0$ veya $n=0,m<0$ durumlarında $12^m$ ve $5^n$ ifadelerinden birisi $1$ olup diğeri tam sayı olmayacağı için farkları da tam sayı olamaz. Dolayısıyla bu durumdan da çözüm gelmez.

Sonuç olarak $m,n$ tam sayıları için $|12^m-5^n|$ ifadesinin alabileceği en küçük pozitif tam sayı değeri $\boxed{4}$'tür.
Başlık: Ynt: Diyafont Denklemler Çalışma Soruları ($138$ Tane)
Gönderen: matematikolimpiyati - Ağustos 26, 2023, 03:57:29 öö
$134)$

$a_1a_2+a_2a_3+ \cdots +a_na_1$ toplamındaki $n$ tane terimin her birisi $1$ veya $-1$'dir. Yani tüm terimler tek sayıdır. Bu tek sayıların toplamının $0$ yani bir çift sayı olabilmesi için çift adette bulunmaları gerekir(Örneğin, $6$ tane tek sayının toplamı çift sayıdır veya $22$ tane tek sayının toplamı çift sayıdır ama $5$ tane tek sayının toplamı çift değildir yine tektir. $6,22 \to$ çift sayı). Dolayısıyla $n$ çift sayıdır. $n=2k$ diyelim. Toplamda bulunan $2k$ tane terimin ($1$ ve $-1$ ler) toplamının $0$ olabilmesi için $k$ tanesinin $1$, $k$ tanesinin de $-1$ olması gerekir. Diğer taraftan bu $2k$ tane terimin çarpımı
$$(1)^k(-1)^k=(a_1a_2)(a_2a_3) \cdots (a_na_1)=a_1^2a_2^2 \cdots a_n^2=1$$
dir. Buradan da $k$ çift sayıdır. $k=2m$ yazarsak $n=2k=4m$ elde edilir ve $4 \mid n$ bulunmuş olur.
Başlık: Ynt: Diyafont Denklemler Çalışma Soruları ($138$ Tane)
Gönderen: matematikolimpiyati - Ağustos 31, 2023, 01:24:03 öö
$113)$

Denklemi $5$ ile çarpıp düzenleyelim:
$$25m^2-30mn+35n^2=9925 \implies 25m^2-30mn+9n^2+26n^2=9925 \implies (5m-3n)^2=9925-26n^2 \tag{1}$$
$(1)$ denkleminin sağ tarafındaki ifade tek bir sayı olup aynı zamanda bir tamkaredir. Bu tek sayıya $2k+1$ diyelim ve $(1)$'de yerine yazalım:
$$9925-26n^2=(2k+1)^2=4k^2+4k+1 \implies 2k^2+2k=4962-13n^2 \tag{2}$$
$(2)$ denkleminin sol tarafı çift olduğundan sağ tarafının da çift olması gerekir buradan da $n$'nin çift olduğunu söyleyebiliriz. $n=2p$ diyelim ve $(2)$'de yerine yazalım:
$$2k^2+2k=4962-52p^2 \implies k^2+k=2481-26p^2 \tag{3}$$
$(3)$ denkleminin sol tarafı çifttir ($k^2+k=k(k+1)$ olup ardışık iki sayıdan birisi çifttir) fakat sağ taraf tek olduğu için çelişki elde ederiz dolayısıyla $5m^2-6mn+7n^2=1985$ denkleminin tam sayılarda çözümü yoktur.
SimplePortal 2.3.3 © 2008-2010, SimplePortal